Flight Dispatcher Licence Review Questions
Flight Dispatcher Licence Review Questions
[1]
What is the name of a plane beyond the end of a runway which does not contain obstructions and can be
considered when calculating takeoff performance of turbine-powered aircraft?
A. Clearway.
B. Stopway.
C. Obstruction clearance plane.
Answer (A) is correct. A clearway is an area beyond the runway, not less than 500 ft. wide, centrally
located about the extended centerline of the runway and under the control of the airport authorities. The
clearway is expressed in terms of a clearway plane, extending from the end of the runway with an
upward slope not exceeding 1.25%, above which no object or terrain protrudes. However, threshold
lights may protrude above the plane if their height above the end of the runway is 26 in. or less and if
they are located on each side of the runway.
Answer (B) is incorrect because a stopway is an area beyond the takeoff runway, no less wide than the
runway and centered upon the extended centerline of the runway, able to support the airplane during an
aborted takeoff.
Answer (C) is incorrect because an obstruction clearance plane is used in calculating the effective length
of a runway for landing limitations, not for calculating takeoff performance in a turbine-powered aircraft.
[2]
Which is a definition of the term "crewmember"?
A. Only a pilot, flight engineer, or flight navigator assigned to duty in an aircraft during flight time.
B. A person assigned to perform duty in an aircraft during flight time.
C. Any person assigned to duty in an aircraft during flight except a pilot or flight engineer.
Answer (A) is incorrect because crewmember includes cabin crew as well as cockpit crew.
Answer (B) is correct. A crewmember is a person assigned to perform duty in an aircraft during flight
time.
Answer (C) is incorrect because crewmember includes cockpit crew as well as cabin crew.
[3]
Regulations concerning the operational control of a flight refer to
A. the specific duties of any required crewmember.
B. exercising authority over initiating, conducting, or terminating a flight.
C. exercising the privileges of pilot in command of an aircraft.
Answer (A) is incorrect because crewmember refers to any person assigned to perform duty in an aircraft
during flight time, which includes cabin crew as well as cockpit crew.
Answer (B) is correct. Operational control of a flight refers to the exercise of authority over initiating,
conducting, or terminating a flight.
Answer (C) is incorrect because pilot in command refers to the pilot responsible for the operation and
safety of an aircraft during flight time.
[4]
What is an area identified by the term "stopway"?
A. An area, at least the same width as the runway, capable of supporting an airplane during a normal takeoff.
B. An area designated for use in decelerating an aborted takeoff.
C. An area, not as wide as the runway, capable of supporting an airplane during a normal takeoff.
Answer (A) is incorrect because an area, at least the same width as the runway, capable of supporting
an airplane during a normal takeoff describes the non-landing portion of a runway, which is before a
displaced threshold and may be suit able for taxiing, landing rollout, and takeoff of aircraft.
Answer (B) is correct. A stopway is an area beyond the takeoff runway, no less wide than the runway and
centered upon the extended center-line of the runway, able to support the airplane during an aborted
takeoff without causing structural damage to the airplane.
Answer (C) is incorrect because an area, not as wide as the runway, capable of supporting an airplane
during a normal takeoff describes an area that exists before a displaced threshold.
[5]
Which is the correct symbol for design cruising speed?
A. VC.
B. VS.
C. VMA.
Answer (A) is correct. VC is design cruising speed. Design cruising speed is established by the
manufacturer and is used to define the flight envelope parameters in the aircraft certification.
Answer (B) is incorrect because VS is the stalling speed or minimum steady flight speed at which the
airplane is controllable.
Answer (C) is incorrect because VMA is a nonsense concept in that it is not defined.
[6]
Which is a definition of V2 speed?
A. Takeoff decision speed.
B. Takeoff safety speed.
C. Minimum takeoff speed.
Answer (A) is incorrect because V2 is takeoff safety speed, not takeoff decision speed.
Answer (B) is correct. V2 is takeoff safety speed. Takeoff safety speed means a referenced airspeed
obtained after liftoff at which the required one-engine-inoperative climb performance can be achieved.
Answer (C) is incorrect because V2MIN, not V2, is minimum takeoff safety speed.
[7]
Which speed symbol indicates the maximum operating limit speed for an airplane?
A. VLE.
B. VMO/MMO.
C. VLO /MLO.
Answer (A) is incorrect because VLE is maximum landing gear extended speed.
Answer (B) is correct. VMO/MMO is maximum operating limit speed. Maximum operating limit speed is also
known as maximum Mach number, which is analogous to VNE of a small airplane.
Answer (C) is incorrect because VLO is the maximum speed for operating the landing gear.
[8]
What is the correct symbol for minimum unstick speed?
A. VMU.
B. VMD.
C. VFC.
Answer (A) is correct. VMU is minimum unstick speed. Minimum unstick speed is the minimum airspeed at
which the airplane can be made to take off without demonstrating hazardous characteristics while
continuing the takeoff.
Answer (B) is incorrect because VMD is a nonsense concept in that it is not defined.
Answer (C) is incorrect because VFC/MFC is maximum speed for stability characteristics.
[9]
Which is the correct symbol for the stalling speed or the minimum steady flight speed at which the airplane is
controllable?
A. VS0.
B. VS.
C. VS1.
Answer (A) is incorrect because VS0 is the stalling speed or the minimum steady flight speed in the
landing configuration.
Answer (B) is correct. VS is the stalling speed or the minimum steady flight speed at which the airplane is
controllable.
Answer (C) is incorrect because VS1 is the stalling speed or the minimum steady flight speed obtained in
a specific configuration.
[10]
Which is the correct symbol for the minimum steady-flight speed or stalling speed in the landing configuration?
A. VS.
B. VS1.
C. VS0.
Answer (A) is incorrect because V S is the stalling speed or the minim um steady flight speed at which the
airplane is controllable.
Answer (B) is incorrect because V S1 is the stalling speed or the minimum steady flight speed obtained in
a specific configuration.
Answer (C) is correct. VS0 is the stalling speed or the minimum steady flight speed in the landing
configuration.
[11]
The symbol for the speed at which the critical engine is assumed to fail during takeoff is
A. V2.
B. V1.
C. VEF.
Answer (A) is incorrect because V2 is the takeoff safety speed, not the speed at which the critical engine
is assumed to fail during takeoff.
Answer (B) is incorrect because V1 is the maximum speed in the takeoff at which the pilot must take the
first action to stop the airplane within the accelerate-stop distance, not the speed at which the critical
engine is assumed to fail during takeoff.
Answer (C) is correct. VEF means the speed at which the critical engine is assumed to fail during takeoff.
[12]
The maximum speed during takeoff that the pilot may abort the takeoff and stop the airplane within the
accelerate-stop distance is
A. V2.
B. VEF.
C. V1.
Answer (A) is incorrect because V2 is the takeoff safety speed, not the maximum speed during takeoff
that the pilot may abort the takeoff and stop the airplane within the accelerate-stop distance.
Answer (B) is incorrect because VEF means the speed at which the critical engine is assumed to fail
during takeoff, not the maximum speed during takeoff that the pilot may abort the takeoff and stop the
airplane within the accelerate-stop distance.
Answer (C) is correct. V1 means the maximum speed in the takeoff at which the pilot must take the first
action (e.g., apply brakes, reduce thrust, deploy speed brakes) to stop the airplane within the accelerate-
stop distance.
[13]
An applicant who is scheduled for a practical test for an airline transport pilot certificate, in an approved flight
simulator, is
A. required to have at least a current third-class medical certificate.
B. not required to have a medical certificate.
C. required to have a first-class medical certificate.
Answer (A) is incorrect because an applicant is required to have at least a third-class medical certificate if
the ATP practical test is conducted in an airplane, not a flight simulator.
Answer (B) is correct. When taking the practical test for an airline transport pilot certificate in a flight
simulator, an applicant is not required to have a medical certificate.
Answer (C) is incorrect because an applicant is required to have a first-class medical certificate to
exercise the privileges of an ATP certificate, not to take the ATP practical test.
[14]
An applicant who is taking a practical test for a type rating to be added to a commercial pilot certificate, in an
approved simulator, is
A. required to have a first-class medical certificate.
B. required to have a second-class medical certificate.
C. not required to have a medical certificate.
Answer (A) is incorrect because a person is required to have a first-class medical certificate to exercise
the privileges of an ATP certificate, not to take a practical test for a type rating in a flight simulator.
Answer (B) is incorrect because a person is required to have a second-class medical certificate to exercise
the privileges of a commercial pilot certificate, not to take a practical test for a type rating in a flight
simulator.
Answer (C) is correct. A person who is taking a practical test for a type rating to be added to a commercial
pilot certificate, in a flight simulator, is not required to have a medical certificate.
[15]
When a type rating is to be added to an airline transport pilot certificate, and the practical test is scheduled in an
approved flight training device and/or approved flight simulator, the applicant is
A. required to have at least a third-class medical certificate.
B. not required to have a medical certificate.
C. required to have a first-class medical certificate.
Answer (A) is incorrect because a third-class medical certificate is required if any part of the practical test
is conducted in an airplane, not if the practical test is conducted entirely in a flight simulator or flight
training device.
Answer (B) is correct. A person who is taking a practical test in a flight simulator or flight training device for
a type rating to be added to an ATP certificate is not required to have a medical certificate.
Answer (C) is incorrect because a person is required to have a first-class medical certificate to exercise
the privileges of an ATP certificate, not to take a practical test to add a type rating to an ATP certificate.
[16]
Unless otherwise authorized, when is the pilot in command required to hold a type rating?
A. When operating an aircraft that is certificated for more than one pilot.
B. When operating an aircraft having a gross weight of more than 12,500 pounds.
C. When operating a multiengine aircraft having a gross weight of more than 6,000 pounds.
Answer (A) is incorrect because, although an aircraft may require two pilots for its operation, only turbojet
aircraft and other aircraft in excess of 12,500 lb. require a type rating.
Answer (B) is correct. A person may not act as pilot in command of any turbojet or other large aircraft (an
aircraft of more than 12,500 lb. maximum certificated takeoff weight) unless (s)he holds a type rating for
that aircraft.
Answer (C) is incorrect because large aircraft are those aircraft in excess of 12,500 lb., not 6, 000 lb. They
can be either single-engine or multiengine aircraft.
[17]
What instrument flight time may be logged by a second in command of an aircraft requiring two pilots?
A. All of the time the second in command is controlling the airplane solely by reference to flight instruments.
B. One-half the time the flight is on an IFR flight plan.
C. One-half the time the airplane is in actual IFR conditions.
Answer (A) is correct. A pilot may log as instrument flight time only that time during which (s)he operates
the aircraft solely by reference to the flight instruments under actual or simulated instrument flying
conditions.
Answer (B) is incorrect because only when the pilot is flying in actual or simulated instrument flying
conditions as sole manipulator of the controls may (s)he log instrument flight time.
Answer (C) is incorrect because a pilot may log all flight time in which (s)he is sole manipulator of the
controls in IFR conditions as instrument flight time.
[18]
A person may not act as a crewmember of a civil aircraft if alcoholic beverages have been consumed by that
person within the preceding
A. 8 hours.
B. 12 hours.
C. 24 hours.
Answer (A) is correct. No person may act or attempt to act as a crewmember of a civil aircraft within 8 hr.
after the consumption of any alcoholic beverage, while under the influence of drugs or alcohol, or while
having .04 percent by weight or more alcohol in the blood.
Answer (B) is incorrect because a person is required to wait only 8 hr., not 12 hr., after the consumption of
alcohol to act as a crewmember provided that (s)he is not still under the influence.
Answer (C) is incorrect because a person is required to wait only 8 hr., not 24 hr., after the consumption of
alcohol to act as a crewmember provided that (s)he is not still under the influence.
[19]
When may ATC request a detailed report on an emergency even though a rule has not been violated?
A. When priority has been given.
B. Anytime an emergency occurs.
C. When the emergency occurs in controlled airspace.
Answer (A) is correct. Each pilot in command who is given priority by ATC in an emergency shall submit a
detailed report of that emergency within 48 hr. to the manager of that ATC facility, if requested by ATC.
Answer (B) is incorrect because, during an emergency when no rule has been violated, ATC may request
a detailed report only if priority is given, not anytime an emergency occurs.
Answer (C) is incorrect because, during an emergency when no rule has been violated, ATC may request
a detailed report only if priority is given, not anytime an emergency occurs.
[20]
Which checks and inspections of flight instruments or instrument systems must be accomplished before an aircraft
can be flown under IFR?
A. VOR within 30 days and altimeter systems and transponder within 24 calendar months.
B. ELT test within 30 days, altimeter systems within 12 calendar months, and transponder within 24 calendar
months.
C. Airspeed indicator within 24 calendar months, altimeter system within 24 calendar months, and transponder
within 12 calendar months.
Answer (A) is correct. No person may operate a civil aircraft under IFR unless (1) the VOR has been
operationally checked within the preceding 30 days and (2) the altimeter systems and transponder have
been tested and inspected and found to comply with the Authority’s standards within the preceding 24
calendar months.
Answer (B) is incorrect because ELTs do not have to be tested every 30 days, and the altimeter must be
checked along with the transponder every 24 calendar months, not 12 months.
Answer (C) is incorrect because the transponder must be inspected every 24 calendar months, not 12
months.
[21]
Which entry shall be recorded by the person performing a VOR operational check?
A. Frequency, radial and facility used, and bearing error.
B. Flight hours and number of days since last check, and bearing error.
C. Date, place, bearing error, and signature.
Answer (A) is incorrect because the frequency and radial used are not required entry items; however, date
and place must be entered along with signature.
Answer (B) is incorrect because flight hours and number of days since last check are not required entry
items; however, date, place, and signature must be included.
Answer (C) is correct. Each person making a VOR operational check shall enter the date, place, and
bearing error, and sign the aircraft log or other record.
[22]
What altitude is a pilot authorized to fly when cleared for an ILS approach? The pilot
A. may begin a descent to the procedure turn altitude.
B. must maintain the last assigned altitude until established on a published route or segment of the approach with
published altitudes.
C. may descend from the assigned altitude only when established on the final approach course.
Answer (A) is incorrect because descent to the procedure turn altitude can be commenced only when you
are established on that segment of the instrument approach.
Answer (B) is correct. When operating on an unpublished route or while being vectored to an ILS
approach, the pilot, upon receiving an approach clearance, shall maintain the last altitude assigned to
him/her until the aircraft is established on a segment of a published route or instrument approach
procedure unless a different altitude is assigned by ATC.
Answer (C) is incorrect because the pilot does not have to be established on the final approach course to
descend from the last assigned altitude, if (s)he is established on a published route segment with a
specified lower minimum altitude than the last assigned altitude.
[23]
When must the pilot initiate a miss ed approach procedure from an ILS approach?
A. At the DH when the runway is not clearly visible.
B. When the time has expired after reaching the DH and the runway environment is not clearly visible.
C. At the DH, if the visual references for the intended runway are not distinctly visible or anytime thereafter that
visual reference is lost.
Answer (A) is incorrect because the runway itself does not have to be visible at the DH to continue with the
approach. A pilot may use the approach lighting system to within 100 ft. of the TDZE or any of the
following: (1) threshold lights, (2) REIL lights, (3) VASI lights, (4) TDZ lights, or (5) runway lights.
Answer (B) is incorrect because, as soon as the DH is reached on an ILS approach, regardless of the
elapsed time, a missed approach procedure should be executed if visual references are not obtained or if,
at any time after the DH is reached, visual reference is lost.
Answer (C) is correct. Each pilot operating a civil aircraft shall immediately execute an appropriate missed
approach procedure upon arrival at the missed approach point (DH on an ILS approach) if the visual
runway references are not distinctly visible or at any time thereafter if those visual references are lost
below the DH.
[24]
Which facility may be substituted for the middle marker during a Category I ILS approach?
A. VOR/DME FIX.
B. Surveillance radar.
C. Compass locator.
Answer (A) is incorrect because VOR/DME can be substituted for the outer marker only, not the middle
marker.
Answer (B) is incorrect because surveillance radar may be substituted for the outer marker only, not the
middle marker.
Answer (C) is correct. The basic ground components of an ILS are the localizer, glide slope, outer marker,
and middle marker. A compass locator or precision radar may be substituted for the outer or middle
marker.
[25]
If a pilot is being radar vectored in IFR conditions and loses radio communications with ATC, what action should
be taken?
A. Fly directly to the next point shown on the IFR flight plan and continue the flight.
B. Squawk 7700 and climb to VFR on Top.
C. Fly direct to a fix, route, or airway specified in the vector clearance.
Answer (A) is incorrect because the route shown on the flight plan should be used only if an assigned
route, vector, or expected route has not been received.
Answer (B) is incorrect because a climb should be initiated only in order to establish the highest of either
the assigned, MEA, or expected altitude, and the transponder should be set to 7600, not 7700, to indicate
communications failure.
Answer (C) is correct. If two-way radio communications failure occurs in IFR weather conditions and the
pilot is being vectored by ATC, the pilot should proceed by the direct route from the point of radio failure to
the fix, route, or airway specified in the vector clearance.
[26]
While in IFR conditions, a pilot experiences two-way radio communications failure. Which route should be flown in
the absence of an ATC assigned route or a route ATC has advised to expect in a further clearance?
A. The most direct route to the filed alternate airport.
B. An off-airway route to the point of departure.
C. The route filed in the flight plan.
Answer (A) is incorrect because, upon two-way radio communications failure, an approach and landing
should be made or attempted at the destination airport first before going to the filed alternate airport.
Answer (B) is incorrect because, in the absence of an assigned route or expected route, only the filed
route in the flight plan may be used. An off-airway route should not be used unless it was filed as part of
the flight plan.
Answer (C) is correct. If two-way radio communications failure occurs in IFR conditions, each pilot shall
continue with the flight according to the following : (1) by the route assigned in the last ATC clearance; (2)
if being radar vectored, by the direct route from the point of radio failure to the fix, route, or airway
specified in the vector clearance; (3) in the absence of an assigned route, by the route that ATC has
advised may be expected in a further clearance; or (4) by the route filed in the flight plan.
[27]
What altitude and route should be used if the pilot is flying in IFR weather conditions and has two-way radio
communications failure?
A. Continue on the route specified in the clearance and fly the highest of the following: the last assigned altitude,
altitude ATC has informed the pilot to expect, or to the MEA.
B. Descend to MEA and, if clear of clouds, proceed to the nearest appropriate airport. If not clear of clouds,
maintain the highest of the MEA's along the clearance route.
C. Fly the most direct route to the destination, maintaining the last assigned altitude or MEA, whichever is higher.
Answer (A) is correct. If radio communication failure occurs in IFR conditions and a clearance has been
received, the pilot should fly the route assigned in the last ATC clearance and the highest of either the
assigned altitude, MEA, or altitude to be expected when informed by ATC.
Answer (B) is incorrect because the highest of either the MEA, expected altitude, or assigned altitude
should be used.
Answer (C) is incorrect because, if ATC advises the pilot that (s)he may expect an altitude after reaching a
clearance limit, and it is higher than the published MEA or assigned altitude, the expected altitude should
be used. The route to be used should be the one assigned by C, as specified in a vector clearance, the
route ATC has advised may be expected, or in the absence of all of these, the route as filed in the flight
plan, not the most direct route.
[28]
A pilot is flying in IFR weather conditions and has two-way radio communications failure. What altitude should be
used?
A. Last assigned altitude, altitude AT C has advised to expect, or the M EA, whichever is highest.
B. An altitude that is at least 1,000 feet above the highest obstacle along the route.
C. A VFR altitude that is above the MEA for each leg.
Answer (A) is correct. If a pilot experiences two-way radio communications failure, (s)he should maintain
the highest of the following altitudes or flight levels for the route segment being flown: (1) the altitude or
flight level assigned in the last ATC clearance received, (2) the minimum altitude/flight level, or (3) the
altitude or flight level ATC has advised may be expected in a further clearance.
Answer (B) is incorrect because 1,000 ft. above the highest obstacle along the route is what a MOCA, or
minimum obstruction clearance altitude, provides.
Answer (C) is incorrect because VFR altitudes or regulations should never be used while flying in IFR
weather conditions. A VFR altitude should be used only in VFR weather conditions if VFR-on-top
clearance has been approved by ATC.
[29]
After experiencing two-way radio communications failure en route, when should a pilot begin the descent for the
instrument approach?
A. Upon arrival at any initial approach fix for the instrument approach procedure but not before the flight plan ETA
as amended by ATC.
B. Upon arrival at the holding fix depicted on the instrument approach procedure at the corrected ETA, plus or
minus
3 minutes.
C. At the primary initial approach fix for the instrument approach procedure at the ETA shown on the flight plan or
the EFC time, whichever is later.
Answer (A) is correct. If two-way radio communication failure occurs enroute and your clearance limit is
not a fix from which an approach begins, leave the clearance limit at the expect-further-clearance time if
one has been received, or if no EFC has been received, proceed to clearance limit, and proceed to the
appropriate initial approach fix. Commence descent and approach to coincide as closely as possible with
the ETA.
Answer (B) is incorrect because an approach should begin at the initial approach fix, not at a holding fix,
as close as possible to the ETA.
Answer (C) is incorrect because an EFC time supersedes a flight plan ETA and should be used if one has
been received (no matter if the EFC is sooner or later than the flight plan ETA).
[30]
A pilot is holding at an initial approach fix after having experienced two-way radio communications failure. When
should that pilot begin descent for the instrument approach?
A. At the EFC time, if this is within plus or minus 3 minutes of the flight plan ETA as amended by ATC.
B. At flight plan ETA as amended by ATC.
C. At the EFC time as amended by ATC.
Answer (A) is incorrect because the radio failure occurred prior to arrival at the initial approach fix, so it is
unlikely an expect-further-clearance (EFC) time would have been received. Additionally, the approach
should begin either at the EFC time or, if one has not been received, at flight plan ETA.
Answer (B) is correct. The radio failure was experienced before the pi lot entered the holding pattern at the
initial approach fix. Since the pilot would not have received an expect-further-clearance (EFC) time, (s)he
should commence descent for the instrument approach as close as possible to the estimated time of
arrival (ETA), as calculated from the filed or amended (with ATC) estimated time enroute.
Answer (C) is incorrect because the radio failure occurred before the pilot entered the holding pattern at
the initial approach fix; thus, (s)he would not have received holding instructions that would have contained
an EFC time.
[31]
While flying IFR in controlled airspace, if one of the two VOR receivers fails, which course of action should the pilot
in command follow?
A. No call is required if one of the two VOR receivers is operating properly.
B. Advise ATC immediately.
C. Notify the dispatcher via company frequency.
Answer (A) is incorrect because any malfunction of a navigational radio should be reported, no matter how
slightly it may affect the conduct of the flight.
Answer (B) is correct. The pilot in command of each aircraft operated in controlled airspace under IFR
shall report as soon as practicable to ATC any malfunction of navigational, approach, or communication
equipment occurring in flight.
Answer (C) is incorrect because, although the operations manual may require that the dispatcher be
notified, the pilot in command must report the failure to ATC immediately.
[32]
What action is necessary when a partial loss of ILS receiver capability occurs while operating in controlled
airspace under IFR?
A. Continue as cleared and file a written report to the Administrator if requested.
B. If the aircraft is equipped with other radios suitable for executing an instrument approach, no further action is
necessary.
C. Report the malfunction immediately to ATC.
Answer (A) is incorrect because any malfunction of approach equipment must be reported in flight, not by
a written report.
Answer (B) is incorrect because, although another type of instrument approach may be executed if
permission is granted by ATC, any malfunction of approach equipment should be reported.
Answer (C) is correct. The pilot in command of each aircraft operated in controlled airspace under IFR
shall report as soon as practicable to ATC any malfunction of navigational, approach, or communication
equipment occurring in flight.
[33]
What action should be taken if one of the two VHF radios fail while IFR in controlled airspace?
A. Notify ATC immediately.
B. Squawk 7600.
C. Monitor the VOR receiver.
Answer (A) is correct. The pilot in command of each aircraft operated in controlled airspace under IFR
shall report as soon as practicable to ATC any malfunction of navigational, approach, or communication
equipment occurring in flight.
Answer (B) is incorrect because, although a communications failure has been experienced, it is only a
partial one. One operational VHF radio is still available, and all other radios are working normally. Thus, a
squawk of 7600 is not needed.
Answer (C) is incorrect because an operable VHF radio is still available for communication. Monitoring a
NAVAID is not needed. The only pilot action required is notification to ATC of the problem.
[34]
A function of the minimum equipment list is to indicate required items which
A. are required to be operative for overwater passenger air carrier flights.
B. may be inoperative for a one-time ferry flight of a large airplane to a maintenance base.
C. may be inoperative prior to beginning a flight in an aircraft.
Answer (A) is incorrect because an MEL prescribes which instruments and equipment may be inoperative
prior to beginning a flight, not which items are required to be operative for overwater air carrier flights.
Answer (B) is incorrect because a special flight permit, not an MEL, authorizes a one-time ferry flight of a
large airplane to a maintenance base with inoperative instruments and/or equipment.
Answer (C) is correct. A function of an approved minimum equipment list (MEL) is to indicate the required
items (instruments and/or equipment) which may be inoperative prior to beginning a flight in an aircraft and
to state under what conditions the flight can be operated with that inoperative equipment.
[35]
Information obtained from flight data and cockpit voice recorders shall be used only for determining
A. who was responsible for any accident or incident.
B. evidence for use in civil penalty or certificate action.
C. possible causes of accidents or incidents.
Answer (A) is incorrect because flight data or cockpit voice recorders are used only to determine causes of
accidents or incidents, not to determine who was responsible or at fault.
Answer (B) is incorrect because flight data or cockpit voice recorders may not be used for any civil penalty
or certificate action.
Answer (C) is correct. Information obtained from flight data and cockpit voice recorders is used to assist in
determining the cause of accidents or occurrences.
[36]
For what purpose may cockpit voice recorders and flight data recorders NOT be us ed?
A. Determining causes of accidents and occurrences under investigation.
B. Determining any certificate action, or civil penalty, arising out of an accident or occurrence.
C. Identifying procedures that may have been conducive to any accident, or occurrence resulting in investigation
Answer (A) is incorrect because cockpit voice recorders and flight data recorders are used to determine
causes of accidents or occurrences.
Answer (B) is correct. Information obtained from flight data and cockpit voice recorders is used to assist in
determining the cause of accidents or occurrences.
Answer (C) is incorrect because flight data recorders and cockpit voice recorders are used to identify any
procedure, malfunction, or failure that may have contributed to an accident or occurrence.
[37]
How long is cockpit voice recorder and flight recorder data kept, in the event of an accident or occurrence resulting
in terminating the flight?
A. 60 days.
B. 90 days.
C. 30 days.
Answer (A) is correct. In the event of an accident or occurrence that results in the termination of the flight,
any operator who has installed approved flight recorders and cockpit voice recorders shall keep the
recorded information for at least 60 days or, if requested, for a longer period.
Answer (B) is incorrect because the information need be kept for only 60 days, not 90 days.
Answer (C) is incorrect because the information must be kept for a minim um of 60 days, not 30 days.
[38]
Which operational requirement must be observed when ferrying an air carrier airplane when one of its three
turbine engines is inoperative?
A. The weather conditions at takeoff and destination must be VFR.
B. The flight cannot be conducted between official sunset and official sunrise.
C. Weather conditions must exceed the basic VFR minimums for the entire route, including takeoff and landing.
Answer (A) is correct. The holder of an air carrier operating certificate may conduct a ferry flight of an
aircraft equipped with three turbine engines, one of which is inoperative, to a base for the purpose of
repairing that engine only if the weather conditions at the takeoff and destination airports are VFR.
Answer (B) is incorrect because a ferry flight may be conducted after sunset and before sunrise as long as
the takeoff and destination airports are VFR.
Answer (C) is incorrect because the weather conditions must be VF R only for takeoff and landing, not
over the entire route.
[39]
When a turbine-engine-powered airplane is to be ferried to another base for repair of an inoperative engine, which
operational requirement must be observed?
A. Only the required flight crewmembers may be on board the airplane.
B. The existing and forecast weather for departure, enroute, and approach must be VFR.
C. No passengers except authorized maintenance personnel may be carried.
Answer (A) is correct. The holder of an air carrier operating certificate may conduct a ferry flight of an
aircraft with one of its turbine engines inoperative to a base for the purpose of repairing that engine with
only the required flight crewmembers on board the airplane.
Answer (B) is incorrect because the weather conditions must be VFR only for takeoff and landing, not over
the entire route.
Answer (C) is incorrect because only the required flight crewmembers may be aboard. Maintenance
personnel, mechanics, and others may not be carried.
[40]
Which operational requirement must be observed by a commercial operator when ferrying a large, three-engine,
turbojet-powered airplane from one facility to another to repair an inoperative engine?
A. The computed takeoff distance to reach V1 must not exceed 70 percent of the effective runway length.
B. The existing and forecast weather for departure, enroute, and approach must be VFR.
C. No passengers m ay be carried.
Answer (A) is incorrect because runway length allowing V1 in less than 70% of the runway is not required
for ferry flights with one engine inoperative.
Answer (B) is incorrect because the weather conditions must be VFR only for takeoff and landing, not over
the entire route.
Answer (C) is correct. The holder of an air carrier operating certificate may conduct a ferry flight of an
aircraft with one of its turbine engines inoperative to a base for the purpose of repairing that engine with
only the required flight crewmembers on board the airplane. No passengers may be carried.
[41]
When a passenger notifies the certificate holder prior to checking baggage that an unloaded weapon is in the
baggage, what action is required by regulation regarding this baggage?
A. The baggage may be carried in the flight crew compartment, provided the baggage remains locked, and the key
is given to the pilot in command.
B. The baggage must remain locked and carried in an area that is inaccessible to the passenger, and only the
passenger retains the key.
C. The baggage must remain locked and stored where it would be inaccessible, and custody of the key shall
remain with a designated crewmember.
Answer (A) is incorrect because the baggage containing the unloaded weapon must be carried in an area
other than, not in, the flight crew compartment, and the person who checked the baggage, not the PIC,
should retain the key.
Answer (B) is correct. When a passenger notifies the certificate holder prior to checking baggage that an
unloaded weapon is in his/her baggage, that baggage must remain locked, and only the passenger who
checked the baggage may retain the key or combination. The baggage containing the weapon is carried in
an area, other than the flight crew compartment, that is inaccessible to passengers.
Answer (C) is incorrect because the person who checked the baggage, not a designated crewmember,
retains custody of the key.
[42]
A certificate holder is notified that a person specifically authorized to carry a deadly weapon is to be aboard an
aircraft. Except in an emergency, how long before loading that flight should the air carrier be notified?
A. Notification is not required, if the certificate holder has a security coordinator.
B. A minimum of 1 hour.
C. A minimum of 2 hours.
Answer (A) is incorrect because, except in an emergency, a certificate holder must be notified at a
minimum of 1 hr. before a flight, regardless of whether the certificate holder is required to have a security
coordinator.
Answer (B) is correct. Except in an emergency, a certificate holder should be notified at a minimum of 1 hr.
before the flight on which a person who is specifically authorized to carry a deadly weapon is to be aboard
that aircraft.
Answer (C) is incorrect because, except in an emergency, a certificate holder must be notified at a
minimum of 1 hr., not 2 hr., before a flight.
[43]
Which applies to the carriage of a person in the custody of law enforcement personnel?
A. The air carrier is not allowed to serve beverages to the person in custody or the law enforcement escort.
B. No more than one person considered to be in the maximum risk category may be carried on a flight, and that
person must have at least two armed law enforcement escorts.
C. The person in custody must be seated between the escort and the aisle.
Answer (A) is incorrect because the air carrier can serve nonalcoholic beverages to the law enforcement
escort. Additionally, food or beverage m ay be served to the person in custody with the authorization of the
law enforcement escort.
Answer (B) is correct. No more than one person who the certificate holder has been notified is in a
maximum risk category may be carried on the airplane, and that person must have at least two armed law
enforcement escorts.
Answer (C) is incorrect because the law enforcement escort must be seated between the person in
custody and the aisle; the person in custody is not to be seated between the escort and the aisle.
[44]
When a person in the custody of law enforcement personnel is scheduled on a flight, what procedures are required
regarding boarding of this person and the escort?
A. They shall be boarded before all other passengers board, and deplaned after all the other passengers have left
the aircraft.
B. They shall be boarded after all other passengers board, and deplaned before all the other passengers leave the
aircraft.
C. They shall board and depart before the other passengers.
Answer (A) is correct. Each person in the custody of law enforcement personnel will be boarded before all
other passengers board and deplaned after all other passengers have left the aircraft.
Answer (B) is incorrect because the person in custody will be boarded before, not after, all other
passengers board and deplaned after, not before, all other passengers leave the aircraft.
Answer (C) is incorrect because the person in custody will be deplaned after, not before, all other
passengers leave the aircraft.
[45]
An airport approved by the Administrator for use by an air carrier certificate holder for the purpose of providing
service to a community when the regular airport is not available is a/an:
A. destination airport.
B. provisional airport.
C. alternate airport.
Answer (A) is incorrect because a destination airport is the airport of intended landing when departing on a
flight, not necessarily an airport for use by an air carrier certificate holder for the purpose of providing
service to a community when the regular airport is not available.
Answer (B) is correct. Provisional airport is defined as an airport approved by the Administrator for use by
a certificate holder for the purpose of providing service to a community when the regular airport used by
the certificate holder is not available.
Answer (C) is incorrect because an alternate airport means an airport at which an aircraft may land if a
landing at the intended airport becomes inadvisable, not necessarily an airport approved for use by an air
carrier certificate holder for the purpose of providing service to a comm unity when the regular airport is
not available.
[46]
Which document specifically authorizes a person to operate an aircraft in a particular geographic area?
A. Operations Specifications.
B. Operating Certificate.
C. Dispatch Release.
Answer (A) is correct. No person may operate an aircraft in a geographical area unless its operations
specifications specifically authorize the certificate holder to operate in that area.
Answer (B) is incorrect because an operating certificate is issued to a business to conduct operations; it
does not state where the aircraft can operate.
Answer (C) is incorrect because a dispatch release authorizes a flight to begin, not the geographic area in
which a person can operate an aircraft.
[47]
The kinds of operation that a certificate holder is authorized to conduct are specified in the
A. certificate holder's operations specifications.
B. application submitted for an Air Carrier or Operating Certificate, by the applicant.
C. Air Carrier Certificate or Operating Certificate.
Answer (A) is correct. The kinds of operation that a certificate holder is authorized to conduct are specified
in the certificate holder's operations specifications
Answer (B) is incorrect because, while an applicant must inform the Authority as to what kinds of operation
(s)he wants to operate when applying for an Air Carrier or Operating Certificate, the kinds of operation that
a certificate holder is authorized to conduct are specified in the operations specifications.
Answer (C) is incorrect because the kinds of operation that a certificate holder is authorized to conduct are
specified in the operations specifications, not on the Air Carrier or Operating Certificate.
[48]
What effective runway length is required for a turbojet-powered airplane at the destination airport if the runways
are forecast to be wet or slippery at the ETA?
A. 70 percent of the actual runway available, from a height of 50 feet over the threshold.
B. 115 percent of the runway length required for a dry runway.
C. 115 percent of the runway length required for a wet runway.
Answer (A) is incorrect because 70% of the actual runway available is a landing limitation for a
turbopropeller airplane, not a turbojet-powered airplane landing at an airport that forecasts the runways to
be wet or slippery at the ETA.
Answer (B) is correct. No person may take off a turbojet-powered airplane when the appropriate weather
reports and forecasts, or a combination thereof, indicate that the runways at the destination airport may be
wet or slippery at the estimated time of arrival unless the effective runway length at the destination airport
is at least 115% of the runway length required for a dry runway.
Answer (C) is incorrect because, if the runways are forecast to be wet or slippery at the ETA, the effective
runway length for a turbojet-powered airplane is 115% of the runway length required for a dry, not wet,
runway.
[49]
What restrictions must be observed regarding the carrying of cargo in the passenger compartment of an airplane?
A. All cargo must be separated from the passengers by a partition capable of withstanding certain load stresses.
B. All cargo must be carried in a suitable flame-resistant bin and the bin must be secured to the floor structure of
the airplane.
C. Cargo may be carried aft of a divider if properly secured by a safety belt or other tiedown devices to withstand
certain load stresses.
Answer (A) is incorrect because cargo may be carried in the passenger compartment if it is carried in an
approved cargo bin that meets certain requirements. A partition separating the cargo from the passengers
is not required.
Answer (B) is incorrect because the cargo bin may be attached to either seat tracks or the floor structure,
not only the floor structure, of the airplane.
Answer (C) is correct. Cargo may be carried aft of a bulkhead or divider in any passenger compartment
provided the cargo is restrained and is properly secured by a safety belt or other tiedown devices having
enough strength to eliminate the possibility of shifting under all normally anticipated flight and ground
conditions.
[50]
What requirement must be met regarding cargo that is carried anywhere in the passenger compartment of an air
carrier airplane?
A. The bin in which the cargo is carried may not be installed in a position that restricts access to, or use of, any
exit. B. The bin in which the cargo is carried may not be installed in a position that restricts access to, or use of,
any aisle
in the passenger compartment.
C. The container or bin in which the cargo is carried must be made of material which is at least flash resistant.
Answer (A) is incorrect because the cargo bin may not be installed in a position that restricts access to, or
use of, any emergency exit, not any exit, in the airplane.
Answer (B) is correct. A cargo bin installed in the passenger compartment may not be installed in a
position that restricts access to, or use of, any emergency exit or aisle in the passenger compartment.
Answer (C) is incorrect because the container or bin in which the cargo is carried must be made of
material which is at least flame, not flash, resistant.
[51]
Which restriction applies to a cargo bin in a passenger compartment? The bin
A. may have an open top if it is placed in front of the passengers and the cargo is secured by a cargo net.
B. must withstand the load factor required of passenger seats, multiplied by 1.15, using the combined weight of the
bin and the maximum weight of the cargo that may be carried in the bin.
C. must be constructed of flame-retardant material and fully enclosed.
Answer (A) is incorrect because the cargo bin must be fully enclosed, not have an open top if it is placed in
front of the passengers.
Answer (B) is correct. A cargo bin installed in a passenger compartment must withstand the load factors
and emergency landing conditions applicable to the passenger seats of the airplane in which the bin is
installed, multiplied by a factor of 1.15, using the combined weight of the bin and the maximum weight of
cargo that may be carried in the bin.
Answer (C) is incorrect because the bin must be constructed of flame resistant, not retardant, material.
[52]
Where should the portable battery-powered megaphone be located if only one is required on a passenger-carrying
airplane?
A. The most forward location in the passenger cabin.
B. In the cabin near the over-the-wing emergency exit.
C. The most rearward location in the passenger cabin.
Answer (A) is incorrect because the portable battery-powered megaphone must be in the most rearward,
not forward, location in the passenger cabin.
Answer (B) is incorrect because the portable battery-powered megaphone must be in the most rearward
location in the cabin, not near the over-the-wing emergency exit.
Answer (C) is correct. One megaphone is required on each airplane with a seating capacity of more than
60 and less than 100 passengers at the most rearward location in the passenger cabin, where it would be
readily accessible to a normal flight attendant seat.
[53]
How many portable battery-powered megaphones are required on an air carrier airplane with a seating capacity of
100 passengers on a trip segment when 45 passengers are carried?
A. Two; one at the forward end, and the other at the most rearward location in the passenger cabin.
B. Two; one at the most rearward and one in the center of the passenger cabin.
C. Two; one located near or accessible to the flight crew, and one located near the center of the passenger cabin.
Answer (A) is correct. Two megaphones are required in the passenger cabin on each airplane with a
seating capacity of more than 99 passengers, one installed at the forward end and the other at the most
rearward location, where it would be readily accessible to a normal flight attendant seat.
Answer (B) is incorrect because one megaphone must be at the forward end, not in the center, and the
other in the most rearward location in the passenger cabin.
Answer (C) is incorrect because the megaphones are located in the passenger cabin, one at the forward
end, not necessarily accessible to the flight crew, and the other at the most rearward location, not the
center, of the passenger cabin.
[54]
How many portable battery-powered megaphones are required on an air carrier airplane with a seating capacity of
150 passengers on a trip segment when 75 passengers are carried?
A. Two; one located near or accessible to the flight crew, and one located near the center of the passenger cabin.
B. Two; one at the most rearward and one in the center of the passenger cabin.
C. Two; one at the forward end, and the other at the most rearward location of the passenger cabin.
Answer (A) is incorrect because one megaphone must be located at the forward end, not necessarily near
or accessible to the flight crew, and the other at the most rearward location, not near the center, of the
passenger cabin.
Answer (B) is incorrect because one megaphone must be at the forward end, not center, and the other at
the most rearward location in the passenger cabin.
Answer (C) is correct. Two megaphones are required in the passenger cabin on each airplane with a
seating capacity of more than 99 passengers, one installed at the forward end and the other at the most
rearward location, where it would be readily accessible to a normal flight attendant seat.
[55]
Which factor determines the minimum number of hand fire extinguishers required for flight?
A. Number of passengers and crewmembers aboard.
B. Number of passenger cabin occupants.
C. Airplane passenger seating accommodations.
Answer (A) is incorrect because the minimum number of hand fire extinguishers required is determined by
the airplane passenger seating accommodations, not the actual number of passengers and crew aboard a
given flight.
Answer (B) is incorrect because the minimum number of hand fire extinguishers required is determined by
the airplane passenger seating accommodations, not the actual number of passenger cabin occupants.
Answer (C) is correct. An airplane's passenger seating accommodations determine the minimum number
of hand fire extinguishers required for flight.
[56]
Which requirement applies to emergency equipment (fire extinguishers, megaphones, first-aid kits, and crash axe)
installed in an air carrier airplane?
A. All emergency equipment must be readily accessible to the passengers.
B. Emergency equipment cannot be located in a compartment or area where it is not immediately visible to a flight
attendant in the passenger compartment.
C. Emergency equipment must be clearly identified and clearly marked to indicate its method of operation.
Answer (A) is incorrect because only that emergency equipment located in the passenger compartment,
not all emergency equipment, must be readily accessible to passengers.
Answer (B) is incorrect because there is no requirement that the emergency equipment be visible to a
flight attendant in the passenger compartment. Emergency equipment can be carried in a compartment or
container as long as it is marked as to the contents.
Answer (C) is correct. All emergency equipment installed in an air carrier airplane must be clearly
identified and clearly marked to indicate its method of operation.
[57]
The emergency lights on a passenger-carrying airplane must be armed or turned on during
A. taxiing, takeoff, cruise, and landing.
B. taxiing, takeoff, and landing.
C. takeoff, cruise, and landing.
Answer (A) is incorrect because the emergency lights must be armed or turned on during taxiing, takeoff,
and landing, not cruise.
Answer (B) is correct. The emergency lights on a passenger-carrying airplane must be armed or turned on
during taxiing, takeoff, and landing.
Answer (C) is incorrect because the emergency lights must be armed or turned on during taxiing, takeoff,
and landing, not cruise.
[58]
If a passenger-carrying landplane is required to have an automatic deploying escape slide system, when must this
system be armed?
A. For taxi, takeoff, and landing.
B. Only for takeoff and landing.
C. During taxi, takeoff, landing, and after ditching.
Answer (A) is correct. Automatic deploying escape slide systems must be armed during taxi, takeoff, and
landing.
Answer (B) is incorrect because an automatic deploying escape slide system must be armed for taxi as
well as takeoff and landing.
Answer (C) is incorrect because an automatic deploying escape slide system may be deployed, not
armed, after ditching.
[59]
If there is a required emergency exit located in the flight crew compartment, the door which separates the
compartment from the passenger cabin must be
A. unlocked during takeoff and landing.
B. locked at all times, except during any emergency declared by the pilot in command.
C. latched open during takeoff and landing.
Answer (A) is incorrect because the door must always be latched open, not unlocked, during takeoff and
landing.
Answer (B) is incorrect because the door must always be latched open, not locked, during takeoff and
landing.
Answer (C) is correct. If it is necessary to pass through a doorway separating the passenger cabin from
other areas to reach any required emergency exit from any passenger seat, the door must have a means
to latch it in open position, and the door must be latched open during each takeoff and landing.
[60]
When may two persons share one approved safety belt in a lounge seat?
A. When one is an adult and one is a child under 3 years of age.
B. Only during the enroute flight.
C. During all operations except the takeoff and landing portion of a flight.
Answer (A) is incorrect because the regulations do not specify an age of persons sharing a seat belt on a
lounge seat. Sharing a seat belt in a lounge seat can be done only during enroute flight.
Answer (B) is correct. Two persons occupying a berth may share one approved safety belt and two
persons occupying a multiple lounge or divan seat may share one approved safety belt during enroute
flight only.
Answer (C) is incorrect because two persons may share one seat belt in a lounge seat only during the
enroute portion of the flight, which excludes taxi, takeoff, and landing.
[61]
An air carrier airplane must have an operating public address system if it
A. has a seating capacity of 19 passengers.
B. has a seating capacity for more than 19 passengers.
C. weighs more than 12,500 pounds.
Answer (A) is incorrect because an air carrier airplane must have an operating public address system if it
has a seating capacity of more than, not equal to, 19 passengers.
Answer (B) is correct. No person may operate an air carrier airplane with a seating capacity of more than
19 passengers unless it is equipped with an operating public address system.
Answer (C) is incorrect because an air carrier airplane is required to have an operating public address
system if it has a seating capacity of more than 19 passengers, not if it weighs more than 12, 500 lb.
[62]
A crewmember interphone system is required on which airplane?
A. A large airplane.
B. A turbojet airplane.
C. An airplane with more than 19 passenger seats.
Answer (A) is incorrect because the crewmember interphone system requirement is based up on the
number of seats, not the size of the airplane. A crewmember interphone system is required if the airplane
has more than 19 passenger seats.
Answer (B) is incorrect because the number of passenger seats, not the type of propulsion, determines
whether a crewmember interphone system is required.
Answer (C) is correct. No person may operate an airplane with a seating capacity of more than 19
passengers unless the airplane is equipped with a crewmember interphone system.
[63]
For a 2-hour flight in a reciprocating-engine-powered airplane at a cabin pressure altitude of 12,000 feet, how
much supplemental oxygen for sustenance must be provided? Enough oxygen for
A. 30 minutes for 10 percent of the passengers.
B. 10 percent of the passengers for 1.5 hours.
C. each passenger for 30 minutes.
Answer (A) is correct. For flights in a reciprocating-engine-powered airplane of more than 30 min. duration
at cabin pressure altitudes above 8,000 ft. up to and including 14,000 ft., enough oxygen must be provided
for 30 min. for 10% of the passengers.
Answer (B) is incorrect because enough oxygen for 10% of the passengers for 1.5 hr. is the oxygen
required for a 2-hr. flight at a cabin pressure altitude of 12,000 ft. for a turbine-engine-powered airplane,
not a reciprocating-engine-powered airplane.
Answer (C) is incorrect because there must be enough oxygen for 10% of the passengers, not each
passenger, for 30 mi n.
[64]
What is the passenger oxygen supply requirement for a flight, in a turbine-powered aircraft, with a cabin pressure
altitude in excess of 15,000 feet? Enough oxygen for
A. each passenger for the entire flight above 15,000 feet cabin altitude.
B. 30 percent of the passengers.
C. 10 percent of the passengers for 30 minutes.
Answer (A) is correct. For flights at cabin pressure altitudes above 15, 000 ft. in a turbine-powered
airplane, there must be enough oxygen available for each passenger carried during the entire flight at
those altitudes.
Answer (B) is incorrect because enough oxygen for 30% of the passengers is the oxygen supply
requirement for the time at cabin pressure altitudes above 14,000 ft., up to and including 15,000 ft., not
above 15,000 ft.
Answer (C) is incorrect because enough oxygen for 10% of the passengers is the oxygen supply
requirement for flights of more than 30 min. at cabin pressure altitudes above 10,000 ft., up to and
including 14,000 ft., not for cabin pressure altitudes above 15,000 ft.
[65]
At which cabin altitude must oxygen be provided for all passengers during the entire flight at those altitudes?
A. 15,000 feet.
B. 16,000 feet.
C. 14,000 feet.
Answer (A) is incorrect because, at a cabin pressure altitude of 15,000 ft., there must be enough oxygen
available for 30% of the passengers, not all passengers, during the entire flight at that altitude.
Answer (B) is correct. For flights at cabin pressure altitudes above 15,000 ft., there must be enough
oxygen available for all passengers during the entire flight at those altitudes. Thus, at a cabin pressure
altitude of 16,000 ft., there must be enough oxygen available for all passengers during the entire flight at
that altitude.
Answer (C) is incorrect because, at a cabin pressure altitude of 14,000 ft., there must be enough oxygen
available for 10% of the passengers, not all the passengers, for flight at that altitude that is more than 30
min. in duration, not for the entire flight at that altitude.
[66]
How much supplemental oxygen for emergency descent must a pressurized turbine-powered air transport airplane
carry for each flight crewmember on flight deck duty when operating at flight altitudes above 10,000 feet?
A. A minimum of 2-hours' supply.
B. Sufficient for the duration of the flight above 8,000 feet cabin pressure altitude.
C. Sufficient for the du ration of the flight at 10,000 feet flight altitude, not to exceed 1 hour and 50 minutes.
Answer (A) is correct. When operating a turbine-engine-powered airplane with a pressurized cabin at flight
altitudes above 10,000 ft., there must be a minimum of a 2-hr. supply of supplemental oxygen for each
flight crewmember on flight deck duty in the event of an emergency descent. The 2-hr. oxygen supply is
that required for a constant rate of descent from the airplane's maximum certificated operating altitude to
10,000 ft. in 10 min., followed by 1 hr. 50 min. at 10,000 ft.
Answer (B) is incorrect because oxygen requirements at a cabin pressure altitude above 8,000 ft. is for
passengers in reciprocating-engine-powered airplanes, not for flight crewmembers on a turbine-engine-
powered airplane in the event of an emergency descent.
Answer (C) is incorrect because a minimum of 2 hr. of supplemental oxygen is required for each flight
crewmember on flight deck duty. The minimum 2-hr. supply includes 10 min. for the descent to 10,000 ft.,
followed by 1 hr. 50 min. at 10,000 ft.
[67]
What is the highest flight level that operations may be conducted without the pilot at the controls wearing and
using an oxygen mask, while the other pilot is away from the duty station?
A. FL 240.
B. FL 250.
C. Above FL 250.
Answer (A) is incorrect because the highest flight level at which operations may be conducted without the
pilot at the controls wearing and using an oxygen mask while the other pilot is away from the duty station
is FL 250, not FL 240.
Answer (B) is correct. If for any reason at any time it is necessary for one pilot to leave his/her station at
the controls of the airplane when operating at flight altitudes above FL 250, the remaining pilot at the
controls shall put on and use his/her oxygen mask until the other pilot has returned to his/her duty station.
Thus, FL 250 is the highest flight level at which operations may be conducted without the pilot at the
controls wearing and using an oxygen mask while the other pilot is away from the duty station.
Answer (C) is incorrect because at flight levels above FL 250, the pilot at the controls must wear and use
an oxygen mask anytime the other pilot is away from the duty station.
[68]
If either pilot of an air carrier airplane leaves the duty station while flying at FL410, the other pilot
A. and the flight engineer shall put on their oxygen masks and breathe oxygen.
B. shall put on the oxygen mask and breathe oxygen.
C. must have a quick-donning type oxygen mask available.
Answer (A) is incorrect because only the remaining pilot at the flight controls, not the flight engineer, is
required to put on the oxygen mask and breathe oxygen.
Answer (B) is correct. If for any reason at any time it is necessary for one pilot to leave his/her duty station
at the controls of the airplane when operating at flight altitudes above FL250, the remaining pilot at the
controls must put on and use his/her oxygen mask until the other pilot has returned to his/her duty station.
Answer (C) is incorrect because, if one pilot leaves the duty station when above FL250, the remaining pilot
must put on the oxygen mask and breathe oxygen, not just have a quick-donning-type oxygen mask
available.
[69]
If a turbine-engine-powered, pressurized airplane is not equipped with quick-donning oxygen masks, what is the
maximum flight altitude authorized without one pilot wearing and using an oxygen mask?
A. FL 200.
B. FL 300.
C. FL 250.
Answer (A) is incorrect because the maximum altitude authorized without one pilot wearing and using an
oxygen mask in a turbine-engine-powered, pressurized airplane that is not equipped with quick-donning-
type oxygen masks is FL 250, not FL200.
Answer (B) is incorrect because the maximum altitude authorized without one pilot wearing and using an
oxygen mask in a turbine-engine-powered, pressurized airplane that is not equipped with quick-donning-
type oxygen masks is FL 250, not FL300.
Answer (C) is correct. If a turbine-engine-powered, pressurized airplane is not equipped with quick-
donning-type oxygen masks, the maximum altitude authorized without one pilot at the controls wearing
and using an oxygen mask is FL250.
[70]
Each air carrier flight deck crewmember on flight deck duty must be provided with an oxygen mask that can be
rapidly placed on his face when operating at flight altitudes
A. of FL 260.
B. of FL 250.
C. above FL 250.
Answer (A) is incorrect because each flight crewmember on flight deck duty must be provided with an
oxygen mask that can be rapidly placed on his/her face when operating at flight altitudes above FL 250,
not FL 260.
Answer (B) is incorrect because each flight crewmember on flight deck duty must be provided with an
oxygen mask that can be rapidly placed on his/her face when operating at flight altitudes above, not at, FL
250.
Answer (C) is correct. Each flight crewmember on flight deck duty must be provided with an oxygen mask
that can be rapidly placed on his/her face from its ready position, properly secured, sealed, and supplying
oxygen on demand, when operating at flight altitudes above FL 250.
[71]
A flight crewmember must be able to don and use a quick-donning oxygen mask within
A. 5 seconds.
B. 10 seconds.
C. 15 seconds.
Answer (A) is correct. It must be possible to place a quick-donning oxygen mask on the face from its ready
position, properly sealed, secured, and supplying oxygen on demand, with one hand within 5 seconds.
Answer (B) is incorrect because it must be possible to put on a quick-donning oxygen mask with one hand
and use it within 5 seconds, not 10 seconds.
Answer (C) is incorrect because it must be possible to put on a quick-donning oxygen mask with one hand
and use it within 5 seconds, not 15 seconds.
[72]
What is the minimum number of acceptable oxygen-dispensing units for first-aid treatment of occupants who might
require undiluted oxygen for physiological reasons?
A. Two.
B. Four.
C. Three.
Answer (A) is correct. There must be a minimum of two acceptable oxygen-dispensing units for first-aid
treatment of occupants who might require undiluted oxygen for physiological reasons.
Answer (B) is incorrect because the minimum number of oxygen-dispensing units for first-aid treatment is
two, not four.
Answer (C) is incorrect because the minimum number of oxygen-dispensing units for first-aid treatment is
two, not three.
[73]
A passenger briefing by a crewmember shall be given, instructing passengers on the necessity of using oxygen in
the event of cabin depressurization, prior to flights conducted above
A. FL 200.
B. FL 240.
C. FL 250.
Answer (A) is incorrect because only if the flight is conducted above FL 250, not FL 200, is an oxygen
briefing required.
Answer (B) is incorrect because only if the flight is conducted above FL 250, not FL 240, is an oxygen
briefing required.
Answer (C) is correct. Before flight is conducted above FL 250, a crewmember shall instruct the
passengers on the necessity of using oxygen in the event of cabin depressurization and shall point out to
them the location and demonstrate the use of the oxygen-dispensing equipment.
[74]
The supplemental oxygen requirements for passengers when a flight is operated at FL 250 is dependent upon the
airplane's ability to make an emergency descent to a flight altitude of
A. 10,000 feet within 4 minutes.
B. 14,000 feet within 4 minutes.
C. 12,000 feet within 4 minutes or at a minimum rate of 2,500 ft/min, whichever is quicker.
Answer (A) is incorrect because the airplane must be able to descend to 14,000 ft., not 10,000 ft., in 4 min.
Answer (B) is correct. The supplemental oxygen requirements for passengers when a flight is operated at
FL 250 is dependent upon the airplane's ability to make an emergency descent to a flight altitude of
14,000 ft. or less within 4 min.
Answer (C) is incorrect because the regulations do not specify a rate of descent in feet per minute. The
determining factor is the ability to descend to 14,000 ft., not 12,000 ft., in 4 min.
[75]
What emergency equipment is required for extended overwater operations?
A. A portable survival emergency locator transmitter for each life raft.
B. A pyrotechnic signaling device for each life preserver.
C. A life preserver equipped with a survivor locator light, for each person on the airplane.
Answer (A) is incorrect because only one portable survival type emergency locator transmitter is required
to be carried on the airplane, not one for each life raft.
Answer (B) is incorrect because there should be at least one pyrotechnic signaling device for each life raft,
not for each life preserver.
Answer (C) is correct. No person may operate an airplane in extended overwater operations unless there
is a life preserver, equipped with an approved survivor locator light, for each person on the airplane.
[76]
Each large aircraft operating over water must have a life preserver for each
A. aircraft occupant.
B. seat on the aircraft.
C. passenger seat, plus 10 percent.
Answer (A) is correct. No person may operate a large airplane in any overwater operation unless it is
equipped with life preservers or with an approved flotation means for each airplane occupant.
Answer (B) is incorrect because each large airplane operating over water must have a life preserver for
each occupant, not each seat, on the airplane.
Answer (C) is incorrect because each large airplane operating over water must have a life preserver for
each occupant, not each passenger seat plus 10%.
[77]
Life preservers required for overwater operations are stored
A. within easy reach of each passenger.
B. under each occupant seat.
C. within easy reach of each seated occupant.
Answer (A) is incorrect because life preservers must be stored within easy reach of each seated occupant
(flight crewmembers and passengers), not within easy reach of each passenger.
Answer (B) is incorrect because life preservers must be stored within easy reach of each seated occupant,
not under each occupant seat.
Answer (C) is correct. Life preservers required for overwater operations must be stored within easy reach
of each seated occupant.
[78]
For the purpose of testing the flight recorder system,
A. a minimum of 1 hour of the oldest recorded data must be erased to get a valid test.
B. a total of 1 hour of the oldest recorded data accumulated at the time of testing may be erased.
C. a total of no more than 1 hour of recorded data may be erased.
Answer (A) is incorrect because, for the purpose of testing the flight recorder system, a maximum, not
minimum, of 1 hr. of the oldest recorded data may be erased.
Answer (B) is correct. For the purpose of testing the flight recorder or flight recorder system, a total of 1 hr.
of the oldest recorded data accumulated at the time of testing may be erased.
Answer (C) is incorrect because the recorded data that is erased must be the oldest at the time of testing,
not just any data.
[79]
When an air carrier flight is operated under IFR, which navigation equipment is required to be installed in
duplicate?
A. VOR.
B. ADF.
C. VOR and DM E.
Answer (A) is correct. No person may operate an airplane under IFR or over-the-top on VHF airways
unless the airplane is equipped with that radio equipment necessary to receive satisfactorily, by either of
two independent systems, radio navigational signals from all primary enroute and approach navigational
facilities intended to be used, which are VOR/VORTAC stations. Equipment provided to receive signals
enroute may be used to receive signals on approach, if it is capable of receiving both signals.
Answer (B) is incorrect because ADF equipment is not required for a flight operated IFR or over-the-top on
VHF airways.
Answer (C) is incorrect because only one, not two, DME is required to be installed whenever VOR
navigation equipment is required.
[80]
When a pilot plans a flight using NDB NAVAIDS, which rule applies?
A. The airplane must have sufficient fuel to proceed, by means of VOR NAVAIDS, to a suitable airport and land.
B. The pilot must be able to return to the departure airport using other navigation radios.
C. The airplane must have sufficient fuel to proceed, by means of VOR NAVAIDS, to a suitable airport and
complete an instrument approach by use of the remaining airplane radio system.
Answer (A) is incorrect because the airplane must have sufficient fuel to proceed, by means of VOR
NAVAIDs, to a suitable airport and complete an instrument approach, not only land.
Answer (B) is incorrect because the airplane must be able to fly to a suitable airport, not necessarily the
departure airport, and complete an instrument approach.
Answer (C) is correct. When operating over routes on which navigation is based on NDB NAVAIDs, only
one ADF receiver need be installed if the airplane is equipped with two VOR receivers and if VOR
navigational aids are so located and the airplane is so fueled that, in the case of failure of the ADF
receiver, the flight may proceed safely to a suitable airport, by means of VOR aids, and complete an
instrument approach by use of the remaining airplane radio system.
[81]
If an air carrier airplane is flying IFR using a single ADF navigation receiver and the ADF equipment fails, the flight
must be able to
A. proceed safely to a suitable airport using VOR aids and complete an instrument approach by use of the
remaining airplane radio system.
B. continue to the destination airport by means of dead reckoning navigation.
C. proceed to a suitable airport using VOR aids, complete an instrument approach, and land.
Answer (A) is correct. In the case of operation over routes on which navigation is based on ADF, only one
ADF receiver need be installed if the airplane is equipped with two VOR receivers and if VOR navigational
aids are so located and the airplane is so fueled that, in the case of failure of the ADF receiver, the flight
may proceed safely to a suitable airport, by means of VOR aids, and complete an instrument approach by
use of the remaining airplane radio system.
Answer (B) is incorrect because the flight must be able to proceed to a suitable airport, not necessarily the
destination airport, by means of VOR equipment, not dead reckoning navigation, and complete an
instrument approach utilizing the remaining airplane radio system.
Answer (C) is incorrect because the flight to the sui table airport must be done safely, and the instrument
approach at the suitable airport may be completed by the use of the remaining airplane radio systems.
[82]
For a flight over uninhabited terrain, an airplane operated by a flag or supplemental air carrier must carry enough
appropriately equipped survival kits for
A. all of the passengers, plus 10 percent.
B. all aircraft occupants.
C. all passenger seats.
Answer (A) is incorrect because there must be enough appropriately equipped survival kit s for all
occupants on the airplane (including the crew), not enough for all the passengers plus 10%.
Answer (B) is correct. No air carrier may conduct an operation over an uninhabited area or any other area
that requires equipment for search and rescue in case of an emergency unless it has enough survival kits,
appropriately equipped for the route to be flown, for the number of occupants of the airplane.
Answer (C) is incorrect because there must be enough appropriately equipped survival kits for all aircraft
occupants, not all passenger seats.
[83]
An airplane operated by a commercial operator flying over uninhabited terrain must carry which emergency
equipment?
A. A signal mirror and colored smoke flares.
B. Survival kit for each passenger.
C. An approved survival-type emergency locator transmitter.
Answer (A) is incorrect because a signal mirror is not required for a flight over uninhabited terrain, but a
suitable pyrotechnic signaling device, not colored smoke flares, is required.
Answer (B) is incorrect because the airplane must carry a survival kit for each occupant (including the
crew), not each passenger.
Answer (C) is correct. No airplane operated by a commercial operator may conduct operations over an
uninhabited area or any other area that requires equipment for search and rescue in case of an
emergency unless the airplane has an approved survival-type emergency locator transmitter.
[84]
If an air carrier airplane's airborne radar is inoperative and thunderstorms are forecast along the proposed route of
flight, an airplane may be dispatched only
A. when able to climb and descend VFR and maintain VFR enroute.
B. in VFR conditions.
C. in day VFR conditions.
Answer (A) is incorrect because, with an inoperative airborne weather radar, the airplane may be
dispatched to fly only during the day, not night, in VFR weather conditions.
Answer (B) is incorrect because, with an inoperative airborne weather radar, the airplane may be
dispatched to fly only during day, not night, VFR weather conditions.
Answer (C) is correct. If an air carrier airplane's airborne weather radar is inoperative and thunderstorms
are forecast along the proposed route of flight, that airplane may be dispatched only in day VFR
conditions.
[85]
An air carrier airplane's airborne radar must be in satisfactory operating condition prior to dispatch, if the flight will
be
A. conducted under VFR conditions at night with scattered thunderstorms reported en route.
B. carrying passengers, but not if it is "all cargo."
C. conducted IFR, and ATC is able to radar vector the flight around areas of weather.
Answer (A) is correct. No person may dispatch an airplane (or begin the flight of an airplane in the case of
an air carrier or commercial operator that does not use a dispatch system) under IFR or night VFR
conditions when current weather reports indicate that thunderstorms, or other potentially hazardous
weather conditions that can be detected with airborne weather radar, may reasonably be expected along
the route to be flown, unless the airborne weather radar equipment is in satisfactory operating condition.
Answer (B) is incorrect because the requirement that the airplane be equipped with an operating airborne
weather radar applies t o a transport-category aircraft, regardless of whether the airplane is used to carry
passengers or cargo.
Answer (C) is incorrect because an airplane's airborne weather radar must be satisfactorily operating
before conducting an IFR flight if thunderstorms can be expected along the route to be flown, not if ATC is
able to provide vectors for weather.
[86]
What action should be taken by the pilot in command of a transport category airplane if the airborne weather radar
becomes inoperative en route on an IFR flight for which weather reports indicate possible thunderstorms?
A. Request radar vectors from ATC to the nearest suitable airport and land.
B. Proceed in accordance with the approved instructions and procedures specified in the operations manual for
such an event.
C. Return to the departure airport if the thunderstorms have not been encountered, and there is enough fuel
remaining.
Answer (A) is incorrect because, if the airborne weather radar becomes inoperative en route, the pilot in
command must follow the instructions and procedures specified in the operations manual, not request
radar vectors from ATC to the nearest suitable airport and land.
Answer (B) is correct. If the airborne weather radar becomes inoperative en route, the pilot in command
must proceed in accordance with the approved instructions and procedures specified in the operations
manual for such an event.
Answer (C) is incorrect because, if the airborne weather radar becomes inoperative en route, the pilot in
command must follow the instructions and procedures specified in the operations manual, not return to the
departure airport if the thunderstorms have not been encountered and if there is enough fuel remaining.
[87]
Information recorded during normal operation of a cockpit voice recorder in a large pressurized airplane with four
reciprocating engines
A. may all be erased or otherwise obliterated except for the last 30 minutes.
B. may be erased or otherwise obliterated except for the last 30 minutes prior to landing.
C. may all be erased, as the voice recorder is not required on an aircraft with reciprocating engines.
Answer (A) is correct. All of the information recorded during normal operation of a cockpit voice recorder,
in a large pressurized airplane with four reciprocating engines, may be erased or otherwise obliterated
except for the last 30 min.
Answer (B) is incorrect because the information recorded on a cockpit voice recorder may be erased or
otherwise obliterated except for the last 30 min., not the last 30 min. prior to landing.
Answer (C) is incorrect because an approved cockpit voice recorder is required on large turbine-powered
airplanes and on large pressurized airplanes with four reciprocating engines.
[88]
Which rule applies to the use of the cockpit voice recorder erasure feature?
A. All recorded information may be erased, except for the last 30 minutes prior to landing.
B. Any information more than 30 minutes old may be erased.
C. All recorded information may be erased, unless the Authority needs to be notified of an occurrence.
Answer (A) is incorrect because only information more than 30 min. old may be erased, not all information
except the last 30 mi n. prior to landing.
Answer (B) is correct. An approved cockpit voice recorder having an erasure feature may be used so that,
at any time during the operation of the recorder, information recorded more than 30 min. earlier may be
erased or otherwise obliterated.
Answer (C) is incorrect because only information older than 30 min., not all information, may be erased. If
an occurrence requiring immediate notification of the Authority results in the termination of a flight, the
certificate holder must keep the recorded information for at least 60 days.
[89]
A cockpit voice recorder must be operated
A. from the start of the before starting engine checklist to completion of final checklist upon termination of flight.
B. from the start of the before starting engine checklist to completion of checklist prior to engine shut down.
C. when starting to taxi for takeoff to the engine shut down checklist after termination of the flight.
Answer (A) is correct. No certificate holder may operate a large turbine-engine-powered airplane or a large
pressurized airplane with four reciprocating engines unless an approved cockpit voice recorder is installed
in that airplane and is operated continuously from the start of the use of the checklist (before starting
engines for the purpose of flight) to completion of the final checklist at the termination of the flight.
Answer (B) is incorrect because the cockpit voice recorder must be operated to completion of the final
checklist upon termination of the flight, not the checklist prior to engine shut down.
Answer (C) is incorrect because the cockpit voice recorder must be operated from the beginning of the
start-engine checklist, not from the start of the taxi for takeoff, to the final checklist after engine shutdown,
not just engine shut down.
[90]
Which airplanes are required to be equipped with a ground proximity warning glide slope deviation alerting
system? A. All turbine-powered airplanes.
B. Passenger-carrying turbine-powered airplanes only.
C. Large turbine-powered airplanes only.
Answer (A) is correct. No person may operate a turbine-powered airplane unless it is equipped with a
ground proximity warning-glide slope deviation alerting system.
Answer (B) is incorrect because all turbine-powered airplanes are required to be equipped with a ground
proximity warning-glide slope deviation alerting system, regardless of whether that airplane is used for
passenger or cargo operations.
Answer (C) is incorrect because all, not only large, turbine-powered airplanes are required to be equipped
with a ground proximity warning-glide slope deviation alerting system.
[91]
Which equipment requirement must be met by an air carrier that elects to use a dual Inertial Navigation System
(INS) on a proposed flight?
A. The dual system must consist of two operative INS units.
B. A dual VORTAC/ ILS system may be substituted for an inoperative INS.
C. Only one INS is required to be operative, if a Doppler Radar is substituted for the other INS.
Answer (A) is incorrect because the dual INS system may consist of two INS units or one INS unit and one
Doppler radar unit. It does not have to be two INS units.
Answer (B) is incorrect because VOR TAC/ ILS systems are not authorized as substitutes for an
inoperative INS.
Answer (C) is correct. If an applicant elects to use an Inertial Navigation System, it must be at least a dual
system (including navigational computers and reference units). At least two systems must be operational
at takeoff. The dual system may consist of either two INS units or one INS unit and one Doppler radar unit.
[92]
An air carrier that elects to use an Inertial Navigational System (INS) must meet which equipment requirement
prior to takeoff on a proposed flight?
A. The INS system must consist of two operative INS units.
B. Only one INS is required to be operative, if a Doppler Radar is substituted for the other INS.
C. A dual VORTAC/ ILS system may be substituted for an inoperative INS.
Answer (A) is incorrect because the dual INS system may consist of two INS units or one INS unit and one
Doppler radar unit.
Answer (B) is correct. If an applicant elects to use an Inertial Navigation System, it must be at least a dual
system (including navigational computers and reference units). At least two systems must be operational
at takeoff. The dual system may consist of either two INS units or one INS unit and one Doppler radar unit.
Answer (C) is incorrect because VORTAC/ ILS systems are not authorized as substitutes for an
inoperative INS
[93]
An air carrier uses an airplane that is certified for operation with a flight crew of two pilots and one flight engineer.
In case the flight engineer becomes incapacitated,
A. at least one other flight crewmember must be qualified to perform flight engineer duties.
B. one crewmember must be qualified to perform the duties of the flight engineer.
C. one pilot must be qualified and have a flight engineer certificate to perform the flight engineer duties.
Answer (A) is correct. On each flight requiring a flight engineer, at least one flight crewmember, other than
the flight engineer, must be qualified to provide emergency performance of the flight engineer's functions
for the safe completion of the flight if the flight engineer becomes ill or is otherwise incapacitated. A pilot
need not hold a flight engineer's certificate to perform the flight engineer's functions in such a situation.
Answer (B) is incorrect because the requirement is for at least one flight crewmember, not just one
crewmember, to be qualified to perform the flight engineer's duties.
Answer (C) is incorrect because, if the flight engineer becomes incapacitated, a pilot may perform the
duties of the flight engineer if that pilot is qualified to perform the duties, but (s)he is not required to have a
flight engineer certificate.
[94]
Where are the routes listed that require special navigation equipment?
A. Certificate holder's Operations Specifications.
B. International Aeronautical Information Manual.
C. International Notices to Airmen.
Answer (A) is correct. Routes that require special navigation equipment are listed in the certificate holder's
operations specifications.
Answer (B) is incorrect because the International Aeronautical Information Manual is a publication
designed primarily as a preflight planning guide for international flights. It does not list routes which require
special navigation equipment.
Answer (C) is incorrect because international NOTAMs contain information concerning the establishment
or condition of, or change in, any facility, service, or procedure or hazards for flights. They do not list
routes which require special navigation equipment.
[95]
An airplane has seats for 149 passengers and eight crewmembers. What is the minimum number of flight
attendants required with 97 passengers aboard?
A. Four.
B. Three.
C. Two.
Answer (A) is incorrect because four flight attendants are required on an airplane with a seating capacity
between 151 and 200, not 149.
Answer (B) is correct. The number of required flight attendants is based up on passenger seating capacity,
not the number of passengers on a given flight. A seating capacity of 51 to 100 requires two flight
attendants, after which one flight attendant is required for each unit of 50 seats. A seating capacity of 149
requires three flight attendants.
Answer (C) is incorrect because two flight attendants are required on an airplane with a seating capacity of
97, not 149. The number of required flight attendants is based on passenger seating capacity, not the
number of passengers on a given flight.
[96]
Which document includes descriptions of the required crewmember functions to be performed in the event of an
emergency?
A. Airplane Flight Manual.
B. Certificate holder's manual.
C. Pilot's Emergency Procedures Handbook.
Answer (A) is incorrect because the airplane flight manual contains emergency procedures appropriate to
the airplane, not the required crewmember functions during an emergency.
Answer (B) is correct. The certificate holder's manual must include descriptions of the required
crewmember functions to be performed in the event of an emergency.
Answer (C) is incorrect because the description of the required crewmember functions in an emergency is
found in the certificate holder's manual, not a pi lot 's emergency procedures handbook.
[97]
The required crewmember functions that are to be performed in the event of an emergency shall be assigned by
the
A. pilot in comm and.
B. air carrier's chief pilot.
C. certificate holder.
Answer (A) is incorrect because the required crewmember functions that are to be performed in the event
of an emergency must be assigned by the certificate holder, not the pilot in comm and.
Answer (B) is incorrect because certificate holder, not the chief pilot, must assign the required
crewmember functions that are to be performed in the event of an emergency.
Answer (C) is correct. The certificate holder must assign the required crewmember functions that are to be
performed in the event of an emergency.
[98]
The training required by flight crewmembers who have not qualified and served in the same capacity on another
airplane of the same group (e.g., turbojet powered) is
A. upgrade training.
B. transition training.
C. initial training.
Answer (A) is incorrect because upgrade training is required for a flight crewmember who has qualified
and served as second in command or flight engineer on a particular airplane type, before (s)he serves as
pilot in command or second in command, respectively, on that airplane.
Answer (B) is incorrect because transition training is required for flight crewmembers who have qualified
and served in the same capacity on another airplane in the same group.
Answer (C) is correct. Initial training is required for flight crewmembers who have not qualified and served
in the same capacity on another airplane of the same group (e.g., turbojet powered).
[99]
A crewmember who has served as second in command on a particular type airplane (e.g., B-727-100), may serve
as pilot in command upon completing which training program?
A. Upgrade training.
B. Recurrent training.
C. Initial training.
Answer (A) is correct. Upgrade training is required for a crewmember who has served as second in
command on a particular type airplane (e.g., B-727-100) before (s)he may serve as pilot in command on
that airplane.
Answer (B) is incorrect because recurrent training is a scheduled periodic training requirement to ensure
that a crewmember is adequately trained and currently proficient with respect to the type of airplane and
crewmember position involved.
Answer (C) is incorrect because initial training is required for a crewmember who has not qualified and
served in the same capacity on another airplane of the same group.
[100]
The training required for crewmembers or dispatchers who have been qualified and served in the same capacity
on other airplanes of the same group is
A. difference training.
B. transition training.
C. upgrade training.
Answer (A) is incorrect because difference training is required for crewmembers and dispatchers who
have qualified and served on a particular type airplane.
Answer (B) is correct. Transition training is required for crewmembers and dispatchers who have qualified
and served in the same capacity on another airplane of the same group.
Answer (C) is incorrect because upgrade training is required for a crewmember who has qualified and
served as second in command or flight engineer on a particular airplane type, before (s)he serves as pilot
in command or second in command, respectively, on that airplane.
[101]
The air carrier must give instruction on such subjects as respiration, hypoxia, and decompression to crewmembers
serving on pressurized airplanes operated above
A. FL 180.
B. FL 200.
C. FL 250.
Answer (A) is incorrect because crewmembers who serve in operations above FL 250, not FL 180, must
receive instruction on such subjects as respiration, hypoxia, and decompression.
Answer (B) is incorrect because crewmembers who serve in operations above FL 250, not FL 200, must
receive instruction on such subjects as respiration, hypoxia, and decompression.
Answer (C) is correct. Crewmembers who serve in operations above FL 250 must receive instruction on
such subjects as respiration, hypoxia, and decompression.
[102]
How often must a crewmember actually operate the airplane emergency equipment, after initial training? Once
every
A. 6 calendar months.
B. 12 calendar months.
C. 24 calendar months.
Answer (A) is incorrect because a crewmember must actually operate emergency equipment once every
24 calendar months, not 6 calendar months.
Answer (B) is incorrect because a crewmember must actually operate emergency equipment once every
24 calendar months, not 12 calendar months.
Answer (C) is correct. During initial training and once each 24 calendar months during recurrent training,
each crewmember must perform certain emergency drill s and operate specified emergency equipment.
[103]
In a cargo-only operation, cargo must be loaded
A. so that it does not obstruct the aisle between the crew and cargo compartments.
B. in such a manner that at least one emergency or regular exit is available to all occupants.
C. in such a manner that at least one emergency or regular exit is available to all crewmembers, if an emergency
occurs.
Answer (A) is incorrect because cargo must be loaded so that it does not obstruct the aisle between the
crew and passenger, not cargo, compartments in a passenger-carrying flight, not in a cargo-only
operation.
Answer (B) is correct. In a cargo-only operation, cargo must be loaded so that at least one emergency or
regular exit is available to provide all occupants of the aircraft a means of unobstructed exit from the
aircraft if an emergency occurs.
Answer (C) is incorrect because access to at least one emergency or regular exit must be available to all
occupants, not just crewmembers, if an emergency occurs.
[104]
What is one of the requirements that must be met by an airline pilot to re-establish recency of experience?
A. At least one landing must be made from a circling approach.
B. At least one full stop landing must be made.
C. At least one precision approach must be made to the lowest minimums authorized for the certificate holder.
Answer (A) is incorrect because one landing must be made from an ILS approach to the lowest ILS
minimums, which implies a straight-in approach, not a circling approach.
Answer (B) is correct. A required pilot flight crewmember who has not made three takeoffs and landings
within the preceding 90 days must reestablish recency of experience as follows: Under the supervision of
a check airman, make at least three takeoffs and landings in the type airplane in which that person is to
serve or in a simulator; 1. At least one takeoff with a simulated failure of the most critical powerplant; 2. At
least one landing from an ILS approach to the lowest ILS minimum authorized for the certificate holder; 3.
At least one landing to a full stop
Answer (C) is incorrect because a landing must be made, not just an approach, from an ILS approach, not
any precision approach, to the lowest ILS minimum authorized for the certificate holder.
[105]
Which is one of the requirements that must be met by a required pilot flight crewmember in re-establishing recency
of experience?
A. At least one landing must be made with a simulated failure of the most critical engine.
B. At least one ILS approach to the lowest ILS minimums authorized for the certificate holder and a landing from
that approach.
C. At least three landings must be made to a complete stop.
Answer (A) is incorrect because at least one takeoff, not landing, is required with a simulated failure of the
most critical engine.
Answer (B) is correct. A required pilot flight crewmember who has not made three takeoffs and landings
within the preceding 90 days must reestablish recency of experience as follows: Under the supervision of
a check airman, make at least three takeoffs and landings in the type airplane in which that person is to
serve or in a simulator; 1. At least one takeoff with a simulated failure of the most critical powerplant; 2. At
least one landing from an ILS approach to the lowest ILS minimum authorized for the certificate holder; 3.
At least one landing to a full stop
Answer (C) is incorrect because only one landing, not three landings, to a complete stop is required.
[106]
A pilot in command must complete a proficiency check or simulator training within the preceding
A. 6 calendar months.
B. 12 calendar months.
C. 24 calendar months.
Answer (A) is correct. A pilot in comm and must have completed a proficiency check within the preceding
12 calendar months and, in addition, within the preceding 6 calendar months, either a proficiency check or
simulator training.
Answer (B) is incorrect because, within the preceding 12 calendar months, a pilot in comm and must have
completed a proficiency check, not a proficiency check or simulator training.
Answer (C) is incorrect because, within the preceding 24 calendar months, all pilots other than the pilot in
command must have completed a proficiency check or line-oriented simulator training.
[107]
A pilot flight crewmember, other than pilot in command, must have received a proficiency check or line-oriented
simulator training within the preceding
A. 6 calendar months.
B. 12 calendar months.
C. 24 calendar months.
Answer (A) is incorrect because, within the preceding 6 calendar months, a pilot in command, not any
other pilot, must have completed a proficiency check or simulator training.
Answer (B) is incorrect because, within the preceding 12 calendar months, all pilots other than the pilot in
command must have completed a proficiency check or any approved simulator training course, not
necessarily line-oriented simulator training.
Answer (C) is correct. No certificate holder may use any person, nor may any person serve, as a required
pilot flight crewmember other than a pilot in command unless that person has satisfactorily completed
within the preceding 24 calendar months either a proficiency check or the line-oriented simulator training
course.
[108]
To remain current as an aircraft dispatcher, a person must, in addition to other requirements,
A. within the preceding 12 calendar months, spend 2.5 hours observing flight deck operations, plus two additional
takeoffs and landings, in one of the types of airplanes in each group he/she is to dispatch.
B. within the preceding 12 calendar months, spend at least 5 hours observing flight deck operations in one of the
types of airplanes in each group he/she is to dispatch.
C. within the preceding 12 calendar months, spend at least 5 hours observing flight deck operations in each type
of airplane, in each group that he/she is to dispatch.
Answer (A) is incorrect because a dispatcher may reduce the 5 hr. of observing flight deck operations
requirement to a minimum of 2.5 hr. by the substitution of one additional takeoff and landing for an hour of
flight. In this case, the aircraft dispatcher must spend 2.5 hr. observing flight deck operations plus three,
not two, additional takeoffs and landings in one of the types of airplanes in each airplane group (s)he is to
dispatch.
Answer (B) is correct. No air carrier may use any person, nor may any person serve, as an aircraft
dispatcher unless within the preceding 12 calendar months (s)he has satisfactorily completed operating
familiarization consisting of at least 5 hr. observing flight deck operations in one of the types of airplanes in
each airplane group (s)he is to dis patch.
Answer (C) is incorrect because the aircraft dispatcher is required to spend at least 5 hr. observing flight
deck operations in one of the types of airplanes, not each type of airplane, in each airplane group (s)he is
to dispatch.
[109]
Normally, a dispatcher should be scheduled for no more than
A. 8 hours of service in any 24 consecutive hours.
B. 10 hours of duty in any 24 consecutive hours.
C. 10 consecutive hours of duty.
Answer (A) is incorrect because a dispatcher should normally be scheduled for no more than 10
consecutive hr. of duty, not 8 hr. of service in any 24-consecutive hr.
Answer (B) is incorrect because a dispatcher may be scheduled for more than 10 hr. of duty in any 24
consecutive hr. if the air carrier provides the dispatcher a rest period of at least 8 hr. at or before the end of
10 hr. of duty.
Answer (C) is correct. Except in cases in which circumstances or emergency conditions beyond the control
of the air carrier require otherwise, no air carrier may schedule a dispatcher for more than 10 consecutive
hr. of duty.
[110]
If an air carrier schedules a dispatcher for 13 hours of duty in a 24-consecutive-hour period, what action is
required?
A. The dispatcher should be given a rest period of 24 hours at the end of the 13 hours.
B. The dispatcher should refuse to be on duty 13 hours
C. The dispatcher should be given a rest period of at least 8 hours at or before the completion of 10 hours of duty.
Answer (A) is incorrect because the air carrier must provide the dispatcher a rest period of at least 8 hr.,
not 24 hr., at or before the end of 10 hr. of duty, not at the end of 13 hr. of duty.
Answer (B) is incorrect because there will be no violation if the air carrier provides the dispatcher a rest
period of at least 8 hr. at or before the end of 10 hr. of duty.
Answer (C) is correct. If an air carrier schedules a dispatcher for more than 10 hr. of duty in 24
consecutive hr., the carrier must provide the dispatcher a rest period of at least 8 hr. at or before the end
of 10 hr. of duty.
[111]
An aircraft dispatcher shall receive at least 24 consecutive hours of rest during
A. every 7 consecutive days.
B. any 7 consecutive days or the equivalent thereof within any calendar month.
C. each calendar week.
Answer (A) is incorrect because an aircraft dispatcher must receive at least 24 consecutive hr. of rest
during any 7 consecutive days within any calendar month, not every 7 consecutive days.
Answer (B) is correct. An aircraft dispatcher must be relieved of all duty with the air carrier for at least 24
consecutive hr. during any 7 consecutive days or the equivalent thereof within any calendar month.
Answer (C) is incorrect because an aircraft dispatcher must receive at least 24 consecutive hr. of rest
during any 7 consecutive days within any calendar month, not during each calendar week.
[112]
Duty and rest period rules for air carrier operations require that a flight crewmember
A. not be assigned to any duty with the air carrier during any required rest period.
B. not be on duty aloft for more than 100 hours in any 30-day period.
C. be relieved of all duty for at least 24 hours during any 7 consecutive days.
Answer (A) is correct. No air carrier may assign any flight crewmember and no flight crewmember may
accept assignment to any duty with the air carrier during any required rest period.
Answer (B) is incorrect because the total flight time limitation is 100 hr. in any calendar month, not in any
30-day period.
Answer (C) is incorrect because an air carrier must relieve a flight crewmember from all duty for at least 24
consecutive hr., not just 24 hr., during any 7 consecutive days.
[113]
How does deadhead transportation, going to or from a duty assignment, affect the computation of flight time limits
for air carrier flight crewmembers? It is
A. considered part of the rest period if the flight crew includes more than two pilots.
B. considered part of the rest period for flight engineers and navigators.
C. not considered part of a rest period.
Answer (A) is incorrect because time spent in deadhead transportation to or from a duty assignment is not
considered to be a part of a rest period, regardless of the number of pilots in the flight crew.
Answer (B) is incorrect because time spent in deadhead transportation to or from a duty assignment is not
considered to be a part of a rest period. This requirement includes pilots, flight engineers, and flight
navigators.
Answer (C) is correct. For the computation of flight time limits for an air carrier flight crewmember, time
spent in deadhead transportation to or from a duty assignment is not considered to be a part of a rest
period.
[114]
The maximum number of consecutive hours of duty that an aircraft dispatcher may be scheduled is
A. 12 hours.
B. 10 hours.
C. 8 hours.
Answer (A) is incorrect because an aircraft dispatcher may be scheduled for a maximum of 10 consecutive
hr. of duty, not 12 consecutive hr.
Answer (B) is correct. No air carrier may schedule a dispatcher for more than 10 consecutive hr. of duty.
Answer (C) is incorrect because an aircraft dispatcher may be scheduled for a maximum of 10 consecutive
hr. of duty, not 8 consecutive hr.
[115]
The persons jointly responsible for the initiation, continuation, diversion, and termination of a commercial operator
flight are the
A. pilot in command and chief pilot.
B. pilot in command and director of operations.
C. pilot in command and the flight follower.
Answer (A) is incorrect because the pilot in command and the director of operations, not the chief pilot, of
are jointly responsible for the initiation, continuation, diversion, and termination of a flight.
Answer (B) is correct. The pilot in command and the director of operations are jointly responsible for the
initiation, continuation, diversion, and termination of an air carrier flight. The director of operations may
delegate the functions for the initiation, continuation, diversion, and termination of a flight, but (s)he may not
delegate the responsibility for those functions.
Answer (C) is incorrect because the pi lot in command and the director of operations, not a flight follower,
are jointly responsible for the initiation, continuation, diversion, and termination of a flight.
[116]
With regard to flight crewmember duties, which of the following operations are considered to be in the "critical phase
of flight"?
A. Taxi, takeoff, landing, and all other operations conducted below 10,000 feet MSL, including cruise flight.
B. Descent, approach, landing, and taxi operations, irrespective of altitudes MSL.
C. Taxi, takeoff, landing, and all other operations conducted below 10,000 feet, excluding cruise flight.
Answer (A) is incorrect because the critical phases of flight include all ground operations involving taxi,
takeoff, and landing, and all other operations conducted below 10,000 ft., except cruise flight. The critical
phase of flight does not include cruise flight when it is conducted below 10,000 ft.
Answer (B) is incorrect because the critical phases of flight include all ground operations involving taxi,
takeoff, and landing, and all other operations conducted below 10,000 ft., except cruise flight.
Answer (C) is correct. No flight crewmember may perform any duties during a critical phase of flight except
those duties required for the safe operation of the aircraft. The critical phases of flight include all ground
operations involving taxi, takeoff, and landing, and all other flight operations conducted below 10,000 ft.,
except cruise flight.
[117]
Below which altitude, except when in cruise flight, are non-safety-related cockpit activities by flight crewmembers
prohibited?
A. 10,000 feet.
B. 14,500 feet.
C. FL 180.
Answer (A) is correct. No flight crewmember may perform any non-safety-related cockpit duties during the
critical phase of flight. The critical phases of flight include all ground operations involving taxi, takeoff and
landing, and all other flight operations conducted below 10,000 ft., except in cruise flight.
Answer (B) is incorrect because flight crewmembers are prohibited from any non-safety-related cockpit
duties below 10,000 ft., not 14,500 ft., except in cruise flight.
Answer (C) is incorrect because flight crewmembers are prohibited from any non-safety-related cockpit
duties below 10,000 ft., not FL 180, except in cruise flight.
[118]
The pilot in command has emergency authority to exclude any and all persons from admittance to the flight deck
A. except an NCAA inspector doing enroute checks.
B. in the interest of safety.
C. except persons who have authorization from the certificate holder and the Authority.
Answer (A) is incorrect because, while an NCAA inspector may be admitted to the flight deck during normal
operations, the pilot in command has emergency authority to exclude any person from the flight deck in the
interest of safety.
Answer (B) is correct. In the interest of safety, the pilot in command has emergency authority to exclude any
and all persons, without exception, from admittance to the flight deck.
Answer (C) is incorrect because, while persons who have authorization from the certificate holder and the
Authority may be admitted to the flight deck during normal operations, the pilot in command has emergency
authority to exclude any and all persons from admittance to the flight deck in the interest of safety.
[119]
Each crewmember shall have readily available for individual use on each flight a
A. key to the flight deck door.
B. certificate holder's manual.
C. flashlight in good working order.
Answer (A) is incorrect because a key to the flight deck door is not required equipment for each
crewmember.
Answer (B) is incorrect because only appropriate parts of, not the entire, certificate holder's manual must be
accessible to each crewmember.
Answer (C) is correct. Each crewmember shall, on each flight, have readily available for his/ her use a
flashlight in good working order.
[120]
Assuring that appropriate aeronautical charts are aboard an aircraft is the responsibility of the
A. aircraft dispatcher.
B. flight navigator.
C. pilot in command.
Answer (A) is incorrect because the dispatcher may be hundreds of miles from the origination of the flight,
e.g., in a central dispatch office.
Answer (B) is incorrect because, although a flight navigator may be assigned the task of carrying
aeronautical charts, the pilot in command is responsible for ensuring that adequate charts are aboard the
aircraft.
Answer (C) is correct. The pilot in command shall ensure that appropriate aeronautical charts containing
adequate information concerning navigation aids and instrument approach procedures are aboard the
aircraft for each flight.
[121]
Who is required to submit a written report on a deviation that occurs during an emergency?
A. Pilot in command.
B. Dispatcher.
C. Person who declares the emergency.
Answer (A) is incorrect because either the pilot in comm and or the dispatcher must submit a written report,
depending upon who declared an emergency.
Answer (B) is incorrect because either the pilot in command or the dispatcher must submit a written report,
depending upon who declared an emergency.
Answer (C) is correct. Whenever a pilot in command or a dispatcher exercises emergency authority, (s)he
shall keep the appropriate ATC facility and dispatch centers fully informed of the progress of the flight. The
person declaring the emergency shall send a written report of any deviation through the air carrier's
operations manager to the administrator. A dispatcher shall send his /her report within 10 days after the
date of the emergency, and a pilot in command shall send his /her report within 10 days after returning to
his /her home base.
[122]
When the pilot in command is responsible for a deviation during an emergency, the pilot should submit a written
report within
A. 10 days after the deviation.
B. 10 days after returning home.
C. 10 days after returning to home base.
Answer (A) is incorrect because, when the dispatcher, not the pilot in command, declares an emergency
and is responsible for a deviation, (s)he must submit a report, through the air carrier's operations manager,
to the Authority within 10 days after the deviation.
Answer (B) is incorrect because, when the pilot in command declares an emergency and is responsible for
a deviation during that emergency, (s)he must submit a report, through the air carrier's operations manager,
to the Authority within 10 days after returning to his /her home base, not his /her home.
Answer (C) is correct. When the pilot in command declares an emergency and is responsible for a deviation
during that emergency, the pilot should submit a report, through the air carrier's operations manager, to the
Authority within 10 days after returning to his/her home base.
[123]
An aircraft dispatcher declares an emergency for a flight and a deviation results. A written report shall be sent by the
A. dispatcher to the Authority within 10 days.
B. air carrier's operations manager to the Authority within 10 days.
C. pilot in command to the Authority within 10 days.
Answer (A) is incorrect because an aircraft dispatcher must submit a written report through the air carrier's
operations manager before it is routed to the Authority.
Answer (B) is correct. When an aircraft dispatcher declares an emergency for a flight and a deviation
results, the dispatcher must submit a written report, through the air carrier's operations manager to the
Authority within 10 days.
Answer (C) is incorrect because an aircraft dispatcher must submit a written report through the air carrier's
operations manager, not the pilot in command, to the Authority within 10 days.
[124]
If an aircraft dispatcher cannot communicate with the pilot of an air carrier flight during an emergency, the aircraft
dispatcher should
A. take any action considered necessary under the circumstances.
B. comply with the company's lost aircraft plan.
C. phone the ARTCC where the flight is located and ask for a phone patch with the flight.
Answer (A) is correct. If the aircraft dispatcher cannot communicate with the pilot in command of a flight in
an emergency situation that requires immediate decision and action by the aircraft dispatcher, (s)he shall
declare an emergency and take any action that (s)he considers necessary under the circumstances
Answer (B) is incorrect because, if an aircraft dispatcher cannot communicate with the pilot of an air carrier
flight during an emergency, the aircraft dispatcher should take any action considered necessary under the
circumstances, which may or may not include following the company's lost aircraft plan.
Answer (C) is incorrect because, if an aircraft dispatcher cannot communicate with the pilot of an air carrier
flight during an emergency, (s)he should take any action that (s)he considers necessary under the
circumstances. One action may be to contact the ARTCC where the flight is located to ask the pilot to
contact the company.
[125]
If an intoxicated person creates a disturbance aboard an air carrier aircraft, the certificate holder must submit a
report, concerning the incident, to the Authority within
A. 7 days.
B. 5 days.
C. 48 hours.
Answer (A) is incorrect because, if an intoxicated person creates a disturbance aboard an air carrier aircraft,
the certificate holder must submit a report concerning the incident to the Authority within 5 days, not 7 days.
Answer (B) is correct. Each certificate holder shall, within 5 days after the incident, report to the Authority
any disturbance caused by a person who appears to be intoxicated aboard any of its aircraft.
Answer (C) is incorrect because, if an intoxicated person creates a disturbance aboard an air carrier aircraft,
the certificate holder must submit a report concerning the incident to the Authority within 5 days, not 48 hr.
[126]
By regulation, who shall provide the pilot in command of an air carrier airplane information concerning weather, and
irregularities of facilities and services?
A. The aircraft dispatcher.
B. Air route traffic control center.
C. Director of operations.
Answer (A) is correct. The aircraft dispatcher shall provide the pilot in command of an air carrier airplane all
available current reports or information on airport conditions and irregularities of navigation facilities that
may affect the safety of the flight.
Answer (B) is incorrect because, while an air route traffic control center may have information concerning
irregularities of facilities and service, it is not the proper source of that information. That information must be
provided by the aircraft dispatcher.
Answer (C) is incorrect because the director of operations is an administrative person responsible for the
day-to-day operations and is not involved in providing weather and other information to the pilot in
command. Providing that information is the responsibility of the aircraft dispatcher.
[127]
An alternate airport for departure is required
A. if weather conditions are below authorized landing minimums at the departure airport.
B. when the weather forecast at the ETD is for landing minimums only at the departure airport.
C. when destination weather is marginal VFR.
Answer (A) is correct. If the weather conditions at the departure airport are below the landing minimums in
the certificate holder's operations specifications for that airport, no person may dispatch or release an
aircraft from that airport unless the dispatch or flight release specifies an alternate airport.
Answer (B) is incorrect because an alternate airport for departure is required when the weather conditions,
not forecast, are below, not equal to, the authorized landing minim ums at the departure airport.
Answer (C) is incorrect because an alternate airport for departure is required if weather conditions at the
departure airport are below authorized landing minimums at the departure airport. Weather conditions at the
destination have no effect on whether a departure alternate airport is required.
[128]
What is the maximum distance that a departure alternate airport may be from the departure airport for a two-engine
airplane?
A. 1 hour at normal cruise speed in still air with both engines operating.
B. 1 hour at normal cruise speed in still air with one engine operating.
C. 2 hours at normal cruise speed in still air with one engine operating.
Answer (A) is incorrect because the maximum distance that a departure alternate airport may be from the
departure airport for a two-engine airplane is 1 hr. at normal cruising speed with one engine, not both
engines, operating.
Answer (B) is correct. The maximum distance that a departure alternate airport may be from the departure
airport for a two-engine airplane is 1 hr. at normal cruising speed in still air with one engine operating.
Answer (C) is incorrect because the maximum distance that a departure alternate airport may be from the
departure airport for an airplane having three or more engines, not a two-engine airplane, is 2 hr. at normal
cruising speed in still air with one engine inoperative.
[129]
When a departure alternate is required for a three-engine air carrier flight, it must be located at a distance not
greater than
A. 2 hours from the departure airport at normal cruising speed in still air with one engine not functioning.
B. 1 hour from the departure airport at normal cruising speed in still air with one engine inoperative.
C. 2 hours from the departure airport at normal cruising speed in still air.
Answer (A) is correct. When a departure alternate airport is required for a three-engine air carrier flight, it
must be located at a di stance not greater than 2 hr. from the departure airport at normal cruising speed in
still air with one engine inoperative.
Answer (B) is incorrect because, when a departure alternate airport is required for a two-engine, not a
three-engine, air carrier flight, it must be located at a di stance no greater than 1 hr. from the departure
airport at normal cruising speed in still air with one engine inoperative.
Answer (C) is incorrect because, when a departure alternate airport is required for a three-engine air carrier
flight, it must be located at a distance not greater than 2 hr. from the departure airport at normal cruising
speed in still air with one engine inoperative, not all engines operating.
[130]
If a four-engine air carrier airplane is dispatched from an airport that is below landing minimums, what is the
maximum distance that a departure alternate airport may be located from the departure airport?
A. Not more than 2 hours at cruise speed with one engine inoperative.
B. Not more than 2 hours at normal cruise speed in still air with one engine inoperative.
C. Not more than 1 hour at normal cruise speed in still air with one engine inoperative.
Answer (A) is incorrect because the maximum distance that a departure alternate airport may be located
from the departure airport for a four-engine air carrier airplane is not more than 2 hr. at normal cruising
speed, not any cruise speed, in still air with one engine inoperative.
Answer (B) is correct. If a four-engine air carrier airplane is dispatched from an airport that is below
authorized landing minimums, the maximum distance that a departure alternate airport may be located from
the departure airport is not more than 2 hr. at normal cruising speed in still air with one engine inoperative.
Answer (C) is incorrect because the maximum distance that a departure alternate airport may be located
from the departure airport for a two-engine, not a four-engine, air carrier airplane is not more than 1 hr. at
normal cruising speed in still air with one engine inoperative.
[131]
When the forecast weather conditions for a destination and alternate airport are considered marginal for an air
carrier's operation, what specific action should the dispatcher or pilot in comm and take?
A. List an airport where the forecast weather is not marginal as the alternate.
B. Add 1 additional hour of fuel based on cruise power settings for the airplane in use.
C. List at least one additional alternate airport.
Answer (A) is incorrect because the destination and first alternate can remain on the dispatch release when
the weather forecast is marginal for an air carrier's operation. If the weather is marginal, then at least one
additional alternate airport should be designated on the dispatch release.
Answer (B) is incorrect because, when the weather conditions forecast for a destination and alternate
airport are considered marginal for an air carrier's operation, at least one additional alternate airport should
be listed on the dispatch release. Additional fuel is not required by the forecast marginal weather conditions.
Answer (C) is correct. No person may dispatch an air carrier airplane under IFR or over-the-top unless (s)he
lists at least one alternate airport for each destination airport in the dispatch release. When the weather
conditions forecast for the destination and first alternate are marginal, at least one additional alternate
airport must be designated.
[132]
Prior to listing an airport as an alternate airport in the dispatch or flight release, weather reports and forecasts must
indicate that weather conditions will be at or above authorized minimums at that airport
A. for a period 1 hour before or after the ETA.
B. during the entire flight.
C. when the flight arrives.
Answer (A) is incorrect because, prior to listing an airport as an alternate in the dispatch or flight release,
weather reports and forecasts must indicate that weather conditions will be at or above authorized
minimums when the flight arrives, not for a period of 1 hr. before or after the ETA.
Answer (B) is incorrect because, prior to listing an airport as an alternate in the dispatch or flight release,
weather reports and forecasts must indicate that weather conditions will be at or above authorized
minimums when the flight arrives, not for the entire flight.
Answer (C) is correct. No person may list an airport as an alternate in the dispatch or flight release unless
the appropriate weather reports or forecasts, or any combination thereof, indicate that the conditions will be
at or above the alternate weather minimums specified in the certificate holder's operations specifications for
that airport when the flight arrives.
[133]
If a required instrument on a multiengine airplane becomes inoperative, which document dictates whether the flight
may continue en route?
A. An approved Minimum Equipment List for the airplane.
B. Original dispatch release.
C. Certificate holder's manual.
Answer (A) is incorrect because an approved MEL may authorize a takeoff with inoperative instruments or
equipment. It does not authorize a flight to continue after the failure of a required instrument en route.
Answer (B) is incorrect because a dispatch release contains flight-related information, not the procedures to
follow if a required instrument becomes inoperative while en route.
Answer (C) is correct. If any instrument or item of equipment (for the particular operation) becomes
inoperative en route, the pilot in command must comply with the approved procedures as specified in the
certificate holder's manual.
[134]
The pilot in command of an airplane en route determines that icing conditions can be expected that might adversely
affect safety of the flight. Which action is appropriate?
A. The pilot in command may continue to the original destination airport, after climbing to a higher altitude.
B. The pilot in command shall not continue flight into the icing conditions.
C. The flight may continue to the original destination airport, provided all anti-icing and deicing equipment is
operational and is used.
Answer (A) is incorrect because, if the pilot in command determines that icing conditions can be expected
that may compromise the safety of the flight, the pilot in command shall not continue flight into the expected
icing conditions regardless of altitude.
Answer (B) is correct. No person may dispatch or release an aircraft, continue to operate an aircraft en
route, or land an aircraft when, in the opinion of the pilot in command or aircraft dispatcher, icing conditions
are expected or met that might adversely affect the safety of the flight.
Answer (C) is incorrect because, if it is determined that icing can be expected that may compromise the
safety of the flight, the pilot in command m ay not continue regardless of the availability or use of anti-icing
or deicing equipment.
[135]
What action is required prior to takeoff if snow is adhering to the wings of an air carrier airplane?
A. Sweep off as much snow as possible and the residue must be polished smooth.
B. Assure that the snow is removed from the airplane.
C. Add 15 knot s to the normal VR speed as the snow will blow off.
Answer (A) is incorrect because the snow must be removed from the wings of the airplane and any residue
cannot be polished smooth.
Answer (B) is correct. No person may take off an air carrier airplane unless it has been checked to ensure
that the snow has been removed from the wings.
Answer (C) is incorrect because there is no authorized adjustment to VR to compensate for snow on the
wings. The snow must be removed prior to takeoff.
[136]
Under what conditions may an air carrier pilot continue an instrument approach to the DH, after receiving a weather
report indicating that less than minimum published landing conditions exist at the airport?
A. If the instrument approach is conducted in a radar environment.
B. When the weather report is received as the p lot passes the FAF.
C. When the weather report is received after the pilot has begun the final approach segment of the instrument
approach.
Answer (A) is incorrect because, in order to continue the approach, the pilot must have begun the final
approach segment. Whether or not the approach is in a radar environment is not a factor.
Answer (B) is incorrect because the question refers to an IAP with a DH, which implies a precision
approach. While a precision approach does not have a FAF, the final approach segment begins with the
intercept of the electronic glide slope.
Answer (C) is correct. If a pilot has begun the final approach segment of a precision instrument approach
procedure and after that receives a later weather report indicating below-minimum conditions, the pilot may
continue the approach to the DH.
[137]
Who is responsible for keeping copies of the certificate holder's manual up to date with approved changes or
additions?
A. Each of the certificate holder's employees who are furnished a manual.
B. An employee designated by the certificate holder.
C. A representative of the certificate holder approved by the Authority.
Answer (A) is correct. Each employee of the certificate holder to whom a manual (or portions of it) is furnish
ed is responsible for keeping his/her copy up-to-date with the changes and additions furnished to him/her.
Answer (B) is incorrect because all employees of the certificate holder to whom copies of the manual have
been furnished are responsible for keeping them up-to-date, not an employee designated by the certificate
holder.
Answer (C) is incorrect because a certificate holder must furnish a copy of the manual to a representative of
the Authority, and the certificate holder, not the inspector, is responsible for keeping the inspector's copy
up-to-date.
[138]
Where is the certificate holder required to list the name and title of each person authorized to exercise operational
control for a particular flight?
A. Operations Specifications.
B. Attached to the load manifest.
C. Certificate holder's manual.
Answer (A) is incorrect because the operations specifications prescribe the area in which the certificate
holder can operate, types of instrument approaches that can be flown, aircraft that can be us ed, etc., not
who can exercise operational control.
Answer (B) is incorrect because the load manifest contains specific flight data such as fuel on board,
passengers, cargo, etc., not the name of each person who can exercise operational control.
Answer (C) is correct. Each certificate holder is responsible for operational control and s hall list, in the
operations manual required, the name and title of each person authorized by it to exercise operational
control.
[139]
Who is responsible for the preparation of a required load manifest?
A. PIC or the Dispatcher.
B. Company official designated by the Authority
C. The certificate holder.
Answer (A) is incorrect because, while the PIC must carry a copy of the load manifest in the aircraft to its
destination, the PIC is not responsible for the preparation of the load manifest.
Answer (B) is incorrect because the certificate holder, not any official designated by the Authority, is
responsible for the preparation of a required load manifest.
Answer (C) is correct. For multiengine aircraft, each certificate holder is responsible for the preparation and
accuracy of a load manifest in duplicate containing information concerning the loading of the aircraft. The
load manifest must be prepared before each takeoff.
[140]
Which is NOT a required item on the load manifest?
A. List of passenger names and the weight of each.
B. Aircraft registration number or flight number.
C. Identification of crewmembers and their crew position.
Answer (A) is correct. A list of passenger names and their weights is not required on the load manifest; only
the number of passengers is required.
Answer (B) is incorrect because the aircraft registration number or flight number must be included on the
load manifest.
Answer (C) is incorrect because the identification of crewmembers and their crew position assignments
must be included on the load manifest.
[141]
Which is a requirement governing the carriage of cargo, on a scheduled passenger flight?
A. Cargo must be carried in an approved rack, bin, or compartment.
B. Cargo not stowed in an approved bin must be secured by a safety belt or approved tiedown device.
C. All cargo carried in the passenger compartment must be packaged and s towed ahead of the foremost seated
passenger.
Answer (A) is incorrect because cargo does not have to be carried in an approved bin if it is secured by a
safety belt or an approved tie-down device.
Answer (B) is correct. Cargo not stowed in an approved bin on a scheduled passenger flight must be
properly secured by a safety belt or other approved tie-down device having enough strength to eliminate the
possibility of shifting under all normally anticipated flight and ground conditions.
Answer (C) is incorrect because cargo may be carried in the passenger compartment under specified
conditions; it is not required to be stowed ahead of the foremost seated passenger.
[142]
Which is a requirement governing the carriage of carry-on baggage?
A. All carry-on baggage must be restrained so that its movement is prevented during air turbulence.
B. Carry-on baggage must be stowed under the seat in front of the owner.
C. Pieces of carry-on baggage weighing more than 10 pounds must be carried in an approved rack or bin.
Answer (A) is correct. All carry-on baggage must be restrained so that its movement is prevented during air
turbulence.
Answer (B) is incorrect because there is no requirement that carry-on baggage be stowed under the seat in
front of the owner.
Answer (C) is incorrect because there is no requirement that carry-on baggage weighing more than 10 lb.
be carried in an approved rack or bin.
[143]
If carry-on baggage or cargo is carried in the passenger compartment, it must be
A. stowed ahead of the foremost seated passengers and secured by approved means.
B. placed in an approved rack, bin, or compartment installed in the aircraft.
C. so located that it does not obstruct the access to, or the use of, any required emergency or regular exit.
Answer (A) is incorrect because there is no requirement that carry-on baggage be stowed ahead of the
foremost seated passengers.
Answer (B) is incorrect because carry-on baggage does not have to be placed in an approved rack, bin, or
compartment in the aircraft if it is carried in accordance with other specified conditions.
Answer (C) is correct. If carry-on baggage is not carried in an approved rack, bin, or compartment installed
in the aircraft, it must be located so that is does not obstruct the access to, or the use of, any required
emergency or regular exit.
[144]
At altitudes above 10,000 feet through 12,000 feet MSL, each pilot of an unpressurized airplane must use
supplemental oxygen for that part of the flight that is of a duration of more than
A. 20 minutes.
B. 30 minutes.
C. 45 minutes.
Answer (A) is incorrect because, for that part of the flight that exceeds 30, not 20, min. in duration from
10,000 ft. to 12,000 ft. MSL, oxygen is required.
Answer (B) is correct. Each pilot of an unpressurized aircraft shall use oxygen continuously when flying at
altitudes above 10,000 ft. MSL through 12,000 ft. MSL for that part of the flight at those altitudes that is of
more than 30 min. duration.
Answer (C) is incorrect because, for that part of the flight that exceeds 30, not 45, min. in duration from
10,000 ft. to 12,000 ft. MSL, oxygen is required.
[145]
The two pilot stations of a pressurized aircraft are equipped with approved quick-donning oxygen masks. What is
the maximum altitude authorized if one pilot is not wearing an oxygen mask and breathing oxygen?
A. 41,000 feet MSL.
B. 35,000 feet MSL.
C. 25,000 feet MSL.
Answer (A) is incorrect because at 41,000 ft. at least one pilot at the controls must wear, secured and
sealed, an oxygen mask that either supplies oxygen at all times or automatically supplies oxygen whenever
the cabin pressure altitude exceeds 12,000 ft. MSL.
Answer (B) is correct. The maximum altitude authorized if one pilot at the controls is not wearing an oxygen
mask and breathing oxygen in a pressurized aircraft equipped with quick-donning oxygen masks for both
pilot stations is 35,000 ft. MSL.
Answer (C) is incorrect because 25,000 ft. MSL is the maximum altitude authorized if one pilot at the
controls is not wearing an oxygen mask in an aircraft that does not have quick-donning oxygen masks for
both pilot stations.
[146]
Above which altitude/flight level must at least one of the two pilots, at the controls of a pressurized aircraft (with
quick-donning masks) wear a secured and sealed oxygen mask?
A. FL 300.
B. FL 350.
C. FL 250.
Answer (A) is incorrect because at least one of the two pilots at the controls of a pressurized aircraft,
equipped with quick-donning masks, must wear a secured and sealed oxygen mask above FL 350, not FL
300.
Answer (B) is correct. Whenever a pressurized aircraft equipped with quick-donning masks is operated
above FL 350, at least one of the two pilots at the controls must wear a secured and sealed oxygen mask
that either supplies oxygen at all times or automatically supplies oxygen whenever the cabin pressure
altitude exceeds 12,000 ft. MSL.
Answer (C) is incorrect because FL 250 is the maximum altitude/flight level above which at least one of the
two pilots of a pressurized aircraft must wear a secured and sealed oxygen mask, if the aircraft is not
equipped with quick-donning masks.
[147]
Which is a requirement for flight crew use of oxygen masks in a pressurized cabin airplane?
A. Both pilots at the controls shall use oxygen masks above FL 350.
B. At altitudes above 25,000 feet MSL, if one pilot leaves the pilot duty station, the remaining pilot at the controls
shall use an oxygen mask.
C. At altitudes above FL 250, one of the two pilots at the controls shall use an oxygen mask continuously.
Answer (A) is incorrect because at least one pilot, not both pilots, must use an oxygen mask above FL 350.
Answer (B) is correct. If one pilot leaves the duty station of a pressurized airplane when operating at
altitudes
above 25,000 ft. MSL, the remaining pilot at the controls must put on and use an approved oxygen mask
until the other pilot returns to the pilot duty station of the airplane.
Answer (C) is incorrect because one pilot is required to use an oxygen mask continuously above FL 250
only if the other pilot leaves the duty station.
[148]
A flight attendant crewmember is required on aircraft having a passenger seating configuration, excluding any pilot
seat, of
A. 15 or more.
B. 19 or more.
C. 20 or more.
Answer (A) is incorrect because aircraft with more than 19 seats, not 15, excluding any pilot seat, is the
requirement for a flight attendant.
Answer (B) is incorrect because the flight attendant requirement specifies "more than 19" not "19 or more."
Answer (C) is correct. No certificate holder may operate an aircraft that has a passenger seating
configuration, excluding any pilot seat, of more than 19 (i.e., 20 or more) unless there is a flight attendant
crewmember on board the aircraft.
[149]
Which aircraft must be equipped with an approved public address and crewmember interphone system?
A. All turbine-engine-powered aircraft having a seating configuration of more than 19 seats.
B. Aircraft having a passenger seating configuration, excluding any pilot seat, of more than 19 seats.
C. Multiengine aircraft having a passenger seating configuration of 10 seats or more.
Answer (A) is incorrect because the requirement for a public address and crewmember interphone system
is based on passenger seating, excluding any pilot seat, not the total airplane seating configuration.
Answer (B) is correct. All aircraft having a passenger seating configuration of more than 19 seats (excluding
any pilot seat) are required to be equipped with an approved public address and crewmember interphone
system.
Answer (C) is incorrect because passenger seating of 10 seats or more requires a second in command, not
a public address system.
[150]
The crewmember interphone system on a large turbojet-powered airplane, provides a means of two-way
communications between ground personnel and at least one of two flight crewmembers in the pilot compartment,
when the aircraft is on the ground. The interphone station for use by ground personnel must be located so that
those using the system from that station
A. are always visible from within the airplane.
B. are able to avoid the intake areas of the engines.
C. may avoid visible detection from within the airplane.
Answer (A) is incorrect because the interphone system for ground personnel use must be located to avoid
visible detection, not be visible, from within the airplane.
Answer (B) is incorrect because the interphone station for ground personnel use must be located to avoid
visible detection from within the airplane, not to avoid the intake areas of the engines.
Answer (C) is correct. When a large turbojet-powered airplane is on the ground, the crewmember
interphone system provides a means of two-way communication between ground personnel and at least
one of two flight crewmembers in the pilot compartment. The interphone station for use by ground personnel
must be located so that those using the system from that station may avoid visible detection from within the
airplane.
[151]
An approved cockpit voice recorder is required equipment in
A. large turbine-powered airplanes having a maximum passenger capacity of 20 or more seats.
B. multiengine, turbine-powered airplanes having a passenger seating configuration of 20 or more seats.
C. all aircraft operated in commuter air carrier service having a passenger seating configuration of 20 seats or more.
Answer (A) is incorrect because a cockpit voice recorder is required in multiengine, turbine-powered
airplanes, not only large turbine-powered airplanes, having a maximum passenger seating capacity of 20 or
more seats.
Answer (B) is correct. No person may operate a multiengine, turbine-powered airplane having a passenger
seating configuration of 20 or more seats unless it is equipped with an approved cockpit voice recorder.
Answer (C) is incorrect because a cockpit voice recorder is required in multiengine, turbine-powered
airplanes, not in all aircraft operated in commuter air carrier service, having a passenger seating
configuration of 20 or more seats.
[152]
Information recorded during normal operation of a cockpit voice recorder in a multiengine turbine-powered airplane
A. may all be erased or otherwise obliterated except for the last 30 minutes.
B. may all be erased or otherwise obliterated except for the last 30 minutes prior to landing.
C. may all be erased, prior to each flight, unless the Authority has requested that it be kept for 60 days.
Answer (A) is correct. Information recorded during normal operation of a cockpit voice recorder in a
multiengine turbine-powered airplane (having a seating configuration of 20 orm ore seats) may all be
erased or otherwise obliterated except for the last 30 min.
Answer (B) is incorrect because information recorded may be erased or otherwise obliterated except for the
last 30 min., not the last 30 min. prior to landing.
Answer (C) is incorrect because information recorded may be erased or otherwise obliterated except for the
last 30 min., not prior to each flight.
[153]
In which airplanes is a ground proximity warning system required?
A. All airplanes having a passenger seating configuration, excluding any pilot seat, of 10 seats or more.
B. Turbine-powered airplanes having a passenger seating configuration, excluding any pilot seat, of 10 seats or
more.
C. Turbine-powered aircraft having a passenger seating configuration, including any pilot seat, of 10 seats or more.
Answer (A) is incorrect because only a turbine-powered airplane, not a reciprocating airplane, is required to
have a ground proximity warning system if it has 10 passenger seats or more.
Answer (B) is correct. No person may operate a turbine-powered airplane having a passenger seating
configuration, excluding any pilot seat, of 10 seats or more, unless it is equipped with a ground proximity
warning system or a system that conveys warnings of excessive closure rates with the terrain and any
deviations below glide slope by visual and audible means.
Answer (C) is incorrect because pilot seats are excluded, not included, in the number of seats.
[154]
When a ground proximity warning system is required, it must
A. convey warnings of any deviation below glide slope and of excessive closure rate with the terrain.
B. convey warnings for excessive closure rates with the terrain but not for deviation from an ILS glide slope.
C. alert the pilot by an audible and visual warning signal when deviation above or below glide slope occurs.
Answer (A) is correct. No person may operate a turbine-powered airplane having a passenger seating
configuration, excluding any pilot seat, of 10 seats or more, unless it is equipped with a ground proximity
warning system or a system that conveys warnings of excessive closure rates with the terrain and any
deviations below glide slope by visual and audible means.
Answer (B) is incorrect because the ground proximity warning system must warn the pilot of both excessive
closure rates with the terrain and any deviations below glide slope.
Answer (C) is incorrect because the ground proximity warning system must alert the pilot by an audible and
visual warning signal of excessive closure rates with the terrain and any deviations below, not above, the
glide slope.
[155]
When a ground proximity warning system is required, it must
A. apply corrective control pressure when deviation below glide slope occurs.
B. incorporate a means of alerting the pilot when a system malfunction occurs.
C. incorporate a backup feature that activates automatically upon total failure of the aircraft's electrical generating
system.
Answer (A) is incorrect because there is no requirement for auto-flight or autopilot connection to the ground
proximity warning system. However, a means for alerting the pilot when a malfunction occurs is mandated
by the regulation.
Answer (B) is correct. A ground proximity warning system must have a means of alerting the pilot when a
malfunction occurs in the system.
Answer (C) is incorrect because no backup feature is required. Electrical failure does not mean that the
airplane is too close to the ground or below the glide slope.
[156]
In addition to fully-equipped life rafts and life preservers, what emergency equipment must be provided during
extended overwater operations?
A. One water resistant, self-buoyant, portable survival-type emergency radio transmitter for each life raft.
B. Each aircraft must have at least one life raft, equipped with a survival-type emergency locator transmitter.
C. One pyrotechnic signaling device for each aircraft.
Answer (A) is incorrect because only one, not each, life raft must be equipped with a survival-type
emergency locator transmitter.
Answer (B) is correct. No person may operate an aircraft in extended overwater operations unless there is
attached to one of the required life rafts a survival-type emergency locator transmitter.
Answer (C) is incorrect because a pyrotechnic signaling device is required for each life raft, not one for each
aircraft.
[157]
What emergency equipment is required for extended overwater operations?
A. A portable survival emergency locator transmitter for each life raft.
B. A pyrotechnic signaling device for each life preserver.
C. A life preserver equipped with a survivor locator light, for each person on the airplane.
Answer (A) is incorrect because an approved survival-type emergency locator transmitter is required to be
attached to only one life raft, not each life raft.
Answer (B) is incorrect because a pyrotechnic signaling device is required for each life raft, not each life
preserver.
Answer (C) is correct. Each airplane being operated in extended overwater operations must have a life
preserver equipped with an approved survivor locator light for each person on the airplane.
[158]
Each aircraft being operated in extended overwater operations, must have a life preserver for each
A. aircraft occupant.
B. seat on the aircraft.
C. passenger seat, plus 10 percent.
Answer (A) is correct. Each aircraft being operated in extended overwater operations must have a life
preserver equipped with an approved survivor locator light for each occupant of the aircraft.
Answer (B) is incorrect because a life preserver is required for each aircraft occupant, not for each seat on
the aircraft.
Answer (C) is incorrect because a life preserver is required for each aircraft occupant, not for each
passenger seat, plus 10%.
[159]
Life preservers required for extended overwater operations are stored
A. within easy reach of each passenger.
B. under each occupant seat.
C. within easy access of each seated occupant.
Answer (A) is incorrect because life preservers must be within easy access, not reach, of each seated
occupant, not only passengers.
Answer (B) is incorrect because life preservers must be within easy access of each seated occupant, not
necessarily under each occupant seat.
Answer (C) is correct. Life preservers required for extended overwater operations are stored within easy
access of each seated occupant.
[160]
An airplane, operated by a commuter air carrier, flying in extended overwater operations must carry enough
approved life rafts of a rated capacity and buoyancy to accommodate the occupants of the aircraft. Each life raft
must be equipped with
A. one approved pyrotechnic signaling device.
B. colored smoke flares and a signal mirror.
C. one fishing kit for each person the raft is rated to carry.
Answer (A) is correct. An airplane, operated by a commuter air carrier, flying in extended overwater
operations must carry enough approved life rafts of rated capacity and buoyancy to accommodate the
occupants of the aircraft. Each life raft must be equipped with one approved pyrotechnic signaling device.
Answer (B) is incorrect because colored smoke flares are not required emergency equipment for extended
overwater operations.
Answer (C) is incorrect because one fishing kit for each life raft, not each person the raft is rated to carry, is
required emergency equipment for extended overwater operations.
[161]
Which is a requirement for life preservers during extended overwater operations? Each life preserver must be
equipped with
A. a dye marker.
B. an approved survivor locator light.
C. one flashlight having at least two size "D" cells or equivalent.
Answer (A) is incorrect because the dye marker is for each life raft, not each life preserver.
Answer (B) is correct. No person may operate an aircraft in extended overwater operations unless it carries,
installed in conspicuously marked locations easily accessible to the occupants, an approved life preserver
equipped with an approved survivor locator light for each occupant of the aircraft. The life preserver must be
easily accessible to each seated occupant.
Answer (C) is incorrect because the flashlight and associated batteries are required for each life raft, not
each life preserver.
[162]
Which airplanes must have a shoulder harness installed at each flight crewmember station?
A. All airplanes used in commuter air service, having a passenger seating configuration of 9, excluding any pilot
seat.
B. All airplanes having a seating configuration for 10 persons.
C. All turbojet-powered airplanes.
Answer (A) is incorrect because the requirement is for all turbojet aircraft or aircraft having a passenger
seating configuration, excluding any pilot seat, of 10 seats or more, regardless of scheduled or on-demand
operations.
Answer (B) is incorrect because non turbojet-powered airplanes must have a passenger seating
configuration of 10 seats or more, excluding any pilot seat, not just a total of 10 seats in the airplane.
Answer (C) is correct. No person may operate a turbojet aircraft or an aircraft having a passenger seating
configuration, excluding any pilot seat, of 10 seats or more unless it is equipped with an approved shoulder
harness installed for each flight crewmember station.
[163]
Which group of aircraft must have a shoulder harness installed at each flight crewmember station?
A. Aircraft having a passenger seating configuration, excluding any pilot seat, of 10 seats or more.
B. All passenger-carrying aircraft having a seating configuration for 10 persons.
C. Large aircraft being operated in commuter air service, having a passenger seating configuration of 9, excluding
any pilot seat.
Answer (A) is correct. No person may operate a turbojet aircraft or an aircraft having a passenger seating
configuration, excluding any pilot seat, of 10 seats or more unless it is equipped with an approved shoulder
harness installed for each flight crewmember station.
Answer (B) is incorrect because the requirement is for all turbojet aircraft or aircraft having 10 passenger
seats or more, not all passenger-carrying aircraft.
Answer (C) is incorrect because the requirement is for all turbojet aircraft or aircraft having 10 passenger
seats or more, even if under 12 ,500 lb. gross takeoff weight, regardless of whether the aircraft is
conducting scheduled or on-demand operations.
[164]
When is a pilot not required to keep the shoulder harness fastened during takeoff and landing while at a pilot
station? A. When operating an aircraft having a passenger seating configuration, excluding any pilot seat, of 10
seats or less.
B. When the pilot cannot perform the required duties with the shoulder harness fastened.
C. When serving as pilot in command or second in command of an aircraft having a total seating capacity of eight
seats or less.
Answer (A) is incorrect because a flight crewmember may unfasten his/her shoulder harness to perform
required duties which cannot be performed when in a shoulder harness. The number of passenger seats
has no effect on this requirement.
Answer (B) is correct. Each flight crewmember occupying a station equipped with a shoulder harness must
fasten the shoulder harness during takeoff and landing, except that the shoulder harness may be
unfastened if the crewmember cannot perform the required duties with the shoulder harness fastened.
Answer (C) is incorrect because a flight crewmember may unfasten his/her shoulder harness to perform
required duties which cannot be performed when in a shoulder harness. The number of passenger seats
has no effect on this requirement.
[165]
In which aircraft, or under what conditions, is airborne thunderstorm detection equipment required?
A. Large multiengine turbine-powered aircraft having a passenger seating configuration of 19 seats or more being
operated by a commuter air carrier.
B. Any aircraft having a passenger seating configuration of 19 seats or more that is engaged in passenger-carrying
operations under IFR or at night.
C. Small aircraft having a passenger seating configuration of 10 seats or more, excluding any pilot seat, that are
engaged in passenger-carrying operations.
Answer (A) is incorrect because the requirement is for any airplane, whether single or multiengine, piston or
turbine-powered, that has a passenger seating configuration of 10, not 19, or more.
Answer (B) is incorrect because the requirement is for aircraft passenger seating of 10, not 19, or more
under any conditions, not just under IF R or at night.
Answer (C) is correct. No person may operate an aircraft that has a passenger seating configuration,
excluding any pilot seat, of 10 seats or more in passenger-carrying operations, unless the aircraft is
equipped with either approved thunderstorm detection equipment or approved airborne weather radar
equipment.
[166]
How many, if any, approved first aid kits are required on an aircraft having a passenger seating configuration of 20
seats and a passenger load of 14?
A. None.
B. One.
C. Two.
Answer (A) is incorrect because one approved first-aid kit is required on each aircraft with a seating
capacity of 20 passengers or more. The number of passengers aboard is irrelevant.
Answer (B) is correct. No person may operate an aircraft having a passenger seating configuration,
excluding any pilot seat, of more than 19 seats unless it is equipped with one approved first-aid kit for
treatment of injuries likely to occur in flight or in a minor accident.
Answer (C) is incorrect because one approved first-aid kit is required on each aircraft with a seating
capacity of 20 passengers or more. The number of passengers aboard is irrelevant.
[167]
An aircraft has a passenger seating configuration of 19 seats, excluding any pilot seats. How many, if any, approved
first aid kit s are required?
A. One.
B. Two.
C. None.
Answer (A) is incorrect because, if the passenger seating were more than 19, one first-aid kit would be
required. In this case, the configuration is 19 passenger seats, so no first-aid kit is required.
Answer (B) is incorrect because, if the passenger seating were more than 19, one first-aid kit, not two,
would be required. In this case, the configuration is 19 passenger seats, so no first-aid kit is required.
Answer (C) is correct. Approved first-aid kits are required only on aircraft with a seating configuration,
excluding any pilot seat, of more than 19 seats.
[168]
When a crash axe is required equipment on an aircraft, where should it be located?
A. In the flight crew compartment.
B. At a location inaccessible to the passengers during normal operations.
C. At a location accessible to both the crew and passengers during normal operations.
Answer (A) is incorrect because a crash axe must be located so that it is inaccessible to passengers during
normal operations but need not be in the flight crew compartment.
Answer (B) is correct. A crash axe must be located so that it is accessible to the crew but inaccessible to
passengers during normal operations.
Answer (C) is incorrect because a crash axe must be located so that it is inaccessible, not accessible, to
passengers during normal operations.
[169]
Which document would constitute an approved change to the type design without requiring a recertification?
A. An approved Minimum Equipment List.
B. The Operations Specifications as approved by the Authority.
C. A special flight permit.
Answer (A) is correct. An approved minimum equipment list (MEL), as authorized by the operations
specifications, constitutes an approved change to the type design without requiring recertification.
Answer (B) is incorrect because an approved MEL, not the operations specifications, constitutes an
approved change to the type design without requiring a recertification. The operations specifications
authorize the use of an MEL.
Answer (C) is incorrect because a special flight permit is issued to allow an aircraft to make a flight, under
specified conditions with inoperable instruments or equipment, to a place where maintenance can be
performed.
[170]
No person may takeoff an aircraft under IFR from an airport that has takeoff weather minimums but that is below
landing minimums unless there is an alternate airport within
A. 1 hour at normal indicated airspeed of the departure airport.
B. 1 hour at normal cruise speed in still air of the departure airport.
C. 1 hour at normal cruise speed in still air with one engine operating.
Answer (A) is incorrect because the alternate must be within 1 hr. at normal cruise speed in still air, not at
normal indicated airspeed.
Answer (B) is correct. No person may take off an aircraft under IFR from an airport where weather
conditions are at or above takeoff minimums but are below authorized IFR landing minimums unless there
is an alternate airport within 1 hr. flying time, at normal cruising speed in still air.
Answer (C) is incorrect because an alternate airport within 1 hr. at normal cruise speed in still air with one
engine operating is a requirement for a two-engine airplane.
[171]
A takeoff may not be made from an airport that is below the authorized IFR landing minimums unless
A. there is an alternate airport with the required IFR landing minimums within 60 minutes' flying time, at normal
cruising speed in still air.
B. the departure airport is forecast to have the required IFR landing minimums within 1 hour.
C. there is an alternate airport with the required IFR landing minim ums within 60 minutes' flying time, at normal
cruising speed in still air with one engine inoperative.
Answer (A) is correct. No person may take off an aircraft under IFR from an airport where weather
conditions are at or above takeoff minimums but are below authorized IFR landing minimums unless there
is an alternate airport within 1 hr. flying time, at normal cruising speed in still air.
Answer (B) is incorrect because the regulations state that an alternate airport is required within 1 hr. flying
time.
Answer (C) is incorrect because there is no requirement that flying time be calculated with one engine
inoperative since some operations are conducted with single-engine aircraft.
[172]
After passing the final approach fix on a VOR approach, a weather report is received indicating the visibility is below
prescribed minimums. In this situation, the pilot
A. may continue the approach and land, if at the MDA, the actual weather conditions are at least equal to the
minimums prescribed for the procedure.
B. may continue the approach and land regardless of the visibility observed at the MDA, if prior to beginning the
approach, the visibility was reported at or above minimums.
C. should level off and continue to fly the approach to the MAP, and execute the missed approach.
Answer (A) is correct. If a pilot has begun the final approach segment (e.g., passing the final approach fix
on a VOR approach) and a weather report indicates the visibility is below the prescribed minimums, the pilot
may continue the approach and land if, at the MDA, the pilot finds that the actual weather conditions are at
least equal to the minimums prescribed for the procedure.
Answer (B) is incorrect because, if at the MDA the weather is below published minimums, the pilot must
execute a miss ed approach.
Answer (C) is incorrect because, if at the MDA the weather conditions are at least equal to the published
minimums, the pilot should continue the approach and land, not level off and continue to fly to the MAP and
then execute the missed approach.
[173]
Which is an operational requirement concerning ice, snow, or frost on structural surfaces?
A. A takeoff may be with ice, snow, or frost adhering to the wings or stabilizing or control surfaces, but polished
smooth, if the anti-icing and deicing equipment is operating.
B. If snow, ice, or frost is adhering to the airplane's lift or control surfaces, but polished smooth, a takeoff may be
made.
C. A takeoff may not be made of ice or snow is adhering to the wings or stabilizing or control surfaces.
Answer (A) is incorrect because a takeoff may be made with frost, not ice or snow, adhering to the wings or
stabilizing or control surfaces, if the frost has been polished to make it smooth. There is no requirement that
the anti-icing and deicing equipment be operational.
Answer (B) is incorrect because a takeoff may be made with frost, not snow or ice, adhering to the wings or
stabilizing or control surfaces, if the frost has been polished smooth.
Answer (C) is correct. No pilot may take off an aircraft that has frost, ice, or snow adhering to any propeller,
windshield, wing, or stabilizing or control surface; to a power plant installation; or to an airspeed, altimeter,
rate-of-climb, or flight attitude instrument system.
[174]
What are the minimum certificate and rating requirements for the pilot in command of a multiengine airplane being
operated by a commuter air carrier?
A. Airline transport pilot; airplane category; multiengine class.
B. Commercial pilot; airplane category; multiengine class; instrument rating; airplane type rating, if required.
C. Airline transport pilot; airplane category; multiengine class; airplane type rating, if required.
Answer (A) is incorrect because, in addition to the category and class ratings, an aircraft type rating may be
required for that airplane.
Answer (B) is incorrect because, for the pilot in command of any multiengine aircraft used as a commuter
air carrier, an appropriate airline transport, not a commercial, pilot certificate is required.
Answer (C) is correct. No certificate holder may us e a person as pilot in command in passenger-carrying
operations of a multiengine airplane being operated by a commuter air carrier unless that person holds an
airline transport pilot certificate with appropriate category and class ratings and, if required, an appropriate
type rating for that airplane.
[175]
To satisfy the instrument approach recency experience requirement, a second in command must have made at
least
A. six approaches within the past 6 months; three must have been in the category aircraft to be flown.
B. six approaches within the past 6 months in any airplane, helicopter, approved instrument ground trainer, or
simulator.
C. six approaches and 6 hours of instrument time within the past 6 months, in an airplane, helicopter, approved
instrument ground trainer, or simulator.
Answer (A) is incorrect because all six of the approaches must be in the category of aircraft to be flown.
Answer (B) is correct. To satisfy the instrument approach recency of experience requirement, a second in
command must have made six approaches within the past 6 months in the same category of aircraft, or in a
flight simulator or flight training device that is representative of that aircraft category, for which instrument
privileges are sought.
Answer (C) is incorrect because there is no requirement for a minimum number of hours of instrument time
as long as six approaches in the same aircraft category (or an appropriate flight simulator or flight training
device) have been made.
[176]
What is one of the requirements that must be met by a pilot in command to reestablish recency of experience?
A. At least one full stop landing must be made from a circling approach.
B. Three takeoffs and landings must be made as the sole manipulator of the controls, in the type, if a type rating is
required, if not in the same category and class aircraft that the person is to serve.
C. At least one non precision approach must be made to the lowest minim ums authorized for the certificate holder.
Answer (A) is incorrect because one of the requirements that must be met to reestablish recency of
experience is three takeoffs and landings as sole manipulator of the controls, not a full stop landing from a
circling approach.
Answer (B) is correct. No person may serve as pilot in command of an aircraft carrying passengers unless,
within the preceding 90 days, that person has made three takeoffs and landings as the sole manipulator of
the flight controls in an aircraft of the same category and class and, if a type rating is required, of the same
type in which that person is to serve.
Answer (C) is incorrect because one of the requirements that must be met to reestablish recency of
experience is three takeoffs and landings as sole manipulator of the controls, not a non-precision approach
to the lowest minimums authorized for the certificate holder.
[177]
An employee who performs safety-sensitive functions, for a certificate holder, who has actual knowledge of an
accident involving an aircraft for which he or she performed a safety-sensitive function at or near the time of the
accident shall not use alcohol
A. until 4 hours after the accident.
B. within 8 hours of the accident.
C. until given a release by the Authority.
Answer (A) is incorrect because an employee who has actual knowledge of an accident for which he or she
performed a safety-sensitive function at or near the time of the accident must not use alcohol within 8 hr.,
not 4 hr., of the accident.
Answer (B) is correct. An employee who performs safety-sensitive functions for a certificate holder and who
has actual knowledge of an accident involving an aircraft for which (s)he performed a safety-sensitive
function at or near the time of the accident shall not use alcohol for 8 hr. following the accident, unless he or
she has been given a post-accident test or the employer has determined that the employee's performance
could not have contributed to the accident.
Answer (C) is incorrect because an employee can use alcohol before 8 hr. if he or she has been given a
post -accident test or the employer, not the Authority, has determined that the employee's performance
could not have contributed to the accident.
[178]
A person may not serve as pilot in command in an IFR operation unless that person has passed an
A. aircraft competency, an instrument proficiency, and auto-pilot check within the previous 6 calendar months prior
to the date to serve.
B. instrument proficiency check in the airplane in which to serve, or in an approved aircraft simulator, within the
previous 12 calendar months.
C. instrument proficiency check under actual or simulated IFR conditions, since the beginning of the 6th calendar
month prior to the date to serve.
Answer (A) is incorrect because the aircraft competency and autopilot checks must have been performed
within the previous 12, not 6, calendar months.
Answer (B) is incorrect because the instrument proficiency check is required within the previous 6, not 12,
calendar months.
Answer (C) is correct. No certificate holder may use a pilot, nor may any person serve, as a pilot in
command of an aircraft under IFR unless, since the beginning of the 6th calendar month before that service,
that pilot has passed an instrument proficiency check.
[179]
A person is assigned as pilot in command to fly both single-engine and multiengine airplanes and has passed the
initial instrument proficiency check in a multiengine airplane. Which requirement applies regarding each succeeding
instrument check?
A. The instrument check must be taken every 6 calendar months in both a single-engine and a multiengine airplane.
B. The instrument check must be taken alternately in single-engine and multiengine airplanes every 6 calendar
months.
C. The instrument check may be taken in either a single-engine or multiengine airplane if taken at intervals of 6
calendar months.
Answer (A) is incorrect because, after a pilot initially takes the required instrument proficiency check in a
multiengine airplane, subsequent instrument checks at 6-month intervals must be alternated between the
single-engine airplane and the multiengine airplane.
Answer (B) is correct. If the pilot in command is assigned to pilot both single-engine and multiengine
aircraft, that pilot must initially take the instrument proficiency check in a multiengine aircraft, and each
succeeding check alternately in single-engine and multiengine aircraft, but not more than one flight check
du ring each 6-month period.
Answer (C) is incorrect because each succeeding instrument check must alternate between the single-
engine and multiengine airplane, not be in either one.
[180]
No certificate holder may use a person as pilot in command unless that person has passed a line check
A. since the beginning of the 12th month before serving as pilot in command.
B. since the beginning of the 6th month before serving as pilot in command.
C. within the past 6 months.
Answer (A) is correct. No certificate holder may use a pilot, nor may any person serve, as a pilot in
command of a flight unless, since the beginning of the 12th calendar month before that service, that pilot
has passed a line check in one of the types of aircraft which that pilot is to fly.
Answer (B) is incorrect because an instrument proficiency check, not a line check, is required since the
beginning of the 6th month before serving as PIC.
Answer (C) is incorrect because a line check must have been passed since the beginning of the 12th month
before serving as PIC, not within the past 6 months.
[181]
When computing the takeoff data, what is the percentage of the reported headwind component that may be applied
to the "still air" data?
A. Not more than 150 percent.
B. Not more than 100 percent.
C. Not more than 50 percent.
Answer (A) is incorrect because takeoff data based on still air may be corrected by taking into account not
less than, instead of not more than, 150% of any reported tailwind, not headwind, component.
Answer (B) is incorrect because 100% is not a limit on headwinds or tailwinds for takeoff limitation
computations.
Answer (C) is correct. When computing takeoff limitations, takeoff data based on still air may be corrected
by taking into account not more than 50% of any reported headwind component.
[182]
When computing takeoff data, what is the percentage of the effective tailwind component which may be applied to
the "still air" data?
A. Not less than 150 percent.
B. Not less than 100 percent.
C. Not more than 50 percent.
Answer (A) is correct. To allow for wind effect, takeoff data based on still air may be corrected by taking into
account not more than 50% of any reported headwind component and not less than 150% of any reported
tailwind component.
Answer (B) is incorrect because 100 % is not a limit on headwinds or tailwinds for takeoff limitation
computations.
Answer (C) is incorrect because 50% is the limit on the headwind, not the tailwind, component.
[183]
Certain classes of hazardous material may be shipped by air but are not permitted aboard passenger-carrying
aircraft. How must such material be labeled?
A. DANGEROUS.
B. HAZARDOUS/CLASS X.
C. CARGO AIRCRAFT ONLY.
Answer (A) is incorrect because hazardous material not permitted aboard passenger-carrying aircraft must
be labeled "Cargo Aircraft Only," not "Dangerous. "
Answer (B) is incorrect because hazardous material not permitted aboard passenger-carrying aircraft must
be labeled "Cargo Aircraft Only," not "Hazardous/Class X."
Answer (C) is correct. If hazardous material as presented is not permitted aboard passenger-carrying
aircraft, it must be labeled with a "Cargo Aircraft Only" label.
[184]
The aircraft operator discovers that the label on a container of hazardous material is missing. How should the
appropriate replacement label be determined?
A. Shipping papers.
B. Hazardous material index.
C. Hazardous Materials Tables.
Answer (A) is correct. Lost or detached labels for packages of hazardous materials must be replaced in
accordance with the information provided on the shipping papers.
Answer (B) is incorrect because the shipping papers, not the hazardous material index, are used to
determine the contents of a package with a missing label.
Answer (C) is incorrect because the Hazardous Materials Tables are used to determine the labeling
requirements, but the shipping papers will identify the hazardous material in the container.
[185]
What is the maxim um allowable weight that may be carried on a pall et which has the dimensions of 36 x 48
inches?
A. 1,948.0 pounds.
B. 1,995.0 pounds.
C. 1,981.0 pounds.
[186]
What is the maximum allowable weight that may be carried on a pallet which has the dimensions of 76 x 74 inches?
A. 6, 767.8 pounds.
B. 6, 873.7 pounds.
C. 6, 796. 8 pounds.
[187]
What is the maximum allowable weight that may be carried on a pallet which has the dimensions of 81 x 83 inches?
Floor load limit - 180 lb./sq. ft
Pallet weight - 82 lb.
Tie-down devices - 31 lb.
A. 8, 403.7 pounds.
B. 8, 321.8 pounds.
C. 8, 290.8 pounds.
Answer (A) is incorrect because 8, 403.7 lb. is the maximum allowable weight of the pallet, which is a total
of the weight of the pallet and tie-down devices, plus the amount carried on the pallet.
Answer (B) is incorrect because 8, 321.8 lb. does not include the weight of the tie-down devices as part of
the total loaded pallet weight.
Answer (C) is correct.
Pallet area: 81 x 83 = 6, 723 sq. in.
Pallet sq. ft.: 6, 723 ÷ 144 = 46. 688 sq. ft.
Max. weight: 46. 688 x 180 = 8, 403.8 lb.
Max. cargo weight: 8, 403.8 - 82 - 31 =8, 290.8 lb.
[188]
What is the maximum allowable weight that may be carried on a pallet which has the dimensions of 36.5 x 48.5
inches?
A. 1331.8 pounds.
B. 1302.8 pounds.
C. 1347.8 pounds.
Answer (A) is incorrect because 1, 331.8 lb. does not include the weight of the pallet (45 lb.) as part of the
total loaded pallet weight.
Answer (B) is correct.
Pallet area: 36.5 x 48.5 = 1, 770.25 sq. in.
Pallet sq. ft.: 1, 770. 25 ÷ 144 = 12. 293 sq. ft.
Max. weight: 12. 293 x 112 = 1, 376. 8lb.
Max. cargo weight: 1, 376. 8 - 45 -29 = 1, 302. 8 lb.
Answer (C) is incorrect because 1, 347.8 lb. does not include the weight of the tie-down devices (29 lb.) as
part of the tot al loaded pallet weight.
[189]
What is the maximum allowable weight that may be carried on a pallet which has the dimensions of 42.6 x 48.7
inches?
A. 1, 710.2 pounds.
B. 1, 663.2 pounds.
C. 1, 696.2 pounds.
Answer (A) is incorrect because 1, 710.2 lb. does not include the weight of the tie-down devices (33 lb.) as
part of the total loaded pallet weight.
Answer (B) is correct.
Pallet area: 42.6 x 48.7 = 2, 074.62 sq. in.
Pallet sq. ft.: 2, 074.62 ÷ 144 = 14. 407 sq. ft.
Max. weight: 14.407 x 121 = 1, 743.2 lb.
Max. cargo weight: 1, 743.2 - 47 -33 = 1, 663.2 lb.
Answer (C) is incorrect because 1, 696.2 lb. does not include the weight of the pallet (47 lb.) as part of the
total loaded pallet weight.
[190]
What is the maximum allowable weight that may be carried on a pallet which has the dimensions of 24.6 x 68.7
inches?
A. 924.5 pounds.
B. 968.6 pounds.
C. 953.6 pounds.
[191]
What is the maximum allowable weight that may be carried on a pallet which has the dimensions of 34.6 x 46.4
inches?
A. 914.1 pounds.
B. 940.1 pounds.
C. 981.1 pounds.
[192]
What is the maximum allowable weight that may be carried on a pallet which has the dimensions of 33.5 x 48.5
inches?
A. 744.6 pounds.
B. 681.6 pounds.
C. 663.0 pounds.
Answer (A) is incorrect because 744.6 lb. is the maximum weight of the pallet, which includes the weight of
the pallet, tie-down devices, and cargo.
Answer (B) is correct.
Pallet area: 33.5 x 48.5 = 1, 624.75 sq. in.
Pallet sq. ft.: 1, 624.75 ÷ 144 = 11. 282 sq. ft.
Max. weight: 11. 282 x 66 = 744.6 lb.
Max. cargo weight: 744.6 - 34 - 29 = 681.6 lb.
Answer (C) is incorrect because 663.0 lb. is the maximum cargo weight if the floor load limit is 64 lb. /sq. ft.,
not 66 lb./sq. ft.
[193]
What is the maximum allowable weight that may be carried on a pallet which has the dimensions of 36.5 x 48.5
inches?
A. 1, 295.3 pounds.
B. 1, 212.3 pounds.
C. 1, 245.3 pounds.
Answer (A) is incorrect because 1, 295.3 lb. is the maximum cargo weight if the floor load limit is 111 lb./sq.
ft., not 107 lb./sq. ft.
Answer (B) is incorrect because 1, 212.3 lb. is the maximum cargo weight if the floor load limit is 104 lb./sq.
ft., not 107 lb./sq. ft.
Answer (C) is correct.
Pallet area: 36.5 x 48.5 = 1, 770.25 sq. in.
Pallet sq. ft.: 1, 770.25 ÷ 144 = 12. 293 sq. ft.
Max. weight: 12. 293 x 107 = 1, 315.3 lb.
Max. cargo weight: 1, 315.3 - 37 -33 = 1, 245.3 lb.
[194]
What is the maximum allowable weight that may be carried on a pallet which has the dimensions of 42.6 x 48.7
inches?
A. 1, 611.6 pounds.
B. 1, 654.6 pounds.
C. 1, 601.6 pounds.
[195]
What is the maximum allowable weight that may be carried on a pallet which has the dimensions of 24.6 x 68.7
inches?
A. 884.1 pounds.
B. 857.1 pounds.
C. 841.1 pounds.
Answer (A) is incorrect because 884.1 lb. does not include the weight of the pallet (43 lb.) as part of the total
pallet weight.
Answer (B) is correct.
Pallet area: 24.6 x 68.7 = 1, 690.02 sq. in.
Pallet sq. ft.: 1, 690.02 ÷ 144 = 11. 736 sq. ft.
Max. weight: 11. 736 x 79 = 927.1 lb.
Max. cargo weight: 927.1 - 43 - 27 = 857.1 lb.
Answer (C) is incorrect because 841 1 lb. is the maximum cargo weight if the floor load limit is 78 lb./sq. ft.,
not 79 lb./sq. ft.
[196]
What is the maximum allowable weight that may be carried on a pallet which has the dimensions of 143 x 125.2
inches?
Answer (A) is incorrect because 25, 984.9 lb. is the maximum weight of the pallet, which includes the weight
of the pallet, tie-down devices, and cargo.
Answer (B) is incorrect because 25, 787.9 lb. does not include the weight of the pallet (197 lb.) as part of
the tot al loaded pallet weight.
Answer (C) is correct.
Pallet area: 143 x 125.2 = 17, 903.6 sq. in.
Pallet sq. ft.: 17, 903.6 ÷ 144 = 124.33 sq. ft.
Max. weight: 124.33 x 209 = 25, 984.9lb.
Max. cargo weight: 25, 984.9 - 197 - 66 = 25, 721.9 lb.
[197]
What is the maximum allowable weight that may be carried on a pallet which has the dimensions of 138.5 x 97.6
inches?
Answer (A) is incorrect because 21, 840.9 lb. does not include the weight of the pallet (219 lb.) as part of
the total loaded pallet weight.
Answer (B) is correct.
Pallet area: 138.5 x 97.6 = 13, 517.6 sq. in.
Pallet sq. ft.: 13, 517.6 ÷ 144 = 93. 87 sq. ft.
Max. weight: 93.87 x 235 = 22, 059.9 lb.
Max. cargo weight: 22, 059.9 - 219 - 71 = 21, 769.9 lb.
Answer (C) is incorrect because 22, 059.9 lb. is the maximum weight of the pallet, which includes the weight
of the pallet, tie-down devices, and cargo.
[198]
What is the maximum allowable weight that may be carried on a pallet which has the dimensions of 96.1 x 133.3
inches?
[199]
What is the maximum allowable weight that may be carried on a pallet which has the dimensions of 87.7 x 116.8
inches?
[200]
What is the maximum allowable weight that may be carried on a pallet which has the dimensions of 98.7 x 78.9
inches?
A. 9, 896.5 pounds.
B. 9, 735.5 pounds.
C. 9, 681.5 pounds.
Answer (A) is incorrect because 9, 896.5 lb. is the maximum weight of the pallet, which includes the weight
of the pallet, tie-down devices, and cargo.
Answer (B) is incorrect because 9, 735.5 lb. does not include the weight of the pallet as part of the total
loaded pallet weight.
Answer (C) is correct.
Pallet area: 98.7 x 78.9 = 7, 787.4 sq. in.
Pallet sq. ft.: 7, 787.4 ÷ 144 = 54.08 sq. ft.
Max. weight: 54.08 x 183 = 9, 896.5 lb.
Max. cargo weight: 9, 896.5 - 161 - 54 = 9, 681.5 lb.
[201]
What minimum floor load limit must an aircraft have to carry the following pallet of cargo?
Answer (A) is incorrect because a minimum floor load limit of 182 lb./sq. ft. does not include the weight of
the tie-down devices (54 lb.) as part of the pallet net weight.
Answer (B) is incorrect because a minimum floor load limit of 180 lb./sq. ft. does not include the weight of
the pallet (161 lb.) as part of the pallet net weight.
Answer (C) is correct.
Pallet area: 78.9 x 98.7 = 7, 787.4 sq. in.
Pallet sq. ft.: 7, 787.4 ÷ 144 = 54.08 sq. ft.
Pallet net weight: 161 + 54 + 9, 681.5 = 9, 896.5 lb.
Min. floor load limit: 9, 896.5 ÷ 54.08 = 183 lb./sq. ft.
[202]
What is the minimum floor load limit that an aircraft must have to carry the following pallet of cargo?
[203]
What is the minimum floor load limit that an aircraft must have to carry the following pallet of cargo?
Answer (A) is incorrect because a minimum floor load limit of 152 lb./sq. ft. is required for a pallet net weight
of 1, 384.7 lb., not 1, 312.4 lb.
Answer (B) is incorrect because a minimum floor load limit of 148 lb./sq. ft. is required for a pallet net weight
of 1, 348.3 lb., not 1, 312.4 lb.
Answer (C) is correct.
Pallet area: 37.5 x 35 = 1, 312.5 sq. in.
Pallet sq. ft.: 1, 312.5 ÷ 144 = 9.11 sq. ft.
Pallet net weight: 34 + 23 + 1, 255.4 = 1, 312.4lb.
Min. floor load limit: 1, 312.4 ÷ 9.11 = 144 lb./sq. ft.
[204]
What is the minimum floor load limit that an aircraft must have to carry the following pallet of cargo?
A. 79 lb./sq. ft.
B. 76 lb./sq. ft.
C. 73 lb./sq. ft.
Answer (A) is incorrect because a minimum floor load limit of 79 lb./sq. ft. is required for a pallet net weight
of 891.1 lb., not 857.5 lb.
Answer (B) is correct.
Pallet area: 48.5 x 33.5 = 1, 624.75 sq. in.
Pallet sq. ft.: 1, 624.75 ÷ 144 = 11.28 sq. ft.
Pallet net weight: 44 + 27 + 786.5 = 857.5 lb.
Min. floor load limit: 857.5 ÷ 11.28 = 76 lb./sq. ft.
Answer (C) is incorrect because a minimum floor load limit of 73 lb./sq. ft. does not include the weight of the
tie-down devices (27 lb.) as part of the pallet net weight.
[205]
What is the minimum floor load limit that an aircraft must have to carry the following pallet of cargo?
Answer (A) is incorrect because a minimum floor load limit of 172 lb./sq. ft. is required for a pallet net weight
of 12, 234.4 lb., which is less than the cargo weight of 12, 262.4 lb. The pallet net weight includes the cargo
weight, the pallet weight, and the weight of the tie-down devices.
Answer (B) is correct.
Pallet area: 116.8 x 87.7 = 10, 243.36 sq. in.
Pallet sq. ft.: 10, 243.36 ÷ 144 = 71.13 sq. ft.
Pallet net weight: 137 + 49 + 12, 262.4 = 12, 448.4 lb.
Min. floor load limit: 12, 448.4 ÷ 71.13 = 175 lb./sq. ft.
Answer (C) is incorrect because a minimum floor load limit of 179 lb. /sq. ft. is required for a pallet net
weight of 12, 732.3 lb., not 12, 448. 4 lb.
[206]
What is the minimum floor load limit that an aircraft must have to carry the following pallet of cargo?
Answer (A) is incorrect because a minimum floor load limit of 180 lb./sq. ft. does not include the weight of
the pallet (161 lb.) as part of the pallet net weight.
Answer (B) is incorrect because a mini mum floor load limit of 186 lb./sq. ft. is required for a pallet net
weight of 10, 059 lb., not 9, 896.5 lb.
Answer (C) is correct.
Pallet area: 78.9 x 98.7 = 7, 787.43 sq. in.
Pallet sq. ft.: 7, 787.43 ÷ 144 = 54.08 sq. ft.
Pallet net weight: 161 + 54 + 9, 681.5 = 9, 896.5 lb.
Min. floor load limit: 9, 896.5 ÷54.08 = 183 lb. /sq. ft.
[207]
What will be the ratio between airspeed and lift if the angle of attack and other factors remain constant and airspeed
is doubled? Lift will be
A. the same.
B. two times greater.
C. four times greater.
Answer (A) is incorrect because lift is proportional to the square of the airplane's velocity; it is not constant.
Answer (B) is incorrect because, as airspeed is doubled, the lift will be four times greater, not doubled.
Answer (C) is correct. Lift is proportional to the square of the airplane's velocity, if the angle of attack and
other factors remain constant. For example, an airplane traveling at 200 kt. has four times the lift of the
same airplane traveling at 100 kt.
[208]
What true airspeed and angle of attack should be used to generate the same amount of lift as altitude is increased?
A. The same true airspeed and angle of attack.
B. A higher true airspeed for any given angle of attack.
C. A lower true airspeed and higher angle of attack.
Answer (A) is incorrect because true airspeed must be increased, not constant, as altitude increases.
Answer (B) is correct. At an altitude of 18, 000 ft., the air has one-half the density of air at sea level. Thus, in
order to maintain the same amount of lift as altitude increases, an airplane must fly at a greater true
airspeed for any given angle of attack.
Answer (C) is incorrect because true airspeed must increase, not decrease, for any given angle of attack.
[209]
What is the effect on total drag of an aircraft if the airspeed decreases in level flight below that speed for maximum
L/D?
A. Drag increases because of increased induced drag.
B. Drag increases because of increased parasite drag.
C. Drag decreases because of lower induced drag.
Answer (A) is correct. Total drag is at a minimum for the maximum lift-drag ratio (L/D max) at one specific
angle of attack and lift coefficient. As airspeed decreases, the induced drag will increase because a greater
angle of attack is required to maintain the lift coefficient. The amount of induced drag varies inversely as the
square of the airspeed.
Answer (B) is incorrect because parasite drag varies directly, not inversely, with airspeed.
Answer (C) is incorrect because drag increases, not decreases, from any speed other than that for
maximum L/D.
[210]
Describe dynamic longitudinal stability.
A. Motion about the longitudinal axis.
B. Motion about the lateral axis.
C. Motion about the vertical axis.
Answer (A) is incorrect because motion about the airplane's longitudinal axis is lateral, not longitudinal
stability.
Answer (B) is correct. Longitudinal stability is the quality which makes an airplane stable about its lateral
axis. It involves the pitching motion as the airplane's nose moves up and down in flight. Dynamic stability is
the overall tendency that the airplane displays after its equilibrium is disturbed.
Answer (C) is incorrect because motion about the vertical axis is directional, not longitudinal stability.
[211]
What is a characteristic of longitudinal instability?
A. Pitch oscillations becoming progressively greater.
B. Bank oscillations becoming progressively greater.
C. Aircraft constantly tries to pitch down.
Answer (A) is correct. Longitudinal stability is the quality which makes an airplane stable about its lateral
axis (pitching). A longitudinally unstable airplane has a tendency to dive or climb (oscillations) progressively
into a very steep dive or climb.
Answer (B) is incorrect because longitudinal stability refers to pitch, not bank, oscillations.
Answer (C) is incorrect because the aircraft constantly trying to pitch down is a characteristic of improper
elevator trim, not longitudinal instability.
[212]
Identify the type stability if the aircraft attitude remains in the new position after the controls have been neutralized.
A. Negative longitudinal static stability.
B. Neutral longitudinal dynamic stability.
C. Neutral longitudinal static stability.
Answer (A) is incorrect because a negative longitudinal static stability means the airplane tends to move
even farther from the original position.
Answer (B) is incorrect because, with neutral longitudinal dynamic stability, the airplane continues to
oscillate without a tendency to increase or decrease.
Answer (C) is correct. If the airplane's nose remains in the new position after the elevator control is pressed
forward and released, the aircraft displays neutral longitudinal static stability.
[213]
Identify the type stability if the aircraft attitude tends to return to its original position after the controls have been
neutralized.
A. Positive dynamic stability.
B. Positive static stability.
C. Neutral dynamic stability.
Answer (A) is incorrect because positive dynamic stability refers to oscillations being dampened or
decreasing.
Answer (B) is correct. Imagine that you trim the airplane for hands-off control in level flight and then
momentarily give the controls a slight push to nose the airplane down. If within a brief period the nose rises
to the original position and then stops, the airplane is statically stable. It is positively statically stable
because it has returned to its original position after being displaced.
Answer (C) is incorrect because neutral dynamic stability refers to oscillations continuing without a tendency
to increase or decrease.
[214]
What characteristic should exist if an airplane is loaded to the rear of its CG range?
A. Sluggish in aileron control.
B. Sluggish in rudder control.
C. Unstable about the lateral axis.
Answer (A) is incorrect because an aft location of the CG has a greater effect on the longitudinal stability,
not the lateral (aileron) controllability.
Answer (B) is incorrect because an aft CG has a greater effect on the longitudinal stability, not vertical
(rudder) controllability.
Answer (C) is correct. The rearward CG limit is largely determined by considerations of stability. When
loaded to the rear of its CG limit, an airplane will become unstable about the lateral axis, i.e., longitudinally
unstable. At this point when the nose is momentarily pulled up, it may alternately climb and dive, becoming
steeper with each oscillation.
[215]
What affects indicated stall speed?
A. Weight, load factor, and power.
B. Load factor, angle of attack, and power.
C. Angle of attack, weight, and air density.
Answer (A) is correct. An airplane's indicated stall speed is affected by total airplane weight, weight
distribution within the airplane, angle of bank, pitch attitude, control coordination, drag, and power settings,
but NOT by angle of attack or air density.
Answer (B) is incorrect because indicated stall speed is not affected by the angle of attack.
Answer (C) is incorrect because indicated stall speed is not affected by angle of attack or air density.
[216]
How can turbulent air cause an increase in stalling speed of an airfoil?
A. An abrupt change in relative wind.
B. A decrease in angle of attack.
C. Sudden decrease in load factor.
Answer (A) is correct. Turbulence can cause an aircraft to stall at a significantly higher airspeed than in
stable conditions. A vertical gust or wind shear can cause a sudden change in the relative wind and result in
an abrupt increase in the angle of attack and, thus, result in a stall. Although a gust may not be maintained
long enough for a stall to develop, the aircraft may stall while the pilot is attempting to control the flight path,
particularly during an approach in gusty conditions.
Answer (B) is incorrect because decreased angle of attack decreases, not increases, the possibility of a
stall.
Answer (C) is incorrect because a sudden decrease in load factor decreases, not increases, the stalling
speed.
[217]
What is load factor?
A. Lift multiplied by the tot al weight.
B. Lift subtracted from the total weight.
C. Lift divided by the total weight.
Answer (A) is incorrect because lift is divided, not multiplied, by the total weight.
Answer (B) is incorrect because lift is divided by, not subtracted from, the total weight.
Answer (C) is correct. A load factor is the ratio of the total airload acting on the airplane to the gross weight
of the airplane. For example, if an airplane has a gross weight of 2, 000 lb. and during flight is subjected to
aerodynamic forces which increase the total load the wing must support to 4, 000 lb., the load factor is 2
(4,000 ÷ 2, 000). In this example, the airplane wing is producing lift that is equal to twice the gross weight of
the airplane.
[218]
If an aircraft with a gross weight of 2, 000 pounds were subjected to a total load of 6, 000 pounds in flight, the load
factor would be
A. 2 G's.
B. 3 G's.
C. 9 G's.
[219]
Upon which factor does wing loading during a level coordinated turn in smooth air depend?
A. Rate of turn.
B. Angle of bank.
C. T rue airspeed.
Answer (A) is incorrect because, in a coordinated turn, the rate of turn does not have any impact on the load
factor, which is determined solely by the angle of bank.
Answer (B) is correct. The load factor for a given airplane during a level coordinated turn is determined
solely by the angle of bank.
Answer (C) is incorrect because true airspeed has no impact on the load factor.
[220]
What effect does an increase in airspeed have on a coordinated turn while maintaining a constant angle of bank
and altitude?
A. The rate of turn will decrease resulting in a decreased load factor.
B. The rate of turn will increase resulting in an increased load factor.
C. The rate of turn will decrease resulting in no changes in load factor.
Answer (A) is incorrect because, at a constant bank angle, the higher airspeed will decrease the rate of turn
to compensate for added centrifugal force, allowing the load factor to remain the same, not decrease.
Answer (B) is incorrect because, for any bank angle, the rate of turn varies with the airspeed; the higher the
speed, the slower, not faster, the rate of turn.
Answer (C) is correct. Increasing airspeed while maintaining a coordinated turn with a constant angle of
bank and altitude will decrease the rate of turn. For any given bank angle, the rate of turn varies with the
airspeed; the higher the speed, the slower the rate of turn. This variation compensates for added centrifugal
force, allowing the load factor to remain the same.
[221]
Identify the type stability if the aircraft attitude tends to move farther from its original position after the controls have
been neutralized.
A. Negative static stability.
B. Positive static stability.
C. Negative dynamic stability.
Answer (A) is correct. When the airplane's nose moves farther from its original position, it has the
undesirable characteristic of negative static stability.
Answer (B) is incorrect because, if the aircraft had positive static stability, it would return to its original
position.
Answer (C) is incorrect because negative dynamic stability refers to oscillations that become progressively
larger.
[222]
What flight condition should be expected when an aircraft leaves ground effect?
A. An increase in induced drag requiring a higher angle of attack.
B. A decrease in parasite drag permitting a lower angle of attack.
C. An increase in dynamic stability.
Answer (A) is correct. The reduction of the wingtip vortices due to ground effect alters the spanwise lift
distribution and reduces the induced angle of attack and induced drag. When leaving ground effect, the
airplane will require an increase in angle of attack to maintain the same lift coefficient, which will increase
induced drag.
Answer (B) is incorrect because, at slow airspeeds when taking off, induced, not parasite, drag
predominates.
Answer (C) is incorrect because, when leaving ground effect, expect a decrease, not increase, in stability
and a nose-up change in moment.
[223]
Which condition reduces the required runway for takeoff?
A. Higher-than-recommended airspeed before rotation.
B. Lower-than-standard air density.
C. Increased headwind component.
Answer (A) is incorrect because higher-than-recommended airspeed before rotation increases, not reduces,
the required runway for takeoff.
Answer (B) is incorrect because lower-than-standard air density increases, not decreases, the required
runway for takeoff.
Answer (C) is correct. The effect of a headwind is to allow the airplane to reach lift-off speed at a lower
groundspeed. A headwind which is 10% of the takeoff airspeed will reduce the takeoff distance
approximately 19%.
[224]
What effect does landing at high elevation airports have on groundspeed with comparable conditions relative to
temperature, wind, and airplane weight?
A. Higher than at low elevation.
B. Lower than at low elevation.
C. The same as at low elevation.
Answer (A) is correct. An airplane at altitude will land at the same indicated airspeed as at sea level but,
because of the reduced density (comparable conditions), the true airspeed (TAS) will be greater. Thus, with
a higher TAS, the ground speed will be higher at high elevation airports than at low elevation.
Answer (B) is incorrect because, at high elevation, there is reduced air density, and thus TAS will increase.
As TAS increases, groundspeed will increase, not decrease.
Answer (C) is incorrect because, under comparable conditions, TAS will increase and cause the
groundspeed to be higher, not the same, at higher elevation.
[225]
How can an airplane produce the same lift in ground effect as when out of ground effect?
A. The same angle of attack.
B. A lower angle of attack.
C. A higher angle of attack.
Answer (A) is incorrect because, if the same angle of attack is maintained, an increase in lift coefficient will
result.
Answer (B) is correct. The reduction of the wingtip vortices due to ground effect alters the spanwise lift
distribution and reduces the induced angle of attack and induced drag. The wing will require a lower angle
of
attack in ground effect to produce the same lift coefficient as when out of ground effect.
Answer (C) is incorrect because a lower, not higher, angle of attack is required to produce the same lift in
ground effect as out of ground effect.
[226]
If no corrective action is taken by the pilot as angle of bank is increased, how is the vertical component of lift and
sink rate affected?
A. Lift increases and the sink rate increases.
B. Lift decreases and the sink rate decreases.
C. Lift decreases and the sink rate increases.
[227]
What is the relationship of the rate of turn with the radius of turn with a constant angle of bank but increasing
airspeed? A. Rate will decrease and radius will increase.
B. Rate will increase and radius will decrease.
C. Rate and radius will increase.
Answer (A) is correct. As airspeed is increased, the angle of attack needs to be decreased, which will cause
the rate of turn to decrease. At the same time, the increase in airspeed results in an increase of the turn
radius.
Answer (B) is incorrect because, to maintain a constant angle of bank while increasing airspeed, the radius
of turn will increase, not decrease.
Answer (C) is incorrect because, to maintain a constant angle of bank while increasing airspeed, the rate of
turn will decrease, not increase.
[228]
How can the pilot increase the rate of turn and decrease the radius at the same time?
A. Steepen the bank and increase airspeed.
B. Steepen the bank and decrease airspeed.
C. Shallow the bank and increase airspeed.
Answer (A) is incorrect because a decrease, not increase, in airspeed decreases the turn radius.
Answer (B) is correct. At slower airspeeds, an airplane can make a turn in less distance (smaller radius) and
at a faster rate. Thus, to decrease the radius and increase the rate, one steepens the bank and decreases
airspeed.
Answer (C) is incorrect because a steeper, not shallower, bank and a decrease, not increase, in airspeed
increase the rate of turn.
[229]
Why must the angle of attack be increased during a turn to maintain altitude?
A. Compensate for loss of vertical component of lift.
B. Increase the horizontal component of lift equal to the vertical component.
C. Compensate for increase in drag.
Answer (A) is correct. The lift during a turn is divided into two components, one vertical and the other
horizontal. This divisi on reduces the amount of lift which is opposing gravity. The angle of attack must be
increased until the vertical component of lift is equal to the weight in order to maintain altitude.
Answer (B) is incorrect because angle of attack is increased in order to increase the vertical component of
lift to equal weight.
Answer (C) is incorrect because additional thrust (power), not angle of attack, is used to compensate for
increase in drag.
[230]
By changing the angle of attack of a wing, the pilot can control the airplane's
A. lift, gross weight, and drag.
B. lift, airspeed, and drag.
C. lift and airspeed, but not drag.
Answer (A) is incorrect because angle of attack cannot control the airplane's gross weight.
Answer (B) is correct. The pilot can control the airplane's lift, airspeed, and drag. As the angle of attack is
increased, the lift increases to the stalling angle of attack, airspeed decreases, and induced drag increases
with the increase in lift.
Answer (C) is incorrect because the pilot can control the amount of induced drag by changing the angle of
attack.
[231]
What are some characteristics of an airplane loaded with the CG at the aft limit?
A. Lowest stall speed, highest cruise speed, and least stability.
B. Highest stall speed, highest cruise speed, and least stability.
C. Lowest stall speed, lowest cruise speed, and highest stability.
Answer (A) is correct. The airplane will stall at a higher speed with a forward CG location. The critical angle
of attack is reached at a higher speed due to increased wing loading. Conversely, the airplane will stall at a
lower speed with an aft CG loading. The airplane will cruise faster with an aft CG location because of
reduced drag. The airplane will become less stable as the CG is moved rearward.
Answer (B) is incorrect because an aft CG causes the airplane to have the lowest, not highest, stall speed.
Answer (C) is incorrect because an aft CG causes an airplane to have the highest, not lowest, cruise speed
and the least, not highest, stability.
[232]
What is the most effective way to use the eyes during night flight?
A. Look only at far away, dim lights.
B. Scan slowly to permit off-center viewing.
C. Concentrate directly on each object for a few seconds.
Answer (A) is incorrect because pilots must look at their gauges and instruments, which are about 2 ft. in
front of them.
Answer (B) is correct. Physiologically, the eyes are most effective at seeing objects off center at night.
Accordingly, a pilot should scan slowly to permit off-center viewing.
Answer (C) is incorrect because peripheral (off-center) vision is more effective at night.
[233]
What result does a level turn have on the total lift force and load factor?
A. Lift force remains constant and the load factor increases.
B. Lift force increases and the load factor decreases.
C. Both total lift force and load factor increase.
Answer (A) is incorrect because lift increases, not remains constant, in level turns.
Answer (B) is incorrect because load factor increases, not decreases, in level turns.
Answer (C) is correct. During a level turn, both total lift and load factor increase.
[234]
Where is a common location for an inversion?
A. At the tropopause.
B. In the stratosphere.
C. At the base of cumulus clouds.
Answer (A) is incorrect because a common location for an inversion is in the stratosphere, not at the
tropopause.
Answer (B) is correct. Inversions may occur at any altitude and are comm on in the stratosphere. An
inversion is an increase in temperature with an increase in altitude.
Answer (C) is incorrect because the base of cumulus clouds is where the dew point lapse rate and the dry
adiabatic lapse rate converge. It is not a common location for an inversion.
[235]
What is a characteristic of the troposphere?
A. It contains all the moisture of the atmosphere.
B. There is an overall decrease of temperature with an increase of altitude.
C. The average altitude of the top of the troposphere is about 6 miles.
Answer (A) is incorrect because moisture can be found in the stratosphere, as evidenced by some of the
largest thunderstorms.
Answer (B) is correct. The troposphere is the layer from the surface to an average altitude of 7 mi. It is
characterized by an overall decrease of temperature with an increase in altitude.
Answer (C) is incorrect because the average altitude of the top of the troposphere is about 7 mi., not 6 mi.
[236]
What is the primary cause of all changes in the Earth's weather?
A. Variations of solar energy at the Earth' s surface.
B. Changes in air pressure over the Earth' s surface.
C. Movement of air masses from moist areas to dry areas.
Answer (A) is correct. Every physical process of weather is accompanied by or is the result of a heat
exchange. Unequal solar heating of the Earth' s surface causes differences in air pressure which result in all
changes in the Earth' s weather.
Answer (B) is incorrect because changes in air pressure are a result of the unequal solar heating of the
Earth's surface.
Answer (C) is incorrect because movement of air masses is a result of unequal solar heating of the Earth' s
surface.
[237]
What feature is associated with a temperature inversion?
A. A stable layer of air.
B. An unstable layer of air.
C. Air mass thunderstorms.
Answer (A) is correct. A temperature inversion is defined as an increase in temperature with height; i.e., the
normal lapse rate is inverted. Thus, any warm air rises to its own temperature and forms a stable layer of
air.
Answer (B) is incorrect because instability occurs when the temperature decreases, not increases as in a
temperature inversion, with an increase in altitude and the rising air continues to rise.
Answer (C) is incorrect because air mass thunderstorms result from instability. They do not occur when
there is a temperature inversion.
[238]
What characterizes a ground-based inversion?
A. Convection currents at the surface.
B. Cold temperatures.
C. Poor visibility.
Answer (A) is incorrect because convective currents at the surface do not occur when there is a ground -
based inversion.
Answer (B) is incorrect because, when the temperature is cold, it is difficult for the earth to radiate enough
heat to become colder than the overlying air.
Answer (C) is correct. A ground-based inversion is characterized by poor visibility as a result of trapping fog,
smoke, and other restrictions into low levels of the atmosphere.
[239]
What condition produces the most frequent type of ground- or surface-based temperature inversion?
A. The movement of colder air under warm air or the movement of warm air over cold air.
B. Widespread sinking of air within a thick layer aloft resulting in heating by compression.
C. Terrestrial radiation on a clear, relatively calm night.
Answer (A) is incorrect because the movement of colder air under warm air is what happens when a cold
front is advancing, and the movement of warm air over cold air is the process of an advancing warm front.
Answer (B) is incorrect because widespread sinking of air describes compressional or adiabatic heating.
Answer (C) is correct. An inversion often develops near the ground on clear, cool nights when the wind is
relatively calm. The ground radiates heat and cools much faster than the overlying air. Air in contact with the
ground becomes cold, while the temperature a few hundred feet above changes very little.
[240]
What term describes an elongated area of low pressure?
A. Trough.
B. Ridge.
C. Hurricane or typhoon.
Answer (A) is correct. A trough is defined as an elongated area of low pressure with the lowest pressure
along a line marking maximum cyclonic curvature.
Answer (B) is incorrect because a ridge is an elongated area of high, not low, pressure.
Answer (C) is incorrect because a hurricane or typhoon is a tropical cyclone (low) with highest sustained
winds of 65 kt. or greater.
[241]
En route at FL 270, the altimeter is set correctly. On descent, a pilot fails to set the local altimeter setting of 30.57 in.
If the field elevation is 650 feet, and the altimeter is functioning properly, what will it indicate upon landing?
A. 585 feet.
B. 1, 300 feet.
C. Sea level.
Answer (A) is incorrect because 585 ft. is obtained by subtracting 65 ft. rather than subtracting 650 ft.
Answer (B) is incorrect because 1, 300 ft. is obtained by adding 650 ft. rather than subtracting 650 ft.
Answer (C) is correct. If an altimeter is set to 29.92 for flight at FL 270 and is not adjusted to the correct
altimeter setting of 30.57 during descent, the altimeter will indicate an altitude lower than actual by 650 ft.
(1,000 ft. x the difference between the two altimeter settings, which is .65). Thus, if the airplane lands at an
airport with a field elevation of 650 ft., the altimeter will indicate sea level (650 ft. airport elevation - 650 ft.
altimeter setting error).
[242]
If the ambient temperature is colder than standard at FL 310, what is the relationship between true altitude and
pressure altitude?
A. They are both the same, 31, 000 feet.
B. True altitude is lower than 31, 000 feet.
C. Pressure altitude is lower than true altitude.
Answer (A) is incorrect because both true and pressure altitudes are the same at FL 310 if the ambient air
temperature is standard, not colder than standard.
Answer (B) is correct. At FL 310, the altimeter is set to pressure altitude (29.92). If the outside air
temperature (ambient) is colder than standard, the air is compressed and heavier in weight per unit volume
than on a warm day. Thus, in colder-than-standard air, the true altitude is lower than the pressure altitude.
Answer (C) is incorrect because pressure altitude is lower than true altitude in warmer-, not colder-, than-
standard air temperature.
[243]
If the ambient temperature is warmer than standard at FL 350, what is the density altitude compared to pressure
altitude?
A. Lower than pressure altitude.
B. Higher than pressure altitude.
C. Impossible to determine without information on possible inversion layers at lower altitudes.
Answer (A) is incorrect because density altitude is higher, not lower, when air temperature is warmer than
standard.
Answer (B) is correct. Density altitude, by definition, is pressure altitude adjusted for nonstandard
temperature. When the ambient air temperature is warmer than standard, the expanded air is lighter in
weight per unit volume than on a cold day, and the pressure levels are raised. Thus, the pressure level
where the altimeter indicates FL 350 is higher on a warm day than under standard conditions.
Answer (C) is incorrect because density altitude is pressure altitude corrected for nonstandard
temperatures. Pressure altitude is based on a standard pressure atmosphere at a particular altitude, and
inversion layers at lower levels have no effect on pressure altitude.
[244]
Which pressure is defined as station pressure?
A. Altimeter setting.
B. Actual pressure at field elevation.
C. Station barometric pressure reduced to sea level.
Answer (A) is incorrect because alti meter setting is the value to which the scale of a pressure altimeter is
adjusted to read field elevation.
Answer (B) is correct. Pressure can be measured only at the point of measurement. The pressure
measured at a station or airport is "station pressure," or the actual pressure at field elevation.
Answer (C) is incorrect because station barometric pressure reduced to sea level is a method to readily
compare station pressures between stations at different altitudes.
[245]
Which weather condition is defined as an anticyclone?
A. Calm.
B. High pressure area.
C. COL.
Answer (A) is incorrect because calm is defined as the absence of wind or of apparent motion of the air.
Answer (B) is correct. An anticyclone is an area of high atmospheric pressure. The circulation is clockwise
in the Northern Hemi sphere, counterclockwise in the Southern Hemisphere, and undefined at the Equator.
Answer (C) is incorrect because COL is the neutral area between two highs or two lows. It is also the
intersection of a trough and a ridge.
[246]
What is a feature of air movement in a high pressure area?
A. Ascending from the surface high to lower pressure at higher altitudes.
B. Descending to the surface and then outward.
C. Moving outward from the high at high altitudes and into the high at the surface.
Answer (A) is incorrect because air descends, not ascends, from a high-pressure area, at all altitudes.
Answer (B) is correct. The air movement out of a high-pressure area depletes the quantity of air because
the air is heavier than the low-pressure air surrounding it. Therefore, high-pressure areas are areas of
descending air toward the surface, then outward from the high.
Answer (C) is incorrect because the air always moves outward, not inward, from a high, regardless of
altitude.
[247]
At which location does Coriolis force have the least effect on wind direction?
A. At the poles.
B. Middle latitudes (30° to 60°).
C. At the Equator.
Answer (A) is incorrect because the Coriolis force has the most (not least) effect on wind direction at the
poles.
Answer (B) is incorrect because the effects of the Coriolis force are quite pronounced in middle latitudes.
Answer (C) is correct. The Coriolis force affects the wind direction everywhere except immediately at the
Equator.
[248]
How does Coriolis force affect wind direction in the Southern Hemisphere?
A. Causes clockwise rotation around a low.
B. Causes wind to flow out of a low toward a high.
C. Has exactly the same effect as in the Northern Hemisphere.
Answer (A) is correct. Coriolis force deflects air to the left in the Southern Hemisphere, which is the opposite
from the Northern Hemisphere. In the Southern Hemisphere, the wind blows clockwise around a low.
Answer (B) is incorrect because the wind flows from a high to a low, not a low to a high, in both the Northern
and Southern Hemispheres.
Answer (C) is incorrect because the Coriolis force deflects air to the left in the Southern Hemisphere, which
is the opposite, not the same, effect as in the Northern Hemi sphere.
[249]
What is an important characteristic of wind shear?
A. It is primarily associated with the lateral vortices generated by thunderstorms.
B. It usually exists only in the vicinity of thunderstorms, but may be found near a strong temperature inversion.
C. It may be associated with either a wind shift or a windspeed gradient at any level in the atmosphere.
Answer (A) is incorrect because wind shear can be vertical (as well as lateral) in thunderstorm clouds
between the updrafts and downdrafts, as well as in other areas such as frontal zones and low-level
temperature inversions.
Answer (B) is incorrect because wind shear can be encountered in areas other than thunderstorms, e.g.,
within a frontal zone, in and near the jet stream, in low-level inversions.
Answer (C) is correct. Wind shear is the rate of change of wind velocity. The differences may be in wind
speed, wind direction, or both. An important characteristic is that wind shear may be associated with either
a wind shift or a wind speed gradient at any level in the atmosphere.
[250]
What condition is indicated when ice pellets are encountered during flight?
A. Thunderstorms at higher levels.
B. Freezing rain at higher levels.
C. Snow at higher levels.
Answer (A) is incorrect because ice pellets always indicate freezing rain, not thunderstorms, at higher
altitudes.
Answer (B) is correct. Ice pellets always indicate freezing rain at higher altitudes. Ice pellets are formed
when rain freezes during descent.
Answer (C) is incorrect because freezing rain, not snow, is indicated at higher altitudes when ice pellets are
encountered.
[251]
Which term applies when the temperature of the air changes by compression or expansion with no heat added or
removed?
A. Katabatic.
B. Advection.
C. Adiabatic.
Answer (A) is incorrect because katabatic is a wind blowing down an incline caused by cold, heavier air
spilling down the incline displacing warmer, less dense air.
Answer (B) is incorrect because advection is the horizontal flow in a convective current, i.e., wind.
Answer (C) is correct. When air expands, it cools; when compressed, it warms. These changes are
adiabatic, meaning that no heat is removed from or added to the air.
[252]
Which process causes adiabatic cooling?
A. Expansion of air as it rises.
B. Movement of air over a colder surface.
C. Release of latent heat during the vaporization process.
Answer (A) is correct. Adiabatic cooling is caused by the expansion of air as it rises. Anytime air is moved
upward, it expands because of decreasing atmospheric pressure. When air expands, it cools.
Answer (B) is incorrect because adiabatic cooling means that no heat is removed from the air, as would be
the case if the air were moved over a colder surface.
Answer (C) is incorrect because adiabatic is the process in which no heat is removed from or added to the
air.
[253]
What is indicated about an air mass if the temperature remains unchanged or decreases slightly as altitude is
increased?
A. The air is unstable.
B. A temperature inversion exists.
C. The air is stable.
Answer (A) is incorrect because unstable air would have a uniform decrease (approaching 3°C /1, 000 ft.) in
temperature with an increase in altitude.
Answer (B) is incorrect because in a temperature inversion the temperature increases with increases in
altitude.
Answer (C) is correct. If the air temperature remains unchanged or decreases only slightly as altitude is
increased, the air mass tends to be stable.
[254]
What is a feature of a stationary front?
A. The warm front surface moves about half the speed of the cold front surface.
B. Weather conditions are a combination of strong cold front and strong warm front weather.
C. Surface winds tend to flow parallel to the frontal zone.
Answer (A) is incorrect because the movement of a warm front surface at about half the speed of the cold
front surface is a feature of the movement of a warm, not stationary, front when the general wind flow is the
same in each case.
Answer (B) is incorrect because weather conditions that are a combination of strong cold front and strong
warm front weather are a feature of an occluded, not stationary, front.
Answer (C) is correct. The opposing forces exerted by adjacent air masses of different densities are such
that the frontal surface between them shows little or no movement and a stationary front results. The
surface winds tend to flow parallel to this frontal zone.
[255]
Which event usually occurs after an aircraft passes through a front into the colder air?
A. Temperature/dewpoint spread decreases.
B. Wind direction shifts to the left.
C. Atmospheric pressure increases.
Answer (A) is incorrect because the temperature/dew point spread usually differs across a front. But it may
not decrease if you fly into a cold, dry air mass.
Answer (B) is incorrect because, in the Northern Hemisphere, the wind always shifts to the right, not left,
due to the Coriolis force.
Answer (C) is correct. A front lies in a pressure trough, and pressure generally is higher in cold air. Thus,
when you cross a front directly into colder air, barometric pressure usually increases.
[256]
In which meteorological conditions can frontal waves and low pressure areas form?
A. Warm fronts or occluded fronts.
B. Slow-moving cold fronts or stationary fronts.
C. Cold front occlusions.
Answer (A) is incorrect because occluded fronts are formed by frontal waves and areas of low pressure
which cause a cold front to close together with a warm front. Frontal waves and low-pressure areas
normally form on slow-moving cold, not warm, fronts.
Answer (B) is correct. Frontal waves and areas of low pressure usually form on slow-moving cold fronts or
on stationary fronts.
Answer (C) is incorrect because a cold front occlusion occurs when the air behind the cold front is colder
than the air in advance of the warm front, lifting the warm front aloft.
[257]
What is indicated by the term "embedded thunderstorms"?
A. Severe thunderstorms are embedded in a squall line.
B. Thunderstorms are predicted to develop in a stable air mass.
C. Thunderstorms are obscured by other types of clouds.
Answer (A) is incorrect because a squall line consists of severe thunderstorms which can always be seen.
Answer (B) is incorrect because thunderstorms do not occur in stable air masses.
Answer (C) is correct. The term "embedded thunderstorms " means that the thunderstorms are embedded
in clouds or thick haze layers and cannot be seen.
[258]
What feature is normally associated with the cumulus stage of a thunderstorm?
A. Beginning of rain at the surface.
B. Frequent lightning.
C. Continuous updraft.
Answer (A) is incorrect because the beginning of rain at a surface marks the beginning of the mature stage.
Answer (B) is incorrect because frequent lightning occurs after the downdrafts have developed, producing
the static electricity which causes lightning.
Answer (C) is correct. During the cumulus stage of a thunderstorm, the cumulus cloud is building, and there
are continuous updrafts.
[259]
Which weather phenomenon signals the beginning of the mature stage of a thunderstorm?
A. The appearance of an anvil top.
B. The start of rain at the surface.
C. Growth rate of the cloud is at its maximum.
Answer (A) is incorrect because the appearance of an anvil top occurs during the dissipating stage.
Answer (B) is correct. The mature stage of a thunderstorm begins when rain begins falling at the surface.
This means that downdrafts have developed sufficiently to carry water all the way through the thunderstorm.
Answer (C) is incorrect because the growth rate of a thunderstorm is at its greatest during the cumulus
stage.
[260]
Why are downdrafts in a mature thunderstorm hazardous?
A. Downdrafts are kept cool by cold rain which tends to accelerate the downward velocity.
B. Downdrafts converge toward a central location under the storm after striking the surface.
C. Downdrafts become warmer than the surrounding air and reverse into an updraft before reaching the surface.
Answer (A) is correct. During the mature stage, cold rain in the downdraft slows compressional heating, and
the downdraft remains colder than the surrounding air. Thus, its downward s peed is accelerated and may
exceed 2, 500 fpm.
Answer (B) is incorrect because, after striking the ground, the downdrafts will move away from the storm's
center (not converge).
Answer (C) is incorrect because downdrafts remain colder, not warmer, than the surrounding air and
accelerate downward, not reverse into an updraft.
[261]
During the life cycle of a thunderstorm, which stage is characterized predominately by downdrafts?
A. Cumulus.
B. Dissipating.
C. Mature.
Answer (A) is incorrect because the cumulus stage is the building stage characterized by updrafts.
Answer (B) is correct. Thunderstorms have three life cycles: cumulus, mature, and dissipating. In the
dissipating stage, the storm is characterized by downdrafts as the storm rains itself out.
Answer (C) is incorrect because the mature stage has both updrafts and downdrafts, which create strong
wind shears.
[262]
Where do squall lines most often develop?
A. In an occluded front.
B. Ahead of a cold front.
C. Behind a stationary front.
Answer (A) is incorrect because squall lines most often develop ahead of a cold front, not in an occluded
front.
Answer (B) is correct. A squall line is a non frontal, narrow band of active thunderstorms. Most often squall
lines develop ahead of a cold front in moist, unstable air.
Answer (C) is incorrect because squall lines most often develop ahead of a cold front, not behind a
stationary front.
[263]
Which feature is associated with the tropopause?
A. Absence of wind and turbulence.
B. Absolute upper limit of cloud formation.
C. Abrupt change of temperature lapse rate.
Answer (A) is incorrect because the jet stream (wind) and clear air turbulence are found extensively in the
tropopause.
Answer (B) is incorrect because clouds can be present int o the stratosphere, as in very large
thunderstorms and cirrus clouds made up of ice crystals.
Answer (C) is correct. The tropopause is characterized by an abrupt change in temperature lapse rate. The
temperature above the tropical tropopause increases with height, and the temperature above the polar
tropopause remains about constant with height.
[264]
What weather feature occurs at altitude levels near the tropopause?
A. Maximum winds and narrow wind shear zones.
B. Abrupt temperature increase above the tropopause.
C. Thin layers of cirrus (ice crystal) clouds at the tropopause level.
Answer (A) is correct. Temperature and wind vary greatly in the vicinity of the tropopause. Maximum winds
generally occur near the tropopause. These winds create narrow zones of wind shear which often generate
hazardous turbulence.
Answer (B) is incorrect because abrupt temperature increase does not occur above the tropopause.
Answer (C) is incorrect because thin layers of cirrus (ice crystal) clouds can develop at altitudes below the
tropopause level and extend into the lower stratosphere.
[265]
What weather is predicted by the term VCTS in a Terminal Aerodrome Forecast?
A. Thunderstorms are expected in the vicinity.
B. Thunderstorms may occur over the station and within 50 miles of the station.
C. Thunderstorms are expected between 5 and 25 miles of the runway complex.
Answer (A) is correct. The term VC (vicinity) applies to weather conditions expected within a 5-to-10 SM
radius of the airport, but not at the airport itself. Thus, VCTS in a TAF means thunderstorms are expected in
the vicinity of the airport.
Answer (B) is incorrect because VC (vicinity) means within 5-to-10 SM from the airport, not 50 miles.
Answer (C) is incorrect.
[266]
Which are the only cloud types forecast in the Terminal Aerodrome Forecast?
A. Altocumulus
B. Cumulonimbus
C. Stratocumulus
Answer (A) is incorrect because the only cloud type forecast in the TAF is cumulonimbus, not altocumulus.
Answer (B) is correct. If cumulonimbus clouds are expected at the airport, the contraction CB is appended
to the cloud layer which represents the base of the cumulonimbus cloud(s). Cumulonimbus clouds are the
only cloud type forecast in the T AF.
Answer (C) is incorrect because the only cloud type forecast in the TAF is cumulonimbus, not
stratocumulus.
[267]
A calm wind that is forecast, in the International Terminal Aerodrome Forecast (TAF), is encoded as
A. VRB00 KT.
B. 00000 KT.
C. 00003 KT.
Answer (A) is incorrect because a variable wind is encoded as VRB when a forecast is for low wind speeds
(3 kt. or less) followed by a wind speed, not two zeros. A calm wind will be encoded as 00000 KT.
Answer (B) is correct. In the International Terminal Aerodrome Forecast (TAF), a calm wind that is forecast
is encoded as 00000KT.
Answer (C) is incorrect because a calm wind is encoded as 00000KT, not 00003 KT.
[268]
What type navigation system is Inertial Navigation System (INS)? A navigation computer which provides position
A. from information by compass, airspeed, and an input of wind and variation data.
B. from radar-type sensors that measure ground speed and drift angles.
C. by signals from self-contained gyros and accelerometers.
Answer (A) is incorrect because a Doppler radar uses the airplane's compass as a directional reference. It
uses radar to detect and measure groundspeed, not airspeed, and drift angles.
Answer (B) is incorrect because a Doppler radar, not INS, provides position information from radar-type
sensors that measure groundspeed and drift angles.
Answer (C) is correct. Inertial Navigation System (INS) is a totally self-contained system comprised of
gyros, accelerometers, and a navigation computer, which provides airplane position and navigation
information in response to signals resulting from inertial effects on system components, and does not
require information from external sources. INS may be approved as the sole means of navigation or may be
used in combination with other systems.
[269]
What functions are provided by ILS?
A. Azimuth, distance, and vertical angle.
B. Azimuth, range, and vertical angle.
C. Guidance, range, and visual information.
Answer (A) is incorrect because azimuth and distance information are provided by a TACAN.
Answer (B) is incorrect because a localizer/DME approach provides azimuth and range information.
Answer (C) is correct. The ILS system may be divided functionally into three parts:
1. Guidance information--localizer, glide slope
2. Range information--marker beacon, DME
3. Visual information--approach lights, runway lights
[270]
Identify touchdown zone lighting (TDZL).
A. Two rows of transverse light bars disposed symmetrically about the runway centerline.
B. Flush centerline lights spaced at 50-foot intervals extending through the touchdown zone.
C. Alternate white and green centerline lights extending from 75 feet from the threshold through the touchdown
zone.
Answer (A) is correct. Touchdown zone lighting (TDZL) consists of two rows of transverse light bars
disposed symmetrically about the runway centerline in the runway touchdown zone. The system starts 100
ft. from the landing threshold and extends to 3,000 ft. from the threshold or the midpoint of the runway,
whichever is less.
Answer (B) is incorrect because flush centerline lights spaced at 50-ft. intervals extending the length of the
runway, including the touchdown zone, are runway centerline lighting, not TDZL.
Answer (C) is incorrect because runway centerline lights, not TDZL, extend from 75 ft. from the threshold
through the touchdown zone and are white, not alternating white and green.
[271]
What is the purpose of REIL?
A. Identification of a runway surrounded by a preponderance of other lighting.
B. Identification of the touchdown zone to prevent landing short.
C. Establish visual descent guidance information during an approach.
Answer (A) is correct. Runway end identifier lights (REIL) are installed to provide rapid and positive
identification of the approach end of a particular runway. REIL is effective for the identification of a runway
surrounded by a preponderance of other lighting.
Answer (B) is incorrect because the touchdown zone is identified by in-runway lighting of two rows of
transverse light bars on either side of the runway centerline from 100 ft. to 3, 000 ft. from the landing
threshold.
Answer (C) is incorrect because a VASI, not REIL, assists in providing visual descent guidance information
during an approach.
[272]
Identify REIL.
A. Amber lights for the first 2,000 feet of runway.
B. Green lights at the threshold and red lights at far end of runway.
C. Synchronized flashing lights laterally at each side of the runway thresholds
Answer (A) is incorrect because amber lights are used on the last, not first, 2,000 ft. of runway edge lights to
form a caution zone on instrument runways.
Answer (B) is incorrect because green lights at the threshold mark the runway edge for landing aircraft, and
red lights at the far end mark the runway edge to a departing or landing aircraft.
Answer (C) is correct. Runway end identifier lights (REIL) are installed to provide rapid and positive
identification of the approach end of a particular runway. The system consists of a pair of synchronized
flashing lights located laterally on each side of the runway threshold.
[273]
The higher glide slope of the three-bar VASI is intended for use by
A. high performance aircraft.
B. helicopters.
C. high cockpit aircraft.
Answer (A) is incorrect because the higher glide slope of a three-bar VASI is for use only by high-cockpit
aircraft, which may or may not be high-performance aircraft.
Answer (B) is incorrect because the higher glide slope of a three-bar VASI is for use only by high-cockpit
aircraft, not specifically for use by helicopters.
Answer (C) is correct. The three-bar VASI provides two visual glide paths. The higher glide path is int ended
for use only by high-cockpit aircraft to provide a sufficient threshold crossing height.
[274]
What does the tri-color VASI consist of?
A. Three light bars; red, green, and amber.
B. One light projector with three colors; red, green, and amber.
C. Three glide slopes, each a different color; red, green, and amber.
Answer (A) is incorrect because a tri-color VASI consists of a single light projector, not three light bars.
Answer (B) is correct. Tri-color VASI is normally consist of a single light unit projecting a three-color visual
approach path. The three colors are red, green, and amber. Below the glide path is red, above the glide
path is amber, and on the glide path is green.
Answer (C) is incorrect because a tri-color VASI projects only one glide slope, not three. Below the glide
slope is red, above the glide slope is amber, and on the glide slope is green.
[275]
Which color on a tri-color VASI is a "high" indication?
A. Red.
B. Amber.
C. Green.
Answer (A) is incorrect because a red color on a tri-color VASI indicates below, not above, the glide path.
Answer (B) is correct. The tri-color VASI projects a three-color visual approach path into the final approach
area of the runway. A "high" indication is seen as the color amber.
Answer (C) is incorrect because a green color on a tri-color VASI indicates on, not above, the glide path.
[276]
Which color on a tri-color VASI is an "on course" indication?
A. Red.
B. Amber.
C. Green.
Answer (A) is incorrect because a red color on a tri-color VASI indicates below, not on, the glide path.
Answer (B) is incorrect because an amber color on a tri-color VASI indicates above, not on, the glide path.
Answer (C) is correct. The tri-color VASI projects a three-color visual approach path into the final approach
area of the runway. An "on course," or on glide path, indication is seen as the color green.
[277]
Which color on a tri-color VASI is a "low" indication?
A. Red.
B. Amber.
C. Green.
Answer (A) is correct. The tri-color VASI projects a three-color visual approach path into the final approach
area of the runway. A "low" indication is seen as the color red.
Answer (B) is incorrect because an amber color on a tri-color VASI is an indication of above, not below, the
glide path.
Answer (C) is incorrect because a green color on a tri-color VASI is an indication of on, not below, the glide
path.
[278]
What is the normal range of the tri-color VASI at night?
A. 5 miles.
B. 10 miles.
C. 15 miles.
Answer (A) is correct. Tri-color VASI has a useful range of approximately ½ to 1 mi. during the day and up
to 5 mi. at night depending on the visibility conditions.
Answer (B) is incorrect because a tri-color VASI has a normal range of 5 mi., not 10 mi., at night.
Answer (C) is incorrect because a tri-color VASI has a normal range of 5 mi., not 15 mi., at night.
[279]
What does the pulsating VASI consist of?
A. Three-light system, two pulsing and one steady.
B. Two-light projectors, one pulsing and one steady.
C. One-light projector, pulsing white when above glide slope or red when more than slightly below glide slope,
steady white when on glide slope, steady red for slightly below glide path.
Answer (A) is incorrect because the pulsating VASI is a two-, not three-, light system, with two pulsating and
two, not one, steady.
Answer (B) is incorrect because the pulsating VASI is a single, not double, light projecting unit.
Answer (C) is correct. Pulsating VASI is normally consist of a single light unit projecting a two-color visual
glide path indication in the final approach area of the runway. The below glide path indication is normally
pulsating red, and the above glide path is pulsating white. The on glide path indication is a steady white
light. The slightly below glide path is a steady red. The answer should refer to "pulsating, " not "pulsing."
[280]
What are the indications of the pulsating VASI?
A. High - pulsing white, on glidepath - green, low - pulsi ng red.
B. High - pulsing whit e, on glidepath - steady white, slightly below glide slope - steady red, low - pulsing red.
C. High - pulsing white, on course and on glidepath - steady white, off course but on glidepath - pulsing white and
red; low - pulsing red.
Answer (A) is incorrect because the on glide path indication of a pulsating VASI is either a pulsating red and
white or steady white, not green, light.
Answer (B) is correct. The pulsating VASI provides a two-color visual glide path indication in the final
approach area. It provides only a glide path indication while lateral course guidance is provided by the
runway or runway lights. Above glide path is pulsating white. On glide path is steady white. Slightly below
glide path is steady red. Below glide path is pulsating red. Also, the answer should refer to "pulsating, " not
"pulsing."
Answer (C) is incorrect because the pulsating VASI provides only glide path indications, not lateral, or
course, indications.
[281]
What does the Precision Approach Path Indicator (PAPI) consist of?
A. Row of four lights parallel to the runway; red, white, and green.
B. Row of four lights perpendicular to the runway; red and white.
C. One light projector with two colors; red and white.
Answer (A) is incorrect because PAPI has a row of four lights perpendicular, not parallel, to the runway and
projects red and white, not green, light.
Answer (B) is correct. The PAPI uses lights similar to the two- and three-bar VASI but installed in a single
row of either two or four light units perpendicular to the runway. The row of lights is normally installed on the
left side of the runway. Each light unit appears as either red or white. When two are red and two are white,
you are on the glide path.
Answer (C) is incorrect because PAPI consists of a row of four, not one, light projectors emitting red or
white light.
[282]
What are the indications of Precision Approach Path Indicator (PAPI)?
A. High - white, on glidepath - red and white; low - red.
B. High - white, on glidepath - green; low - red.
C. High - white and green, on glidepath - green; low - red.
Answer (A) is correct. PAPI can indicate five possible glide path indications. All white light indicates "high"
or more than 3.5° glide path. One red and three white indicate a slightly high (3.2°) glide path. Two red and
two white indicate on the glide path (3°). Three red and one white indicate a slightly low (2.8°) glide path.
Four red lights indicate below (less than 2.5°) the glide path.
Answer (B) is incorrect because the on glide path indication of PAPI is both red and white, not green, lights.
Answer (C) is incorrect because above the glide path indication of PAPI is all white (green is not a color
indication of PAPI), on glide path is two red and two white, not green, and below glide path is all red.
[283]
When instructed by ATC to "Hold short of a runway (ILS critical area, etc.)," the pilot should stop
A. with the nose gear on the hold line.
B. so that no part of the aircraft extends beyond the hold line.
C. so the flight deck area of the aircraft is even with the hold line.
Answer (A) is incorrect because you should stop before, not with the nose gear on, the hold line.
Answer (B) is correct. When instructed by ATC to hold short of a runway (ILS critical area, etc.), you should
stop so no part of your aircraft extends beyond the holding position marking.
Answer (C) is incorrect because you should stop before, not so the flight deck is even with, the hold line.
[284]
Which range of codes should a pilot avoid switching through when changing transponder codes?
A. 0000 through 1000.
B. 7200 and 7 500 series.
C. 7500, 7600, and 7 700 series.
Answer (A) is incorrect because codes 0000 through 1000 are acceptable codes that may be used during
normal operations.
Answer (B) is incorrect because the 7200 series can be used during normal operations, while 7500 should
be avoided except in the case of a hijack.
Answer (C) is correct. When making routine code changes, pilots should avoid inadvertent selection of
codes
7500, 7600, or 7700, which can cause momentary false alarms at automated ground facilities. This applies
to non-discrete code 7500 and all discrete codes in the 7600 and 7700 series. Remember that 7500 is for
hijack, 7600 for lost radio communications, 7700 for general emergency.
[285]
What is the hijack code?
A. 7200.
B. 7500.
C. 7777.
Answer (A) is incorrect because 7200 is a code that is used for normal operating procedures.
Answer (B) is correct. Transponder code 7500 means: "I am being hijacked/forced to a new destination."
Code 7500 will never be assigned by ATC without prior notification from the pilot that his /her airplane is
being subjected to unlawful interference. Code 75 00 will trigger special emergency indicators in all radar
ATC facilities.
Answer (C) is incorrect because under no circumstances should a pilot of a civil airplane operate the
transponder on Code 7777. This code is reserved for military interceptor operations.
[286]
Under what condition does ATC issue safety alerts?
A. When collision with another aircraft is imminent.
B. If the aircraft altitude is noted to be in close proximity to the surface or an obstacle.
C. When weather conditions are extreme and wind shear or large hail is in the vicinity.
Answer (A) is incorrect because a safety alert is issued to a pilot if ATC believes that his/her airplane is at
an altitude which will place it in unsafe proximity to another airplane, not that a collision is imminent.
Answer (B) is correct. ATC will issue a safety alert when an airplane under its control is at an altitude which,
in ATC' s judgment, places the aircraft in unsafe proximity to terrain, obstructions, or another aircraft. A
terrain or obstruction alert is issued to an airplane that is at an altitude believed to place the aircraft in
unsafe proximity to terrain or obstructions. An aircraft conflict alert is issued when an airplane under a
controller's direction is at an altitude believed to place the aircraft in unsafe proximity to another airplane not
under that controller's direction.
Answer (C) is incorrect because, when weather conditions are extreme and wind shear or large hail is in the
vicinity, a convective SIGMET, not a safety alert, is broadcast.
[289]
What airport condition is reported by the tower when more than one wind condition at different positions on the
airport is reported?
A. Light and variable.
B. Wind shear.
C. Frontal passage.
Answer (A) is incorrect because light and variable is used to report the wind conditions when wind speed is
less than 5 kt.
Answer (B) is correct. When the tower reports more than one wind condition at different positions on the
airport, it means that wind shear is probable. Various sensors of the Low-Level Wind Shear Alert System
measure the wind, and when differences become excessive, the tower will provide more than one wind
condition at different positions
Answer (C) is incorrect because a frontal passage is normally indicated by a change in wind direction, but it
is usually not reported by the tower.
[290]
TCAS I provides
A. traffic and resolution advisories.
B. proximity warning.
C. recommended maneuvers to avoid conflicting traffic.
Answer (A) is incorrect because TCAS II, not TCAS I, provides traffic and resolution advisories. TCAS I
provides only a proximity warning.
Answer (B) is correct. TCAS I provides only a proximity warning to assist the pilot in the visual acquisition of
intruder aircraft. No recommended avoidance maneuvers are provided or authorized as a direct result of a
TCAS I warning.
Answer (C) is incorrect because TCAS II, not TCAS I, provides recommended maneuvers to avoid
conflicting traffic and resolution advisories. TCAS I provides only a proximity warning.
[291]
TCAS II provides
A. traffic and resolution advisories.
B. proximity warning.
C. maneuvers in all directions to avoid the conflicting traffic.
Answer (A) is correct. TCAS II provides traffic and resolution advisories. Resolution advisories provide
recommended maneuvers in a vertical direction (climb or descend only) to avoid conflicting traffic.
Answer (B) is incorrect because TCAS I, not TCAS II, provides only a proximity warning. TCAS II provides
traffic and resolution advisories.
Answer (C) is incorrect because TCAS II provides recommended maneuvers only in a vertical direction, not
in all directions, to avoid conflicting traffic.
[292]
Except during an emergency, when can a pilot expect landing priority?
A. When cleared for an IFR approach.
B. When piloting a large, heavy aircraft.
C. In turn, on a first-come, first-serve basis.
Answer (A) is incorrect because a clearance for an IFR approach does not mean landing priority will be
given over other traffic.
Answer (B) is incorrect because a large, heavy aircraft will be sequenced for landing on a first-come, first-
served basis, with no special priority over other traffic.
Answer (C) is correct. A clearance for a specific type of approach to an airplane operating on an IFR flight
plan does not mean that landing priority will be given over other traffic. Air traffic control towers handle all
airplanes, regardless of the type of flight plan, on a first-come, first-served basis. A landing sequence will be
issued to each airplane as soon as possible to enable the pilot to adjust his /her flight path properly.
[293]
Under what condition should a pilot on IFR advise ATC of minimum fuel status?
A. When the fuel supply becomes less than that required for IFR.
B. If the remaining fuel suggests a need for traffic or landing priority.
C. If the remaining fuel precludes any undue delay.
Answer (A) is incorrect because a pilot must ensure the minimum amount of fuel is on board the airplane for
the planned IFR flight and alternatives, if needed, during the flight planning phase.
Answer (B) is incorrect because, if the remaining fuel suggests a need for traffic or landing priority, the pilot
should declare an emergency, not minimum fuel status, due to low fuel and report fuel remaining in
minutes.
Answer (C) is correct. A pilot should advise ATC of minimum fuel status when the remaining fuel supply has
reached a state which precludes any undue delay upon reaching destination. A pilot must be aware that this
is not an emergency situation but an advisory that indicates an emergency situation is possible should any
undue delay occur.
[294]
What does the term "minimum fuel" imply to ATC?
A. Traffic priority is needed to the destination airport.
B. Emergency handling is required to the nearest suitable airport.
C. Advisory that indicates an emergency situation is possible should an undue delay occur.
Answer (A) is incorrect because a pilot should declare an emergency due to low fuel and report fuel
remaining in minutes if traffic priority is needed to the destination airport.
Answer (B) is incorrect because emergency handling to the nearest suitable airport is required in a distress
or urgent type of emergency.
Answer (C) is correct. When a pilot advises ATC of a minimum fuel status, ATC will handle this report as an
advisory that indicates an emergency situation is possible should an undue delay occur. A minimum fuel
advisory does not imply a need for traffic priority.
[295]
What minimum condition is suggested for declaring an emergency?
A. Anytime the pilot is doubtful of a condition that could adversely affect flight safety.
B. When fuel endurance or weather will require an en route or landing priority.
C. When distress conditions such as fire, mechanical failure, or structural damage occurs.
Answer (A) is correct. An emergency is defined as either a distress or an urgency condition. A distress
condition can be a fire, mechanical failure, or structural damage. An airplane is in at least an urgency
condition when the pilot becomes doubtful about position, fuel endurance, weather, or any other condition
that could adversely affect flight safety. A pilot should declare an emergency under any of these
circumstances, not after the situation has developed into a distress condition.
Answer (B) is incorrect because, if fuel endurance or weather will require an en route or landing priority, the
situation has progressed beyond an urgency condition and is now a distress condition.
Answer (C) is incorrect because di stress condition implies a serious and/or imminent danger requiring
immediate assistance. This situation is the step after an urgency condition, which is one that has the
potential to become a distress condition.
[296]
Hypoxia is the result of which of these conditions?
A. Insufficient oxygen reaching the brain.
B. Excessive carbon dioxide in the bloodstream.
C. Limited oxygen reaching the heart muscles.
Answer (A) is correct. Hypoxia is a state of oxygen deficiency in the body sufficient to impair functions of the
brain and other organs.
Answer (B) is incorrect because insufficient, not excessive, carbon dioxide in the blood stream describes
hyperventilation.
Answer (C) is incorrect because it is insufficient, not limited, oxygen to the brain, not heart muscles.
[297]
What causes hypoxia?
A. Excessive carbon dioxide in the atmosphere.
B. An increase in nitrogen content of the air at high altitudes.
C. A decrease of oxygen partial pressure.
Answer (A) is incorrect because the percentage of carbon dioxide and oxygen in the atmosphere remains
constant with changes in altitude. There is just less pressure as you increase in altitude.
Answer (B) is incorrect because relative nitrogen content also remains constant at high altitudes. There is
just less pressure.
Answer (C) is correct. Hypoxia from exposure to altitude is due only to the reduced barometric pressures
encountered at altitude. With the decrease in barometric pressure, less oxygen is absorbable by the lungs.
[298]
Loss of cabin pressure may result in hypoxia because as cabin altitude increases
A. the percentage of nitrogen in the air is increased.
B. the percentage of oxygen in the air is decreased.
C. oxygen partial pressure is decreased.
Answer (A) is incorrect because the percentage of nitrogen, carbon dioxide, and oxygen in the atmosphere
remains constant with changes in altitude. There is just less pressure as you increase in altitude.
Answer (B) is incorrect because oxygen remains at about 21% in the atmosphere, i.e., from the ground out
to space. The amount of oxygen decreases, however, due to lower barometric pressure.
Answer (C) is correct. Hypoxia from exposure to altitude is due only to the reduced barometric pressures
encountered at altitude. With the decrease in barometric pressure, less oxygen is absorbable by the lungs.
[299]
What is the effect of alcohol consumption on functions of the body?
A. Alcohol has an adverse effect, especially as altitude increases.
B. Small amounts of alcohol in the human system increase judgment and decision-making abilities.
C. Alcohol has little effect if followed by equal quantities of black coffee.
Answer (A) is correct. Alcohol impairs flying skills. Even small amounts of alcohol render pilots more
susceptible to disorientation and hypoxia.
Answer (B) is incorrect because even small amounts of alcohol impair, not increase, judgment and
decision-making abilities.
Answer (C) is incorrect because there is no way, including drinking black coffee, of increasing the body's
metabolism of alcohol or of alleviating a hangover.
[300]
Which is a common symptom of hyperventilation?
A. Tingling of the hands, legs, and feet.
B. Increased vision keenness.
C. Decreased breathing rate.
Answer (A) is correct. Hyperventilation results from an abnormal increase in the volume of air breathed in
and out of the lungs. It can occur subconsciously when a stressful situation is encountered. The result is an
excessive amount of carbon dioxide removed from the body. The symptoms are lightheadedness,
suffocation, drowsiness, tingling of the extremities, and coolness.
Answer (B) is incorrect because hyperventilation distorts one's abilities, not increases them.
Answer (C) is incorrect because decreasing the breathing rate overcomes hyperventilation and is not a
symptom of it.
[301]
Which would most likely result in hyperventilation?
A. A stressful situation causing anxiety.
B. The excessive consumption of alcohol.
C. An extremely slow rate of breathing and insufficient oxygen.
Answer (A) is correct. Hyperventilation results from an abnormal increase in the volume of air breathed in
and out of the lungs. It can occur subconsciously when a stressful situation is encountered. The result is an
excessive amount of carbon dioxide removed from the body. The symptoms are lightheadedness,
suffocation, drowsiness, tingling of the extremities, and cool ness.
Answer (B) is incorrect because excessive consumption of alcohol would most likely result in hypoxia, not
hyperventilation.
Answer (C) is incorrect because a slow rate of breathing is the cure for hyperventilation, and insufficient
oxygen is the cause of hypoxia, not hyperventilation.
[302]
A pilot is more subject to spatial disorientation when
A. ignoring or overcoming the sensations of muscles and inner ear.
B. eyes are moved often in the process of cross-checking the flight instruments.
C. body sensations are used to interpret flight attitudes.
Answer (A) is incorrect because ignoring or overcoming the sensations of muscles and inner ear is a means
of avoiding (not becoming subject to) spatial disorientation.
Answer (B) is incorrect because rapid eye movements have little or no impact on spatial disorientation, and
visual reference to reliable flight instruments helps avoid spatial disorientation.
Answer (C) is correct. Spatial disorientation is a state of temporary confusion resulting from misleading
information being sent to the brain by various sensory organs. Using body sensations to interpret flight
attitudes makes pilots more susceptible to spatial dis orientation.
[303]
Which procedure is recommended to prevent or overcome spatial disorientation?
A. Reduce head and eye movement to the greatest possible extent.
B. Rely on the kinesthetic sense.
C. Rely entirely on the indications of the flight instruments.
Answer (A) is incorrect because head and eye movement has little effect on spatial disorientation.
Answer (B) is incorrect because relying on the kinesthetic sense encourages, rather than prevents, spatial
disorientation.
Answer (C) is correct. The best way to overcome the effects of spatial disorientation is to rely entirely on the
airplane's flight instruments and/or ground references (if available) and not on body sensations.
[304]
An ATC "instruction"
A. is the same as an ATC "clearance."
B. is a directive issued by ATC for the purpose of requiring a pilot to take a specific action providing the safety of the
aircraft is not jeopardized.
C. must be "read back" in full to the controller and confirmed before becoming effective.
Answer (A) is incorrect because an ATC clearance is an authorization by ATC for an aircraft to proceed
under specified traffic conditions within controlled airspace. An instruction is more specific, e.g., "Turn left
heading 250."
Answer (B) is correct. An ATC instruction is a directive issued by ATC for the purpose of requiring a pilot to
take a specific action, e.g., "Turn left, heading 250," "Go around," "Clear the runway," etc.
Answer (C) is incorrect because, although instructions should be (and some are required to be) read back
to the controller, they are effective immediately.
[305]
Which condition has the effect of reducing critical engine failure speed?
A. Slush on the runway or inoperative anti-skid.
B. Low gross weight.
C. High density altitude.
Answer (A) is correct. Slush on the runway may dictate adjustments, such as a reduction in critical engine
failure speed (V1), an increase in required runway length, or a penalty in runway limited weight. An
inoperative anti-skid system may have the same requirements according to the airplane flight manual or
company operations manual.
Answer (B) is incorrect because low gross weight may allow an increase in critical engine failure speed.
Answer (C) is incorrect because an increase in density altitude requires a higher takeoff velocity (TAS) and
critical engine failure speed.
[306]
During an en route descent in a fixed-thrust and fixed-pitch attitude configuration, both the ram air input and drain
hole of the pitot system become completely blocked by ice. What airspeed indication can be expected?
A. Increase in indicated airspeed.
B. Decrease in indicated airspeed.
C. Indicated airspeed remains at the value prior to icing.
Answer (A) is incorrect because indicated airspeed will decrease, not increase, in a descent.
Answer (B) is correct. If the ram air input and the drain hole are blocked, the pressure is trapped in the
system, and the airspeed indicator will react as an altimeter. During a descent, the airspeed indication will
decrease.
Answer (C) is incorrect because indicated airspeed will remain at the same value during level, not
descending, flight.
[307]
What can a pilot expect if the pitot system ram air input and drain hole are blocked by ice?
A. The airspeed indicator may act as an altimeter.
B. The airspeed indicator will show a decrease with an increase in altitude.
C. No airspeed indicator change will occur during climbs or descents.
Answer (A) is correct. If the ram air and the drain hole are blocked, the pressure is trapped in the system,
and the airspeed indicator will react as an altimeter: higher indicated airspeed at higher altitudes and lower
indicated airspeed at lower altitudes.
Answer (B) is incorrect because the airspeed indicator will show an increase, not decrease, with an
increase in altitude.
Answer (C) is incorrect because differential pressure between the pitot tube and static air source changes
and so does indicated airspeed.
[308]
If both the ram air input and drain hole of the pitot system are blocked by ice, what airspeed indication can be
expected?
A. No variation of indicated airspeed in level flight if large power changes are made.
B. Decrease of indicated airspeed du ring a climb.
C. Constant indicated airspeed during a descent.
Answer (A) is correct. If both the pitot tube input and the drain hole on the pitot system are blocked, the
airspeed indication will be const ant at any given altitude.
Answer (B) is incorrect because, during a climb, it will indicate an increase, not decrease, due to the
stronger differential pressure in the blocked pitot tube relative to the static vents.
Answer (C) is incorrect because indicated airspeed will change with changes in altitude.
[309]
How will the airspeed indicator react if the ram air input to the pitot head is blocked by ice, but the drain hole and
static port are not?
A. Indication will drop to zero.
B. Indication will rise to the top of the scale.
C. Indication will remain constant but will increase in a climb.
Answer (A) is correct. If the ram air input to the pitot head is blocked by ice but the drain hole and static port
remain open, the pressure in the line to the airspeed indicator will vent out through the drain hole. Thus, the
airspeed indication will drop to zero.
Answer (B) is incorrect because the airspeed indication will drop to zero, not rise to the top of the scale, if
only the ram air input is blocked.
Answer (C) is incorrect because the pressure in the airspeed line will vent out through the vent hole, and
the indication will drop to zero. It will not act as an altimeter.
[310]
The adverse effects of ice, snow, or frost on aircraft performance and flight characteristics include decreased lift and
A. increased thrust.
B. a decreased stall speed.
C. an increased stall speed.
Answer (A) is incorrect because the adverse effects of ice, snow, or frost on aircraft performance and flight
characteristics include a decrease, not an increase, in thrust.
Answer (B) is incorrect because the adverse effects of ice, snow, or frost on aircraft performance and flight
characteristics include an increase, not a decrease, in stall speed.
Answer (C) is correct. The adverse effects of ice, snow, or frost on aircraft performance and flight
characteristics are generally reflected in the form of decreased lift, increased stall speed, decreased thrust,
trim changes, and altered stall characteristics and handling qualities.
[311]
Which is an effect of ice, snow, or frost formation on an airplane?
A. Decreased stall speed.
B. Decreased pitchup tendencies.
C. Decreased angle of attack for stalls.
Answer (A) is incorrect because the formation of ice, snow, or frost on the wings of an aircraft increases, not
decreases, stall speed.
Answer (B) is incorrect because the formation of ice, snow, or frost on the wings of some aircraft may
increase, not decrease, pitchup tendencies.
Answer (C) is correct. When ice, snow, or particularly frost forms on the upper surface of a wing, it provides
a surface texture of considerable roughness. The increase in surface roughness increases skin friction drag
and reduces the kinetic energy of the boundary layer, causing incipient stalling of the wing; i.e., separation
will occur at angles of attack lower than for the clean, smooth wing. In some aircraft, stall will occur prior to
activation of stall warning devices.
[312]
Which is an effect of ice, snow, or frost formation on an airplane?
A. Increased stall speed.
B. Increased pitchdown tendencies.
C. Increased angle of attack for stalls.
Answer (A) is correct. When ice, snow, or particularly frost forms on the upper surface of a wing, it provides
a surface texture of considerable roughness. The increase in surface roughness increases skin friction drag
and reduces the kinetic energy of the boundary layer, causing an increase in power required and stall
speed.
Answer (B) is incorrect because the formation of ice, snow, or frost on the wings of some aircraft may
increase pitchup, not pitchdown, tendencies.
Answer (C) is incorrect because the formation of ice, snow, or frost on the wings of an aircraft decreases,
not increases, the angle of attack for stalls. In some aircraft, stall will occur prior to activation of stall warning
devices.
[313]
Which of the following is considered a primary flight control?
A. Slats.
B. Elevator.
C. Dorsal fin.
Answer (A) is incorrect because slats are high-lift devices, not a flight control device.
Answer (B) is correct. The primary flight control surfaces all ow the pilot to control an airplane's movement
around its lateral, longitudinal, and vertical axes. The primary flight controls are the elevator, ailerons, and
rudder.
Answer (C) is incorrect because a dorsal fin is not a flight control but is used to provide directional stability.
[314]
Which of the following are considered primary flight controls?
A. Tabs.
B. Flaps.
C. Outboard ailerons.
Answer (A) is incorrect because tabs are secondary, not primary, flight controls.
Answer (B) is incorrect because flaps are secondary, not primary, flight controls.
Answer (C) is correct. The primary flight control surfaces allow the pilot to control an airplane's movement
around its lateral, longitudinal, and vertical axes. The primary flight controls are elevators, ailerons, and
rudders.
[315]
Which of the following is considered an auxiliary flight control?
A. Ruddervator.
B. Upper rudder.
C. Leading-edge flaps.
Answer (A) is incorrect because a ruddervator is a primary, not auxiliary, flight control surface that
incorporates both a rudder and an elevator into one surface. This is done on "V"-tail-type airplanes.
Answer (B) is incorrect because upper rudders (found on the B-727) are standby rudders, which are used in
the event of a hydraulic system failure. These are primary, not auxiliary, flight controls.
Answer (C) is correct. Leading-edge flaps, speed brakes, spoilers, trailing-edge flaps, and leading-edge
slats are auxiliary (secondary) wing flight control surfaces.
[316]
When are inboard ailerons normally used?
A. Low-speed flight only.
B. High-speed flight only.
C. Low-speed and high-speed flight.
Answer (A) is incorrect because the inboard ailerons are used during both high- and low-speed flight.
Answer (B) is incorrect because the inboard ailerons are used during both high- and low-speed flight.
Answer (C) is correct. Some large aircraft use two sets of ailerons (inboard and outboard). At high
airspeeds, the aerodynamic loads from use of the outboard ailerons twist the wingtips. The outboard
ailerons are locked in the neutral position during high-speed flight.
[317]
When are outboard ailerons normally used?
A. Low-speed flight only.
B. High-speed flight only.
C. Low-speed and high-speed flight.
Answer (A) is correct. Some large aircraft use two sets of ailerons (inboard and outboard). At high
airspeeds, the aerodynamic loads from use of the outboard ailerons twist the wingtips. The outboard
ailerons are locked in the neutral position during high-speed flight.
Answer (B) is incorrect because the outboard ailerons are locked during high-speed flight.
Answer (C) is incorrect because the outboard ailerons are locked during high-speed flight.
[318]
What is the purpose of a control tab?
A. Move the flight controls in the event of manual reversion.
B. Reduce control forces by deflecting in the proper direction to move a primary flight control.
C. Prevent a control surface from moving to a full-deflection position due to aerodynamic forces.
Answer (A) is correct. When hydraulic pressure is lost to the primary controls, the control tabs unlock from
their actuating cylinders and allow control wheel inputs. This method of secondary control is known as
manual reversion.
Answer (B) is incorrect because servo, not control, tabs reduce control forces by deflecting primary flight
controls in the proper direction.
Answer (C) is incorrect because anti-servo, not control, tabs prevent a control surface from moving to a full-
deflection position due to aerodynamic forces.
[319]
What is the purpose of an elevator trim tab?
A. Provide horizontal balance as airspeed is increased to allow hands-off flight.
B. Adjust the speed tail load for different airspeeds in flight allowing neutral control forces.
C. Modify the downward tail load for various airspeeds in flight eliminating flight-control pressures.
Answer (A) is incorrect because the elevator trim tab permits hands-off flight at any airspeed, not only when
the airspeed is increasing, and provides longitudinal, not horizontal, balance.
Answer (B) is incorrect because the elevator trim tab adjusts the downward, not speed, tail load for various
airspeeds in flight allowing neutral control forces.
Answer (C) is correct. The elevator trim tab acts on the elevators, which in turn act upon the entire airplane.
This allows the pilot to adjust the angle of attack of the elevators, thus modifying the downward tail load for
any airspeed in flight and eliminating the need to exert continuous flight-control pressures.
[320]
Which direction from the primary control surface does an elevator adjust able trim tab move when the control
surface is moved?
A. Same direction.
B. Opposite direction.
C. Remains fixed for all positions.
Answer (A) is incorrect because adjustable trim tabs are adjusted in the opposite direction of the primary
control surface as a means of trimming the control surface. Once adjusted, their relationship remains
constant with respect to the primary control surface.
Answer (B) is incorrect because once adjusted, trim tabs remain fixed to the primary control surface as the
primary control surface is moved.
Answer (C) is correct. Once adjusted, an elevator adjustable trim tab remains fixed for all positions relative
to the primary control surface as the primary control surface is moved.
[321]
What is the purpose of a servo tab?
A. Move the flight controls in the event of manual reversion.
B. Reduce control forces by deflecting in the proper direction to move a primary flight control.
C. Prevent a control surface from moving to a full-deflection position due to aerodynamic forces.
Answer (A) is incorrect because, in the event of manual reversion on some transport category aircraft, the
control, not servo, tabs would move the flight controls.
Answer (B) is correct. The servo tab aids the pilot in moving a primary control surface and in holding it in the
desired position.
Answer (C) is incorrect because to preclude full deflection of control surfaces is the purpose of the anti-
servo, not servo, tab.
[322]
Which direction from the primary control surface does a servo tab move?
A. Same direction.
B. Opposite direction.
C. Remains fixed for all positions.
Answer (A) is incorrect because an anti-servo tab, as found on the trailing edge of stabilators, moves in the
same direction as the stabilator to provide a feel to the pilot control pressures.
Answer (B) is correct. Movement of the servo tab in one direction causes the primary control surface to
move in the opposite di rection. If a servo tab on an aileron is deflected upward, the aileron is deflected
downward, increasing the angle of attack on that wing, resulting in greater lift on that wing.
Answer (C) is incorrect because servo tabs move in response to the pilot' s control movements.
[323]
What is the purpose of an anti -servo tab?
A. Move the flight controls in the event of manual reversion.
B. Reduce control forces by deflecting in the proper direction to move a primary flight control.
C. Prevent a control surface from moving to a full-deflection position due to aerodynamic forces.
Answer (A) is incorrect because, in the event of manual reversion, the control, not anti-servo, tabs would
move the flight controls.
Answer (B) is incorrect because a servo, not an anti-servo, tab reduces control forces by deflecting in the
proper direction to move a primary flight control.
Answer (C) is correct. Since the anti-servo tab moves in the same direction as the control surface, it
produces a stabilizing force that increases control force pressures.
[324]
Which direction from the primary control surface does an anti-servo tab move?
A. Same direction.
B. Opposite direction.
C. Remains fixed for all positions.
Answer (A) is correct. An anti-servo tab, as found on the trailing edge of a stabilator, moves in the same
direction as the primary flight control surface.
Answer (B) is incorrect because a servo, not an anti-servo, tab moves in the opposite direction from the
primary control surface.
Answer (C) is incorrect because trim, not anti-servo, tabs remain fixed during control inputs.
[325]
Which is a purpose of leading-edge flaps?
A. Increase the camber of the wing.
B. Reduce lift without increasing airspeed.
C. Direct airflow over the t op of the wing at high angles of attack.
Answer (A) is correct. The leading-edge flap is similar in operation to the plain flap. When actuated, the
leading edge of the wing extends in a downward direction to increase the camber of the wing, providing
greater lift at low flight speeds.
Answer (B) is incorrect because leading-edge flaps increase, not decrease, the lift at slow airspeeds.
Answer (C) is incorrect because a slot, not a leading-edge flap, directs airflow over the top of the wing at
high angles of attack.
[326]
What is a purpose of flight spoilers?
A. Increase the camber of the wing.
B. Reduce lift without increasing airspeed.
C. Direct airflow over the top of the wing at high angles of attack.
Answer (A) is incorrect because flaps, not spoilers, increase the camber of the wing.
Answer (B) is correct. Spoilers are mounted on the upper surface of each wing, and they reduce lift without
increasing airspeed. Their purpose is to spoil or disrupt the smooth flow of air over the wing to reduce the
lifting force of the wing, thus increasing the rate of descent without increasing airspeed.
Answer (C) is incorrect because slots and slats, not spoilers, direct airflow over the top of the wing at high
angles of attack.
[327]
For which purpose may flight spoilers be used?
A. Reduce the wings' lift upon landing.
B. Increase the rate of descent without increasing aerodynamic drag.
C. Aid in longitudinal balance when rolling an airplane into a turn.
Answer (A) is correct. The purpose of spoilers is to spoil or disrupt the smooth flow of air over the wing to
reduce the lifting force of the wing. Thus, flight spoilers may be used to reduce the wings' lift upon landing.
Answer (B) is incorrect because spoilers will increase the aerodynamic drag.
Answer (C) is incorrect because longitudinal balance or stability is primarily achieved by the horizontal
stabilizer, not a spoiler.
[328]
Which is a purpose of ground spoilers?
A. Reduce the wings' lift upon landing.
B. Aid in rolling an airplane into a turn.
C. Increase the rate of descent without gaining airspeed.
Answer (A) is correct. Spoilers are designed to spoil or disrupt the smooth flow of air over the wing to
reduce the lifting force of the wing. Thus, ground spoilers are used to reduce the wings' lift upon landing.
Answer (B) is incorrect because a flight spoiler, not a ground spoiler, can be used to roll an airplane into a
turn.
Answer (C) is incorrect because flight spoilers, not ground spoilers, are used to increase the rate of descent
without gaining airspeed.
[329]
A definition of the term "viscous hydroplaning" is where
A. the airplane rides on standing water.
B. a film of moisture covers the painted or rubber-coated portion of the runway.
C. the tires of the airplane are actually riding on a mixture of steam and melted rubber
Answer (A) is incorrect because dynamic hydroplaning occurs when the airplane tires ride on standing
water.
Answer (B) is correct. Viscous hydroplaning can occur on a thin film of water if it covers the painted or
rubber-coated portion of the runway.
Answer (C) is incorrect because reverted rubber hydroplaning occurs when the tires of the airplane are
actually riding on a mixture of steam and melted rubber.
[330]
The primary purpose of high-lift devices is to increase the
A. L/Dmax.
B. lift at low speeds.
C. drag and reduce airspeed.
Answer (A) is incorrect because increasing the lift component is an objective of high-lift devices which
increase the ratio of L/D. The primary purpose of high-lift devices is to increase lift at low speeds.
Answer (B) is correct. High-lift devices increase lift at low airspeeds, allowing lowered landing and takeoff
speeds.
Answer (C) is incorrect because increasing the drag to reduce airspeed is the function of spoilers, not high-
lift devices.
[331]
What effect does an uphill runway slope have upon takeoff performance?
A. Increases takeoff distance.
B. Decreases takeoff speed.
C. Decreases takeoff distance.
Answer (A) is correct. The upslope of a runway impedes acceleration, resulting in a longer ground run on
takeoff.
Answer (B) is incorrect because indicated airspeed for takeoff remains the same.
Answer (C) is incorrect because the takeoff distance is increased, not decreased.
[331]
Which performance factor decreases as airplane gross weight increases, for a given runway?
A. Critical engine failure speed.
B. Rotation speed.
C. Accelerate-stop distance.
Answer (A) is correct. Critical engine failure speed (i.e., takeoff decision speed or V1) is the speed to which
the airplane can be accelerated, lose an engine, and then either continue the takeoff with the remaining
engine(s) or stop, in the same total runway distance. Critical engine failure speed decreases as the airplane
gross weight increases due to the increased mass that must decelerate and stop in the event of a rejected
takeoff.
Answer (B) is incorrect because an increase in aircraft gross weight usually results in an increase in the
rotation speed (VR).
Answer (C) is incorrect because the accelerate-stop distance will increase as the aircraft gross weight
increases.
[332]
What is the principal advantage of a sweepback design wing over a straightwing design?
A. The critical Mach number will increase significantly.
B. Sweepback will increase changes in the magnitude of force coefficients due to compressibility.
C. Sweepback will accelerate the onset of compressibility effect.
Answer (A) is correct. The sweep of a surface produces a beneficial effect in high speed flight since higher
flight speeds may be obtained before components of speed perpendicular to the leading edge produce
critical conditions on the wing. One of the most important advantages of a sweepback wing design is an
increase in the critical Mach number.
Answer (B) is incorrect because sweepback will reduce, not increase, changes in the magnitude of force
coefficients due to compressibility.
Answer (C) is incorrect because sweepback will delay, not accelerate, the onset of compressibility effect.
[333]
What is one disadvantage of a sweptwing design?
A. The wing root stalls prior to the wingtip section.
B. The wingtip section stalls prior to the wing root.
C. Severe pitchdown moment when the center of pressure shifts forward.
Answer (A) is incorrect because a sweptwing design has a tendency for the wingtip, not wing root, to stall
first.
Answer (B) is correct. The use of sweptwing design produces certain disadvantages which are important
from the standpoint of both airplane design and flight operations. One disadvantage is that, when
sweepback is combined with taper, there is an extremely powerful tendency for the wingtip section to stall
prior to the wing root.
Answer (C) is incorrect because, when the center of pressure shifts forward, there is a pitch up, not down,
moment.